Inman Domain 1, 2, 3, 4 questions

Pataasin ang iyong marka sa homework at exams ngayon gamit ang Quizwiz!

Which statement best describes the "F" component of the FOCUS-PDSA performance improvement model? [MGT:306] a. Identify a process to improve b. Develop a team c. Explain the current process d. Maintain and continue improvement

Explain the current process

Which of the following situations reduces a manager's span of control? [MGT:324] a. Managing less complicated work b. Having a greater dispersion of workers c. Receiving expert advice from colleagues/superiors d. Using clear and comprehensive policies

Having a greater dispersion of workers

a food delivery arrived during tray line service. what should the dietitian do a. ask the driver to wait b. ask the driver to verify the shipment and sign the invoice c. direct the available security guard to verify the shipment d. stop the trayline service and receive the delivery

a. ask the driver to wait

in the nutrition care process, which step may also involve nursing staff a. assessment b. diagnosis c. intervention d. monitor and evaluate

a. assessment

what is the best practical way to measure energy balance a. evaluate weight gain and weight loss b. utilize food intake records c. use activity factors d. calculate the number of calories consumed

a. evaluate weight gain and weight loss

which of the following has the fewest calories a. plain hamburger on a bun b. chef's salad with dressing c. green salad with Thousand Island dressing, and shredded cheese d. fried fish sandwich

a. plain hamburger on a bun

A good source of vitamin A for pre-school children would be: a. grapes b. watermelon c. cantaloupe pieces d. banana

cantaloupe pieces

Glucocorticoids: a. convert glucose into protein b. convert fatty acids into glucose c. convert glucose into fat d. convert protein into glucose

convert protein into glucose

which is the least time efficient way for a dietitian to evaluate a new piece of equipment for purchase a. read specifications provided by the manufacturer b. discuss it with other dietitians via telephone c. meet with the company representatives d. attend food service trade fairs

d. attend food service trade fairs

which hormone is involved in developing the placenta a. progesterone b. estrogen c. lactogen d. testosterone

a. progesterone

in a TPN mixture, what volume of the amino acid solutions listed would come closest to providing 185 calories a. 1000 ml of 8.5% amino acids b. 850 ml of 5.5% amino acids c. 500 ml of 7% amino acids d. 500 ml of 10% amino acids

b. 850 ml of 5.5% amino acids

which of the following self-assessment tools would be given to clients prior to screening at a congregate meal site a. PEACH survey b. DETERMINE checklist c. NSI Level I screen d. NSI Level II screen

b. DETERMINE checklist (nutrition checklist for older adults; Disease, Eating poorly, Tooth loss, Economic, Reduced social contact, Multiple meds, Involuntary weight loss/gain, Needs assistance in ADLs, Elder years)

Following counseling, a client is able to prepare a low sodium diet. The learning domain is: a. affective b. cognitive c. psychomotor d. psychosocial

PSYCHOMOTOR

Margarine has fatty acids in this order of predominance: a. PUFA, MUFA, SFA b. SAT, MUFA, PUFA c. MUFA, SAT, PUFA d. MUFA, PUFA, SAT

PUFA, MUFA, SFA

364. Approximately how many grams of fat would be contained in an 800 kcal meal that provides 50% of the energy from carbohydrates, 20% from protein, and the remainder from fat? [NTR:659] a) 89 b) 240 c) 27 d) 296

c) 27

318. Approximately how many carbohydrate serving(s) are 48 grapes? [NTR:612] a) 1 b) 2 c) 3 d) 4

c) 3

When providing nutrition education to a pregnant teenager, which characteristic of adolescents should be kept in mind? a. interested in planning for the future b. responds more to emotional stimuli c. desire to be like their peers d. has well-defined self-image

desire to be like their peers

The kidneys respond to respiration acidosis by: a. increasing the secretion of magnesium b. exchanging sodium for calcium c. increasing the retention of bicarbonate d. exchanging potassium for hydrogen

increasing the retention of bicarbonate

Which nutrient is added for flour enrichment? a. vitamin A b. iodine c. fluorine d. iron

iron

The sentence "We will produce nutritious, high-quality food that meets our customers' needs for service within 30 minutes" is a [MGT:366] a. vision. b. mission statement. c. Strategy. d. value

mission statement.

A patient's nitrogen balance study showed he consumed 10 grams of protein and released 6 grams of nitrogen in the urine. This is: a. nitrogen balance b. negative nitrogen balance c. positive nitrogen balance d. plus 4 nitrogen balance

negative nitrogen balance

Where is lactose absorbed? a. small intestine b. stomach d. pancreas d. liver

small intestine

Which recipe ingredient should be adjusted if a cake has a crunchy crust and compact moist texture? a. milk b. flour c. sugar d. baking powder

sugar

121. The optimal length of time to breastfeed an infant is: [NTR:515] a) 3 months b) 6 months c) 9 months d) 12 months

d) 12 months

358. After a meal, most of the fat that travels from the intestines into the lymph and eventually the blood is in the form of particles known as: [NTR:653] a) VLDLs b) HDLs c) LDLs d) Chylomicrons

d) Chylomicrons

Which set of directives best describes the management process? [MGT:321] a) Controlling, directing, transforming, planning, staffing b) Planning, staffing, establishing objectives, organizing, directing c) Planning, staffing, controlling, establishing objectives, transforming d) Directing, controlling, organizing, staffing, planning

d) Directing, controlling, organizing, staffing, planning

281. Which micronutrient enhances the absorption of non-heme iron? [NTR:575] a) Calcium b) Copper c) Vitamin A d) Vitamin C

d) Vitamin C

345. Pyridoxine is another name for [NTR:640] a) vitamin C. b) vitamin B1. c) vitamin B12. d) vitamin B6.

d) vitamin B6.

what gas is released from a fire a. helium b. hydrogen c. carbon dioxide d. carbon monoxide

d. carbon monoxide

in cystic fibrosis, the most impaired nutrient is a. protein b. disaccharides c. electrolytes d. fat

d. fat

a pork chop is which type of meat exchange a. high fat b. medium fat c. medium fat plus one fat exchange d. lean

d. lean

which of the following menu items generate the most revenue? Menu Item Selling price Raw food cost Units sold shrimp alfredo $12.95 $4.05 75 chicken marsala $11.50 $3.83 193 prime rib $14.95 $4.39 172 lemon halibut $13.50 $4.29 210 a. shrimp alfredo b. chicken marsala c. prime rib d. lemon halibut

d. lemon halibut

what would be an expected outcome of WIC a. fewer women breast feed b. fewer babies, better birth control c. better educated low income women concerning nutiriton d. less anemia, increased HG and HT levels

d. less anemia, increased HG and HT levels

which would add bulk to frozen yogurt a. carrageenan b. carob gum c. monoglycerides d. microcrystalline cellulose

d. microcrystalline cellulose ( refined wood pulp and is used as a texturizer, an anti-caking agent, a fat substitute, an emulsifier, an extender, and a bulking agent in food production)

a home delivered meals program for the elderly chooses a nonselective menu primarily to a. control food volume b. encourage client participation c. minimize food drop off time d. minimize labor costs

d. minimize labor costs

what law guarantees workers the right to organize and join labor unions a. Taft Hartley b. equal employment act c. labor management relations act d. national labor relations act

d. national labor relations act

quality control is a. outcome or output oriented b. a measure of the quality of care provided c. patient care oriented d. proactive-focuses on the prevention of errors in food service

d. proactive-focuses on the prevention of errors in food service

which has the greatest effect on the renal solute load a. potassium b. sodium c. glucose d. protein

d. protein

an indication for use of a protein hydrolysate infant formula is a. prematurity b. constipation c. phenylketonuria d. protein hypersensitivity

d. protein hypersensitivity

what is the appropriate advice for a person with anorexia nervosa a. offer a wide variety of foods and frequent feedings b. offer liquids high in calories and protein c. offer nasogastric feedings d. regular mealtimes, and a varied moderate intake

d. regular mealtimes, and a varied moderate intake

a useful application of counting FTEs is in planning a. the menu b. the type of service c. the kitchen layout d. the budget for wages

d. the budget for wages

based on the DRGs, Medicare reimbursement depends on a. length of patient stay b. age of patient c. primary diagnosis d. a e. b and c f. a and b g. a, b, and c

e. b and c

which of the following should be included in the diet for celiac disease a. decrease intake of eggs b. use potato and rice flour c. IM vitamin B12 and iron d. low calorie, high fat, high protein e. b and c f. c and d g. a, b and d h. b and d

e. b and c (use potato and rice flour and IM vitamin B12 and iron)

Which of the following combines CoA in two carbon fragments to make acetyl CoA to then enter the Krebs cycle to make energy? a. lactic acid b. fatty acids c. glucose d. fructose

fatty acids

Gastric preteolysis requires: a. pyridoxine b. ascorbic acid c. hydrochloric acid d. the intrinsic factor

hydrochloric acid

The most effective way to increase the incidence and duration of breast-feeding among clients receiving prenatal care is to: a. post breastfeeding posters in the waiting rooms b. conduct prenatal breast-feeding classes c. show breast-feeding videotapes in the waiting room d. implement a breast-feeding peer counselor program

implement a breast-feeding peer counselor program

Which would be best when preparing staff for a new way to deliver trays? a. include them during discussions of proposed methods b. introduce the new method quickly c. let them know it is a better process d. ask employees if they will go along with the decision

include them during discussions of proposed methods

When substituting buttermilk for whole milk in pastry: a. decrease the liquid b. increase the baking soda c. increase the amount of butter d. increase the amount of butter

increase the baking soda

A client states he can no longer return to the clinic for counseling due to a change in jobs. To ensure continued compliance with the diet, you should: a. give additional practice to ensure thorough understanding b. instruct him to call you if he has a problem c. inform him about other resources for follow-up counseling d. provide several educational pamphlets regarding his diet

inform him about other resources for follow-up counseling

What is the effect of excess caffeine intake? a. insomnia b. nausea c. bloating d. weight gain

insomnia

What does the term "USDA graded" on meat tell the consumer? a. it assures wholesomeness b. it was free of disease at the time of slaughter c. the mat is for for human consumption d. it defines quality

it defines quality

You want to keep lemon meringue pie at is peak. What should you do? a. add sugar with eggs b. add flour with butter c. keep the pie at room temperature d. keep the pie refrigerated

keep the pie refrigerated

The purpose of an emulsifier in margarine is to: a. keep water and fat from separating b. keep water and fat separated c. sustain the natural flavor of the ingredients d. retain the color

keep water and fat from separating

In starvation, the brain uses which nutrient as an energy source? a. fatty acid b. lactic acid c. glucose d. ketone bodies

ketone bodies

Bile is produced in the ______ and stored in the ______. a. liver, gallbladder b. gallbladder, liver c. liver, liver d. gallbladder, gallbladder

liver, gallbladder

Patients with early signs of xerophthalmia should consume? a. chicken, fish, summer squash b. liver, milk, eggs c. oatmeal, almonds, berries d. high fiber cereals, salad greens

liver, milk, eggs

Acidic chyme becomes neutralized in the duodenum by: a. mixing with bicarbonate and fluids b. mixing with alkaline intestinal enzymes c. mixing with sodium and potassium d. mixing with bile

mixing with bicarbonate and fluids

A child is at the 95th percentile BMI for age. He is: a. normal b. obese c. overweight d. tall for his age

obese

You are asked to give an inservice to the staff. What should you do first? a. ask the what they want to learn b. read up on the subject c. observe the staff at work to identify problems d. see what other hospitals are teachings

observe the staff at work to identify problems

Energy expenditure of humans can be measured by: a. heat intake b. food intake c. nitrogen output d. oxygen consumption

oxygen consumption

Trypsin and chymotrypsin are: a. pancreatic enzymes that provide proteolytic action b. intestinal enzymes that digest starch c. liver hormones involved in fat digestion d. intestinal enzymes that digest protein

pancreatic enzymes that provide proteolytic action

The best way to improve communication with participants in a nutrition education group would be to focus on: a. paraphrasing b. internalizing c. empathizing d. referencing

paraphrasing

Which hormone regulates calcium levels? a. insulin b. thyroid c. parathyroid c. aldosterone

parathyroid

Which has the lowest cholesterol content? a. peanut butter and crackers b. yogurt with fruit c. chocolate cake d. roast beef sandwich

peanut butter and crackers

Green peas held on the serving line may turn olive green due to: a. pheophytin b. chlorophyllin c. anthoxanthins d. flavones

pheophytin

To prevent frozen gravy from separating when thawed: a. reheat slowly and stir constantly b. thaw it before heating and stir constantly c. prepare it with modified starch d. cook it longer before freezing to stabilize the gravy

prepare it with modified starch

Inferential statistics are associated with: a. probability b. mean c. mode d. graphs

probability

The end product of aerobic glycolysis is: a. acetyl-CoA b. pyruvic acid c. lactic acid d. glycogen

pyruvic acid

The following is an irreversible reaction: a. pyruvic acid into lactic acid b. pyruvic acid into acetyl CoA c. glucose-6-phosphate into glycogen d. glucose-6-phosphate into pyruvate

pyruvic acid into acetyl CoA

The hormone epinephrine: a. lowers blood glucose during catabolic stress b. raises blood glucose during catabolic stress c. is secreted from the pituitary gland d. stimulates glycogen production

raises blood glucose during catabolic stress

Salmonella sources are often: a. reheated foods b. starchy foods and mixtures like casseroles c. cooled slowly and reheated foods d. raw produce and uncooked foods of animal origin

raw produce and uncooked foods of animal origin

Which foods should be washed just before serving? a. plums and grapes b. peaches and carrots c. strawberries and mushrooms d. applies and celery

strawberries and mushrooms

The tatrness of fruits and vegetables is due to: a. flavones b. chlorophyll c. anthoxanthins d. tannins

tannins

A foodservice director using a systems approach to evaluate the effect of a new tray delivery service on the dishroom and the nurse stations is considering: [MGT:382] a. the internal and external environments. b. equifinality of the system. c. dynamic equilibrium of the system d. managerial feedback.

the internal and external environments.

An amino acid that cannot be produced by the body is: a. alanine b. glycine c. tyrosine d. tryptophan

tryptophan

Phenylalanine is precursor for: a. serotonin b. catecholamines c. tryptophan d. tyrosine

tyrosine

Which is the dependent variable? a. leavening b. storage days c. volume d. oven temperature

volume

which hormone regulates breast milk production a. prolactin b. progesterone c. estrogen d. oxytocin

a. prolactin

in severe cirrhosis with ascites, diet therapy should be a. restrict sodium b. restrict CHO and fat c. restrict protein and sodium d. restrict fat

a. restrict sodium

productivity is measured by a. the number of portions served per hour b. the number of employees working each day c. the number of food deliveries d. the inventory

a. the number of portions served per hour

to calculate the estimated caloric needs of a patient who is significantly underweight the Dietitian should use a. the patients actual body weight b. the patients adjusted body weight c. the patients desirable body weight d. 90% of the patients desirable body weight

a. the patients actual body weight

Which of the following stabilizes an egg white foam? a. an acid b. water c. flour d. milk

an acid

In the human, vitamin K is affected by: a. anticoagulants and antibodies b. iodine c. water-soluble vitamins d. gluten

anticoagulants and antibodies

Before you teach a class on nutrition, you should: a. research all current materials at the library b. read a current nutrition textbook c. take a survey to determine what they are interested in c. assess the student's current nutrition knowledge

assess the student's current nutrition knowledge

A focus group contributes what data? a. analytical b. experimental c. attitudinal d. objective

attitudinal

38. Overweight is defined as a BMI of: [NTR:433] a) 19 to 24. b) 25 to 29. c) 27 to 32. d) 31 to 36.

b) 25 to 29.

259. _____ have the highest prevalence rates of diabetes of all the racial and ethnic groups in the United States. [NTR:553] a) Chinese Americans b) American Indians and Alaska Natives c) Jews c) Muslims

b) American Indians and Alaska Natives

The DASH diet plan has been shown to be beneficial for which medical condition? [NTR:397] a) Diabetes b) Hypertension c) Heart failure d) Chronic Obstructive Pulmonary Disease (COPD)

b) Hypertension

70. When using the Nutrition Care Process, what does the dietetics professional use to describe alterations in the patient's/client's nutritional status? [NTR:465] a) Cause/Contributing risk factors b) Problem or nutrition diagnosis labels c) Signs/symptoms d) Knowledge, attitudes, and beliefs

b) Problem or nutrition diagnosis labels

342. Which set of Dietary Intake Values establishes a goal for dietary intake that will meet the needs of nearly all healthy people? [NTR:637] a) Estimated Average Requirement b) Recommended Dietary Allowance c) Adequate Intake d) Tolerable Upper Intake Level

b) Recommended Dietary Allowance

222. Which of the following foods is not a common pediatric food allergen? [NTR:516] a) Cow's milk b) Rice c) Wheat d) Peanuts

b) Rice

according to the diabetic exchange lists, when substituting 2% milk for skim milk, you must omit how many fat exchanges a. 1/2 b. 1 c. 1 1/2 d. 2

b. 1 (skim milk has 0-3g fat and low-fat/2% has 5g fat)

the use of a sugar substitute is regulated by a. NIH b. FDA c. WHO d. FTC

b. FDA

which is the primary fuel used by resting muscle and at low intensity exercise a. glucose b. fatty acids c. branch chain amino acids d. keto-acids

b. fatty acids

the food service supervisor should carefully monitor a cook-chill production because of concerns about a. work simplification techniques b. food safety c. portion control d. product acceptability

b. food safety

the additive influence of foods and constituents which when eaten have a beneficial effect on health refers to a. nutraceuticals b. food synergy c. positive corollaries d. medical foods

b. food synergy

a patient is recovering from an ileocecal resection, where 100cm was removed. what are your recommendations a. oral vitamin B12, increase fluids b. parenteral vitamin B12, increase fluids, increase protein c. decrease fluids, increase protein, parenteral B12 d. decrease protein, decrease fluids, oral B12

b. parenteral B12, increase fluids, increase protein

four patients are admitted to your service. who would you address first a. patient who had a recent MI b. patient with gastric distress c. patient with cancer d. patient admitted for an appendectomy

b. patient with gastric distress

which of the following would be used to analyze tray line accuracy a. Gantt chart b. process control chart c. cause and effect diagram d. break even analysis

b. process control chart

which of the following items is appropriate for a Muslin diet a. sweet and sour pork b. salad with Bacos c. pork fried rice d. chicken in wine sauce

b. salad with Bacos

what elements of the diet vary the most in hepatic disease a. fat, carbohydrate, potassium b. sodium, fluid, protein c. sodium, potassium, protein d. fat, sodium, potassium

b. sodium, fluid, protein

the allowance for food stamps is based on a. the average cost of food used by a family of the same size b. the USDA thrifty food program c. data from the congregate meal program d. the cost of the basic four

b. the USDA thrifty food program

the best way to justify continuation of WIC would be to show improvement in a. the number of participants enrolled b. the infant morbidity rate c. maternal weight gains during pregnancy d. infant head circumference

b. the infant morbidity rate

the VMA test diet measures a. fat malabsorption b. total catecholamine's c. protein malabsorption d. vitamin, mineral absorption

b. total catecholamine's (measures the amount of catecholamine's - epinephrine, norepinephrine, dopamine - in the urine, VMA is the breakdown product of these catecholamine's)

which of the following is permitted on a diet restricting tyramine a. hard cheese like cheddar and swiss b. uncured cheese like cottage, ricotta c. pickled or smoked foods d. tofu

b. uncured cheese like cottage, ricotta

Glycogenolysis is the: a. catabolism of carbohydrate b. creation of glucose from glycerol and amino acids c. breakdown of glycogen releasing glucose d. creation of glycogen

breakdown of glycogen releasing glucose

330. In the ETHNIC model of cultural competent care, the "E" stands for _____. [NTR:624] a) Education b) Empathy c) Explanation d) expectations

c) Explanation

18. Which foods cause the most food allergies? [NTR:413] a) Cheese, strawberries, and beef b) Fish, rice, and corn c) Milk, eggs, and peanuts d) Wheat, tomatoes, and chicken

c) Milk, eggs, and peanuts

51. Interpretation of BMI for children is [NTR:446] a) age specific. b) gender specific. c) age and gender specific. d) the same as adults.

c) age and gender specific.

276. The oils that contain predominantly monounsaturated fatty acids include [NTR:570] a) sunflower oil and canola oil. b) safflower oil and corn oil. c) canola oil and olive oil. d) soybean and olive oil.

c) canola oil and olive oil.

307. One part of the brain that is critical to regulating hunger and satiety is the _____. [NTR:601] a) cerebral cortex b) cerebellum c) hypothalamus d) medulla

c) hypothalamus

if you purchase an oven for $10000, it is expected to last for 10 years, after which it can be sold for $3000. what is its depreciation cost for each year a. $2000 b. $1500 c. $700 d. $500

c. $700 (10000 - 3000 = 7000/10years = 700)

the management function in which performance is measured and corrective action is taken to ensure accomplishment of an organizational goal is known as a. planning b. directing c. controlling d. organizing

c. controlling

which of the following food service systems has the lowest labor costs a. cook freeze b. cook chill c. convenience d. conventional

c. convenience aka assembly serve

what is a major reason for constipation in the elderly a. a diet low in high residue foods b. a diet high in low residue foods c. decreased gastric motility d. lack of physical exercise

c. decreased gastric motility

why are mono- and di-glycerides added to the shortening used in cakes a. increase spreadability b. thickeners c. emulsifiers d. coloring agents

c. emulsifiers

a side effect of antidepressants is a. chronic diarrhea b. muscle wasting c. excess weight gain d. nausea and vomiting

c. excess weight gain

which test helps you determine if a diabetic has been following his diet a. blood glucose b. urine ketones c. glycosylated hemoglobin d. urine sugar

c. glycosylated hemoglobin

increased serum uric acid levels may lead to a. galactosemia b. glycogen storage disease c. gout d. PKU

c. gout

a baby is treated with an UV light for hyperbilirubinemia. you should prescribe a diet that is a. high calorie b. high protein c. high fluid d. high fat

c. high fluid (high fluid levels are used in treatment to dilute/decrease bilirubin levels)

a foodservice is being converted from a conventional system to a ready-prepared system. the dietitian should first consider the adequacy of the a. steam table space b. dry storage space c. low-temperature storage capacity d. oven capacity

c. low-temperature storage capacity

which is most associated with the incidence of type 2 diabetes a. sugar intake b. protein intake c. overweight d. salt intake

c. overweight

a Crohn's patient who was previously well-nourished has a fever and ileus 7 days PO. which feeding method should you recommend a. central venous hyperalimentation b. full liquids c. peripheral IV infusion d. half-strength tube feeding

c. peripheral IV infusion

a tube fed patient develops diarrhea. what needs to be done first a. review their nutritional needs b. increase the rate of the feeding c. review the osmolality of the feeding d. discontinue the feeding until the diarrhea stops

c. review the osmolality of the feeding

What ratio measures the ability of an organization to generate excess income in relation to sales? [MGT:355] a. Profitability b. Solvency c. Liquidity d. Activity

Profitability

Which statement best describes the "C" component of the FOCUS-PDSA performance improvement model? [MGT:308] a. Identify a process to improve b. Develop a team c. Explain the current process d. Recognize sources of variation

Identify a process to improve

What type of budget uses the existing budget as a base and projects changes for the ensuing year in relation to the current budget? [MGT:356] a. Fixed b. Incremental c. Flexible d. Sustained

Incremental

Which of the following statements is accurate concerning fruit juice [NTR:417] a) Intake of juice should be limited to 4-6 oz per day for children ages 1 to 6 years b) Juice may be introduced at 4 months of age. c) Juice should be given in the bedtime bottle to help infants sleep all night. d) It is safe to give young children unpasteurized juice.

Intake of juice should be limited to 4-6 oz per day for children ages 1 to 6 years

What characteristic is KEY to an efficient and effective system? [MGT:315] a. Continuous response and adaptation to the system b. Flow of resources c. Interdependency of parts d. Achievement of subunit objectives

Interdependency of parts

An angel food cake is make without cream of tarter. What is the result? a. a yellowy cake b. a bright white cake c. a cake with large volume d. a very tender cake

a yellowy cake

55. A general rule for appropriate portion sizes for toddlers and preschool children is to offer [NTR:450] a) 1 Tbsp of each food per meal for each year of the child's age. b) ¼ to ½ cup of each of three foods per meal. c) to need only breast milk or iron-fortified formula for the first 6 months of age. d) the same portions as adults.

a) 1 Tbsp of each food per meal for each year of the child's age.

353. A food provides 28 grams of carbohydrates, 8 grams of protein, and 7 grams of fat. What percent of this food's calories comes from fat? [NTR:648] a) 30% b) 33% c) 36% d) 38%

a) 30%

286. Dietetics professionals can expect more than one in _____ of their clients to have a disability that will require special accommodations. [NTR:580] a) 5 b) 15 c) 25 d) 50

a) 5

305. Which of the following accurately describes how alcohol is digested and absorbed? [NTR:599] a) Alcohol requires no digestion and is absorbed by simple diffusion. b) Alcohol is digested in the stomach and absorbed primarily from the small intestine. c) Alcohol is digested in and absorbed primarily from the stomach. d) Alcohol requires no digestion and is absorbed by active transport.

a) Alcohol requires no digestion and is absorbed by simple diffusion.

252. Which of the following nutrients should a pregnant woman avoid excessive intake of? [NTR:546] a) Vitamin A b) Vitamin B6 c) Folic acid d) Iron

a) Vitamin A

352. Which of the following nutrients is required to be included as a Percent Daily Value on food labels? [NTR:647] a) Vitamin A b) Potassium c) Vitamin D d) Iodine

a) Vitamin A

58. Which set of nutrients and/or food components are associated with the formation of cataracts in older adults? [NTR:453] a) Vitamin C, vitamin E, carotenoids b) Thiamin, niacin, riboflavin c) Vitamin C, vitamin E, phytosterols d) Calcium, zinc, magnesium

a) Vitamin C, vitamin E, carotenoids

310. Which of the following individuals would be expected to have the highest basal metabolic rate? [NTR:604] a) a 25 years old male who is 6' tall and weighs 200# b) a 40 years old male who is 6' tall and weighs 204# c) a 30 years old female who is 5'7" tall and weighs 200# d) a 50 years old female who is 5'3" tall and weighs 120#

a) a 25 years old male who is 6' tall and weighs 200#

270. Refeeding syndrome is characterized by low serum concentrations of: [NTR:564] sodium, phosphorus, magnesium. sodium, potassium, phosphorus phosphorus, potassium, magnesium potassium, magnesium, sodium.

a) phosphorus, potassium, magnesium.

268. When completing a physical assessment, in adequate hydration may present as [NTR:562] a) poor skin turgor b) scaly appearance of skin c) dull and thin hair d) angular stomatitis

a) poor skin turgor

using the following information, determine the selling price of the meal used to determine the break-even point fixed costs: $100, variable costs per meal: $5.00, break even point: 20 meals a. $10 b. $15 c. $5 d. $2.50

a. $10 ($5.00 variable cost x 20 meals + 100 fixed cost = 200/20(# of meals to BE)=10)

financial data for 1 month is shown. what is the net profit Cafeteria revenue $119984 patient meals revenue $146250 catering sales $1850 food costs $110234 labor costs $147446 other costs $10204 a. $200 b. $10404 c. $167850 d. $268084

a. $200

your nutrition department has 6.5 FTE dietitians at $20/hour. how much salary is that per week a. $5200 b. $3960 c. $4850 d. $6550

a. $5200 (6.5 FTE x 40hrs x $20)

what is the gross food cost for June using the following information June 1 inventory $5300 July 1 inventory $2700 Food purchases - month of June $3150 a. $5750 b. $3150 c. $4900 d. $600

a. $5750 (5300-2700 +3150)

find the cost per meal if you serve 120,000 meals, spend $46000 in food purchases and $1250 worth of food was removed from the inventory a. 0.39 b. 0.42 c. 0.37 d. 0.43

a. 0.39 (47250/120000=0.3938)

how many minutes are needed to sanitize dishes when washing by hand using chlorine a. 1 b. 2 c. 3 d. 4

a. 1

a diabetic wants to exchange his bedtime snack of 8ounces whole milk. what do you suggest a. 1oz cheese and 1/2 cup orange juice b. 2 graham crackers and 1/2 cup orange juice c. 1 oz cheese d. 2 oz cheese

a. 1 oz cheese and 1/2 cup orange juice

which of the following is equivalent to a very lean meat exchange a. 1/4 cup low fat cottage cheese b. 2 T grated parmesan cheese c. 1 ounce mozzarella cheese d. 1 ounce swiss cheese

a. 1/4 cup low fat cottage cheese

a tube feeding ran from 7pm til 6am, providing 1800 calories, at 1.6 calories/cc. how many cc/hour were given a. 100 b. 125 c. 219 d. 65

a. 100

how many calories are provided in 1.5 liters of D25W a. 1275 b. 2275 c. 1500 d. 2000

a. 1275

an AIDS patient's temperature has risen to 102 degrees. his BMR has increased a. 24% b. 36% c. 12% d. 5%

a. 24% (102-98.6 x 7)

a $240 income was generated selling 40 pieces of fish with a raw food cost of $2.00 each. what is the % food cost a. 33% b. 40% c. 20% d. 10%

a. 33% (2 x 40=80/240=33.34%)

how many non-protein calories are provided by 2 liters of a 7% aminosyn solution and 1 liter of 10% dextrose in water a. 340 b. 390 c. 170 d. 255

a. 340

A meal providing 1200 kcal contains 13 grams of saturated fat, 10 grams of monounsaturated fats, and 27 grams of polyunsaturated fats. what is the percentage of energy supplied by the lipid. [NTR:660] a. 37 b. 10 c. 17 d. 25

a. 37

how many calories are in 300ml of 5% dextrose solution a. 51 b. 60 c. 15 d. 200

a. 51 (300ml x 5% x 3.4 = 51)

a scale drawing of an area showing the path of a worker during a process is a. a flow diagram b. a process chart c. an operations chart d. a cross chart

a. a flow diagram

for which disease would a low sodium diet be recommended a. ascites b. esophageal varices c. pancreatitis d. appendicitis

a. ascites

which food is highest in fiber a. cooked dried beans b. a tomato c. wheat bread d. lettuce

a. cooked dried beans

an acid ash would be created by ingesting a. corn, cranberries b. milk c. vegetables d. fruits

a. corn, cranberries (create an acid ash by increasing anions - Cl, Ph, Su - by adding meats, fish, fowl, eggs, shellfish, cheese, corn, oats, rye)

what should be the final consideration in the make or buy decision of a product a. cost analysis b. labor requirements c. storage space available d. personal relationship with supplier

a. cost analysis

primary considerations in the purchase of foodservice equipment are a. cost and performance b. safety and versatility c. safety and performance d. cleanability and versatility

a. cost and performance

the primary reason for portion control in food service is a. cost control b. to standardize serving sizes c. customer satisfaction d. weight reduction diets

a. cost control

using the scientific approach to management, which of the following would result a. decreased labor minutes per meal b. decreased labor hours per week c. increased labor minutes per meal d. increased labor hours per week

a. decreased labor minutes per meal (work-centered; workers must work at fastest pace possible and at maximum efficiency; systematic approach to improving worker efficiency; find best way of performing tasks; focus on physical aspects of the job)

which item would appear in a capital budget a. food service equipment b. food costs c. labor costs d. laundry service

a. food service equipment (Capital budget includes plant facilities, equipment, cost of improvements and repairs, expansions, replacements)

which is an example of an indirect cost a. fringe benefits b. travel costs c. food d. china

a. fringe benefits (indirect costs: fixed and not affected by sales volume, required for business to exist even if produces nothing; rent, taxes, interest, insurance, depreciation etc.)

a strict Muslim would not consume a. fruited gelatin, shrimp, ham b. dairy products c. non-carbonated beverages d. eggs, cheese

a. fruited gelatin, shrimp, ham

a PEG feeding would be useful in a patient with a. head or neck cancer, who will not be able to eat normally for weeks b. short bowel syndrome c. duodenal ulcers d. pancreatitis

a. head or neck cancer, who will not be able to eat normally for weeks

you are planning a health promotion program for employees. to assess employee's needs, use information from a. health risk appraisals b. nutrition assessments c. medical records d. interviews

a. health risk appraisals

a director from another department sends you an email regarding construction. this is an example of what kind of communication a. horizontal b. downward c. straight d. informal

a. horizontal

a hospitalized patient who had not eaten well for 2 weeks had a 2.3 serum phosphorus level and a 1.3 serum magnesium level. he has a. hypophosphatemia, hypomagnesemia b. hyperphosphatemia, hypomagnesemia c. hyperphosphatemia, hyponatremia d. hyperkalemia, hypermagnesemia

a. hypophosphatemia, hypomagnesemia

what section of the intestine is most likely to be involved in Crohn's disease a. ileum b. jejunum c. duodenum d. appendix

a. ileum

a patient is on a high protein, low carbohydrate diet. what advice should you offer a. increase fluid intake b. decrease the intake of complex carbohydrates c. decrease fluid intake d. decrease the intake of saturated fats

a. increase fluid intake

fiber, like that found in oats, beans and bran cereals, results in a. increased water absorption from the intestine b. binding of bile acids c. decreased sodium excretion in the stool d. small, compact stools

a. increased water absorption from the intestine

if an employee is not performing well the first thing to do is to check the a. job description b. job specification c. job analysis d. work schedule

a. job description (reflects required skills and responsibilities; matches applicants to job, orientation and training, employee appraisal)

what medication is affected by serum sodium levels a. lithium b. mellaril c. valproate d. tetracycline

a. lithium

where should a class B fire extinguisher be placed a. near the range b. near the dishmachine c. near the freezer d. near the refrigerator

a. near the range (used for flammable liquids, gases, greases; made of either foam, carbon dioxide and/or dry chemicals)

when trying to cut down on the number of employees on a tray assembly line, evaluate the a. number of work stations b. amount of idle time c. line speed d. capacity

a. number of work stations

which set of conditions is most commonly seen in American children a. obesity, dental caries, anemia b. hypertension, dental caries, rickets c. hyperkinesis, anemia, dental caries d. diabetes, obesity, anemia

a. obesity, dental caries, anemia

hemodialysis requires supplementation of a. pyridoxine, folic acid, 1-25-dihydroxyvitamin D3 b. pyridoxine, folic and vitamin A c. vitamin C, calcium, 25-hydroxyvitamin D3 d. vitamin C, calcium and vitamin A

a. pyridoxine, folic acid, 1-25-dihydroxyvitamin D3

which diet is restricted in carbohydrate a. reactive hypoglycemia b. diabetes c. PKU d. glactosemia

a. reactive hypoglycemia

the profitability of a coffee shop may be determined by calculating the ratio of profit to a. sales b. labor costs c. liabilities d. indirect costs

a. sales

a single 19 year old female is 6 months pregnant. she has not gained any weight and has not seen a physician. hemoglobin is 9. hematocrit is 30. what is the most important recommendation for her a. see a physician immediately b. apply for WIC immediately c. join a teenage pregnancy support group d. begin taking iron supplements

a. see a physician immediately

a catered lunch is cancelled after the food has been prepared. what should you do a. sell the meals in the cafeteria at a break even price b. give meals to kitchen/cafeteria employees c. incorporate the items into the next meal service d. discard the items

a. sell the meals in the cafeteria at a break even price

which of the following directly assists the greatest number of people a. supplemental nutrition assistance program b. national school lunch program c. child and adult care program d. supplemental nutrition program for women, infants and children

a. supplemental nutrition assistance program

a cancer patient receiving radiation therapy to the head and neck presents with weight loss. the nutrition diagnosis would be a. swallowing difficulty b. involuntary weight loss c. impaired nutrient utilization d. hypermetabolism

a. swallowing difficulty (due to inability to swallow because of radiation therapy)

you have been promoted to a top management position. what adjustment in skills is needed a. technical skills decrease, conceptual skills increase b. conceptual skills decrease, technical and human skills increase c. technical skills increase, conceptual skills decrease d. technical skills decrease, conceptual and human skills increase

a. technical skills decrease, conceptual skills increase

medicare is for a. the elderly b. the financially needy c. those at nutritional risk d. those with medical needs

a. the elderly

a decision to feed patients in their rooms, rather than in the cafeteria is based on a. the philosophy of the institution b. the equipment in the kitchen c. the labor available d. federal regulations

a. the philosophy of the institution

a patient with dysphagia waiting for a swallow test may receive a. thick liquids b. thin liquids c. peripheral parenteral nutrition d. parenteral nutrition

a. thick liquids

a patient who had an ileal resection is placed on a tube feeding and begins to have diarrhea. this may be due to a. use of a hypertonic formula b. use of an isotonic formula c. blockage of the GI tract d. lactose intolerance

a. use of a hypertonic formula

99. Using the Hamwi formula, what is the ideal body weight for a female who is 70 inches tall and has a small body frame? [NTR:493] a) 120 pounds b) 135 pounds c) 150 pounds d) 165 pounds

b) 135 pounds

253. _______ can be defined as the accumulation of a group's learned and shared behaviors in everyday life. [NTR:547] a) ethnicity b) culture c) comportment d) society

b) culture

at what temperature should coffee and items such as thin soups be held for serving a. 140-160F b. 180-190F c. 200-212F d. over 212F

b. 180-190F

which drink would a Mormon choose a. cola b. 7-up c. tea d. coffee

b. 7-up

being well treated by supervisors, being given special attention, and the importance of breaks in increasing productivity, are all considered motivators by a. Herzberg b. Mayo c. Maslow d. MacGregor

b. Mayo (Hawthorne studies; involve people in the process and they become productive; due to special attention, involved in experiences, well-treated by supervisors, work breaks increase productivity; placebo effect - special attention improves behavior)

hypertension will decrease with a decrease in a. fiber b. alcohol c. potassium c. calcium

b. alcohol

acidic chyme in the duodenum is neutralized by pancreatic a. lipase b. bicarbonate c. amylase d. protease

b. bicarbonate

which of the following would likely be a cause of marasmus a. leaving a patient for two weeks on D5W b. chemotherapy c. following a 20 gram protein diet for two months d. antibiotic therapy

b. chemotherapy

in work simplification, which of the following tools is helpful a. labor turnover rate b. flow chart c. meals per labor minute d. meals per labor hour

b. flow chart

a vegan with macrocytic anemia needs which of the following a. iron rich foods b. fortified soy milk c. eggs, wheat bread d. meat, green vegetables

b. fortified soy milk

five pounds of pressure creates a temperature that is ____ the boiling point of water a. less than b. greater than c. equal to d. twice

b. greater than

which of the following indicates stunted growth a. weight for height b. height for age c. weight for age d. weight for length

b. height for age

propionate is used in foods to a. enhance flavor b. increase shelf life by inhibiting mold c. thicken d. retain color

b. increase shelf life by inhibiting mold

food staff from a Commodity Distribution program are not allowed to bring foods to a Jewish facility. what is the main reason for this a. because meat cannot be served with dairy products at the same meal b. kosher foods are not allowed to be mixed with non-kosher foods c. Jewish dietary practices are lactoovovegetarian d. Jewish dietary practices are lactovegetarian

b. kosher foods are not allowed to be mixed with non-kosher foods

a patient on a full liquid diet should not receive a. sherbet b. strained cereal c. pureed fruit d. margarine

b. strained cereal

memory loss may be associated with a deficiency of a. riboflavin b. thiamin c. protein d. electrolytes

b. thiamin

a patient on a clear liquid diet should not be served a. strawberry flavored gelatin b. vegetable stock c. apple juice d. fruit flavored popsicle

b. vegetable stock

a 140 pound female on dialysis should have how many grams of protein a. 38-55 b. 58-63 c. 70-89 d. 30-38

c. 70-89

which group is responsible for nutritional labeling a. USDA b. US dept. of commerce c. FDA d. UPI

c. FDA

congregate meals and home delivered meals fall under a. WIC b. EFNEP c. Older Americans Act, title III d. Medicare

c. Older Americans Act, title III

a meat slicer should be cleaned a. at the end of each day b. at the end of each shift c. after each use d. after each meal

c. after each use

which of the following may help decrease cavities a. raisins b. carrots c. cheese d. yogurt

c. cheese

a person with the dumping syndrome cannot handle a. complex carbohydrates b. polysaccharides c. disaccharides d. amino acids

c. disaccharides

an elevated T4 indicates a. galactosemia b. hypothyroidism c. hyperthyroidism d. ketonuria

c. hyperthyroidism

what would you first advise a constipated patient to do a. take a mild laxative b. eat less fat c. increase fluids and fiber intake d. eat more protein

c. increase fluids and fiber intake

which can be included in a meal for gluten induced enteropathy a. spaghetti and tomato sauce b. cheese with crackers c. potatoes with butter d. oatmeal with sugar

c. potatoes with buttter

a tactic that increases the sale of services through publicity is known as a. pricing b. targeting c. promotion d. segmentation

c. promotion

you are asked to develop quality control guidelines for receiving purchased foods. the most important guidelines would be a. proper counts of the foods received b. proper weights of the foods received c. proper temperatures of the foods received d. accurate prices on the invoices

c. proper temperatures of the foods received

which of the following diseases may lead to anemia a. myocardial infarction b. galactosemia c. renal failure d. diabetes mellitus

c. renal failure

what is the most important reason for a microwave oven in food service a. saves energy b. saves money c. saves time d. saves cleanup

c. saves time

which of the following is not associated with anorexia nervosa a. food aversions b. distorted body image c. schizophrenia d. constant concern about weight status

c. schizophrenia

the hospital foodservice department has developed several high quality, sugar free desserts that are identified as unique to the department. this product represents what type of branding a. manufacturer b. retail item c. signature d. specific

c. signature

what can an employer do regarding an employee who wishes to join the union a. prevent him from going to the union b. talk to him against the union c. tell him about union dues d. tell him he may lose his job if he goes

c. tell him about union dues

hormones that control blood pressure and blood components are produced in a. the heart b. the liver c. the kidneys d. the lungs

c. the kidneys

equipment purchases depend on a. the budget b. what the cook wants c. the menu d. the type of facility

c. the menu

which process results in creative, prioritized decision making a. a focus group b. the Delphi technique c. the nominal group technique d. operational analysis

c. the nominal group technique (more structured and controlled, has authoritative leader, controlled interactions, closely focused goals, rigid enforced procedures, silent generation of ideas by participants, round robin reporting, group RANKS items in priority order, vote for final decision)

the dishwasher temperatures are as follows: pre-wash 140F, wash 150F, rinse 200F. you should a. turn up the rinse temperature b. turn down the pre-wash temperature c. turn down the rinse temperature d. do nothing, they are fine

c. turn down the rinse temperature (mechanical dishwasher: pre-wash 110-140F, wash 140-160F, rinse 170-180F)

the efficiency of a foodservice employee is increased if the work center is designed to allow the employee to a. sit and reach b. stand and reach c. use two hands d. use zigzag motions

c. use two hands

which is the best way to maximize meat storage a. cut the meat into small pieces b. store it in oxygen in packages c. vacuum packaged in oxygen impermeable film d. pack it in oxygen permeable film

c. vacuum packaged in oxygen impermeable film

to reduce the budget, first look at a. menu costs b. fixed costs c. variable costs d. semi-variable costs

c. variable costs

which is the least useful evaluation tool in a community assessment program a. height for age b. weight for height c. weight for age d. length for weight

c. weight for age

The flour with the most starch is: a. cake b. whole wheat c. instantized d. durum

cake

Glycoysis is the: a. catabolism of carbohydrate b. creation of glucose from glycerol and amino acids c. breakdown of glycogen releasing glucose d. creation of glycogen

catabolism of carbohydrate

Th mean is a measure of: a. a normal curve b. central tendency c. a frequency curve d. a straight line

central tendency

Which minerals are involved in carbohydrate metabolism? a. cobalt, zinc b. chromium, zinc c. copper, chromium d. iron, zinc

chromium, zinc

Fats enter the blood as ______ and leave the liver as _______. a. phospholipids, chylomicrons b. chylomicrons, phospholipids c. chylomicrons, lipoproteins d. liproproteins, chylomicrons

chylomicrons, lipoproteins

Winterized oil is: a. cloudy when refrigerated b. clear when refrigerated c. cloudy when stored for long periods of time d. clear only if kept at room temperature

clear when refrigerated

When instituting an employee training program, begin by: a. conducting a needs assessment of all to be trained b. review programs offered by other facilities c. determine who will conduct the sessions d. decide ont eh appropriate sized group for training

conducting a needs assessment of all to be trained

Which method of bread-making is not as dependent on the length of time the dough sits to rise? a. straight dough method b. sponge dough method c continuous method c. prime bake method

continuous method

Pantothenic acid, thiamin, riboflavin and niacin are all involved in: a. converting pyruvic into acetyl CoA b. converting tyrosine into phenylalanine c. converting glycogen into glucose d. converting lactic acid into oxaloacetate

converting pyruvic into acetyl CoA

What is the best way to cook a roast? a. sear the meat first, roast at 400 'F b. cook slowly in the oven at 325 'F c. cook it at 400 'F for 10 minutes, then at 350 'F d. brown the meat, then cook it at 350 'F

cook slowly in the oven at 325 'F

The greatest effect on the yield of meat cooked to a uniform internal temperature is produced by the: a. type of oven and source of the heat b. weight of meat and the oven temp c. degree of tempering and the grade of meat d. cooking time and the oven temperature

cooking TIME and the oven TEMPERATURE

The Schillin test detects defects in the absorption of: a. fatty acids b. cyanocobalamin c. pyridoxine d. iron

cyanocobalamin

297. Patients with acute renal failure have higher needs of which nutrient compared to a similar patient with optimal renal function? [NTR:591] a) Protein b) Potassium c) Fluid d) Energy

d) Energy

319. Which of the following is not a sugar alcohol? [NTR:613] a) Sorbitol b) Xylitol c) Mannitol d) Glycogen

d) Glycogen

249. What is the best way to determine the type of nutrition program that will attract baby boomers who use a city recreation center? [NTR:543] a) Give a nutrition knowledge pre-test and post-test to baby boomers who use the facility. b) Determine what nutrition programs are successful with baby boomers in cities of similar size. c) Offer several screening programs and see which one draws the biggest crowd of baby boomers. d) Perform a needs assessment of baby boomers who use the facility.

d) Perform a needs assessment of baby boomers who use the facility.

63. Which of the following would be considered a low allergen food? [NTR:458] a) Milk b) Soy c) Wheat d) Rice

d) Rice

105. Stacey has been asked to complete a formative evaluation of her community nutrition program designed to encourage more first-time mothers to breastfeed. To do this, she should: [NTR:499] a) determine the percentage of first-time mothers who breastfeed as a result of seeing a motivational DVD during prenatal checkups. b) calculate the increase in the number of first-time mothers who breastfeed after viewing a motivational DVD during prenatal checkups. c) choose a DVD that includes young mothers from varied ethnic backgrounds. d) pilot-test the motivational DVD she plans to use.

d) pilot-test the motivational DVD she plans to use.

333. Which of the following is a secondary characteristic of culture? [NTR:627] a) Occupation b) Age c) Gender d) socioeconomic status

d) socioeconomic status

you find out that next years budget will change by the following 10% increase in labor cost, 5% increase in food cost, 2% increase in overhead what will the total budget be if the following info is from the current year labor costs $100000; food cost $100000; overhead cost $40000 a. $156800 b. $234800 c. $267000 d. $255800

d. $255800

a patient with severe acute pancreatitis experiences nausea and vomiting and requires 2500 calories/day. which of the following would you recommend a. 3000cc of a 1cal/cc tube feeding through the jejunum b. 1500cc of a 2cal/cc nasogastric tube feeding c. 1000cc of D50W, 1000cc of 7% amino acids, 650cc of 20% lipids d. 1000cc of D50W, 1000cc of 7% amino acids, 500cc of 10% lipids

d. 1000cc of D50W, 1000cc of 7% amino acids, 500cc of 10% lipids

what is the labor turnover rate if 6 employees in a department of 55 were terminated and replaced a. 12% b. 10% c. 9% d. 11%

d. 11% (6/55=10.91%)

a #12 scoop, was used to serve 600 servings of mashed potatoes instead of a #16 scoop. how many servings were they short a. 90 b. 25 c. 210 d. 156

d. 156

a food service operates with a 35% food cost. if a menu item has a raw food cost of $1.25, and it takes an employee 45 minutes at $6.00 an hour to prepare, what is the traditional selling price a. 2.75 b. 3.25 c. 2.50 d. 3.58

d. 3.58 (100/35 = 2.857 (mark-up); 2.857 x 1.25 (raw food cost) = 3.57 (selling price) or 1.25/.35 = 3.5715

how much water does an athlete need if they have lost 2 pounds during a marathon a. 8 ounces b. 16 ounces c. 24 ounces d. 32 ounces

d. 32 ounces

in dry storage, the humidity should be a. 20-30% b. 30-40% c. 40-50% d. 50-60%

d. 50-60%

using the above formula, determine the calories coming from lipids per day a. 519 b. 561 c. 577 d. 624

d. 624 (20% lipid has 2kcal/ml, 200mlx1.560=312x2)

how many servings are there in 10 gallons of ice cream using a #16 scoop a. 320 b. 480 c. 560 d. 640

d. 640 (32/16 = 2oz per scoop, 10x128=1280 oz/2oz = 640)

how much hamburger will it take to make 300 three ounce patties with a 20% shrinkage a. 52lbs b. 68lbs c. 90lbs d. 70lbs

d. 70lbs (3oz x 300 servings = 900oz/16 = 56.25lbs/.80(yield) = 70.31lb)

the interstate shipping of shellfish is monitored by the a. DHHS b. USDA c. department of commerce d. FDA

d. FDA

what are the similarities between WIC and Food Stamps a. regulated by USDA b. have at risk criteria c. have income criteria d. a health exam is required e. a, b, c, d f. a, b, c g. b, c, d h. a, c

d. a and c (regulated by USDA and have income criteria)

an infant with milk hypersensitivity could safely consume a. soy milk fortified with calcium b. goat's milk fortified with folic acid c. a modular formula with crystalline amino acids d. a casein hydrolysate formula

d. a casein hydrolysate formula

osteoporosis is most likely to occur in a. the physically active adult b. a non-smoker c. a Black female d. a White female

d. a white female

the stasis of pancreatic juice and bile could result in a. autodigestion b. fat malabsorption c. obstructive jaundice d. a and c e. b and c f. a, b, c

d. a, b, c (autodigestion, fat malabsorption, obstructive jaundice)

an example of self-actualization is a. breathing unassisted b. companionship c. safety d. asking for additional training

d. asking for additional training (self-actualization - realizing your potential growth using creative talents; top of Maslow's hierarchy; desire for self-fulfillment)

which food service system is least likely to have microbial contamination a. conventional b. cook-chill c. cook-freeze d. assembly serve

d. assembly serve

the kitchen layout below is characteristic of which foodservice system refrigerator/freezer/steamers/ovens/microwave/meal assembly/delivery carts a. commissary b. conventional c. ready prepared d. assembly-serve

d. assembly-serve (finish by thawing and heating so only need fridge/freezer for storage and equipment for reheating and serving)

a patient on parenteral support develops sepsis. what is the likely cause a. lack of immune-enhancing nutrients b. poor care of the entry site c. related to the condition that created the need for parenteral support d. bacterial translocation

d. bacterial translocation

which correlates best with energy intake a. lean body mass b. triceps skinfold c. body mass index d. body weight

d. body weight

what diet is typically recommended for a patient who has undergone a gastrectomy a. low fiber foods and two glasses of milk each day b. high fiber foods and restriction of fluids with meals c. carbohydrate, protein and fat at each meal, and no dairy products d. carbohydrate, protein and fat at each meal, and restrictions of fluids with meals

d. carbohydrate, protein and fat at each meal and restrictions of fluids with meals

the dietitian reviewed a monitoring sheet for a diary refrigerator at 9am on Wednesday. what should the initial action be Monday 530am 36F 600pm 40F Tuesday 530am 38F 600pm 44F Wednesday 530am 48F a. discard the products immediately b. review the temperatures with the maintenance department c. monitor the temperature every 4 hours d. check the temperature of the products

d. check the temperature of the products

when instructing a patient on diet therapy needed during intake of a monoamine oxidase inhibitor, which would not be included a. milk, cottage cheese, yogurt b. yogurt, ricotta cheese c. swiss cheese, yogurt d. cheddar cheese, swiss cheese

d. cheddar cheese, swiss cheese

which is the most appropriate bedtime snack for a diabetic on NPH insulin a. apple juice and crackers b. graham crackers and diet soda c. fresh fruit and crackers d. cheese and crackers

d. cheese and crackers

a viscosimeter measures a. temperature b. pH c. stability d. consistency

d. consistency

if a food service reconstitutes frozen food that is made on site, what type of production system is it a. convenience b. conventional c. satellite d. cook-freeze

d. cook-freeze

OSHAs responsibilities do not include a. establishing minimum safety standards b. inspecting facilities c. examining records of occupational injuries d. developing a better safety program for a facility

d. developing a better safety program for a facility

when assessing nutritional status, which would be the most beneficial a. 24 hour recall, history of weight changes b. medical history, diet order, socioeconomic status c. clinical observation, weight status, cultural habits d. diet intake info, clinical data, laboratory data

d. diet intake info, clinical data, laboratory data

which of the following is included in a subjective global assessment a. standard weight for height b. mid-arm muscle circumference c. prealbumin d. dietary intake

d. dietary intake

what is the chemical composition of a class ABC fire extinguisher a. water under pressure b. carbon dioxide under pressure c. carbon monoxide under pressure d. dry inert chemicals under pressure

d. dry inert chemicals under pressure (ABC extinguisher is a multi-purpose extinguisher using dry chemicals)

why should a manager not allow employees to wear long jewelry in the kitchen a. to prevent showing off to other workers b. sanitation considerations c. possible theft d. employee safety and sanitation

d. employee safety and sanitation

Glucophage (Metformin) functions by a. stimulates insulin secretion b. inhibits digestive enzyme action c. decreases peripheral resistance d. enhances insulin action

d. enhances insulin action

what can you do if your company denies you the right to organize and form a labor union a. nothing b. quickly form a union c. change jobs d. file a claim with the NLRB

d. file a claim with the NLRB

HACCP standards would be similar for which of the following foods a. canned corn and frozen corn b. fresh chicken and frozen chicken c. fresh beef and frozen chicken d. frozen beef and frozen corn

d. frozen beef and frozen corn (Hazard Analysis Critical Control Point: identifies food hazards and critical control points in food service where loss of control would result in an unacceptable safety risk)

which snack would you not give to an infant a. yogurt b. cheese sticks c. graham crackers d. grapes

d. grapes

why would you add bicarbonate to a solution for a person with hyperkalemia a. it forces magnesium out of the cell to attract potassium b. it forces chloride out of the cell to attract potassium c. it exchanges phosphorus for potassium d. it exchanges hydrogen for potassium

d. it exchanges hydrogen for potassium

a 9 month old infant being fed iron-fortified, milk based formula is obese. you should recommend a. limiting the formula feeding to 16 ounces b. changing the formula to 2% milk c. changing the formula to whole milk d. monitoring weight and intake

d. monitoring weight and intake

a non-directed interview provides a. the same information on all applicants b. a structured set of questions c. the least amount of information d. more information than a directed interview

d. more information than a directed interview

where should a hand washing sink be located a. near the door entering the kitchen b. near the dishroom c. near the cafeteria counter d. near the food preparation area

d. near the food preparation area

a narrow span of management should be employed with a. routine, repetitive, homogeneous work b. health care professionals who don't require close supervision c. well motivated, highly trained group d. newly hired employees

d. newly hired employees

meatloaf has been left out for 5 hours. should you serve it a. yes b. yes because bacteria was destroyed during the heating process c. no because of the risk of botulism d. no because of the risk of staphylococcus

d. no because of the risk of staphylococcus (grows best at 70-97F, not destroyed by cooking, found in reheated foods, meat products, milk products, stuffing, prepared salads; found in humans from nose, hands, intestines, cuts, sores)

an infant wants a bottle at bedtime. you should counsel the parents to a. give the bottle only at bedtime b. give the bottle only if the baby insists c. only give milk in the bottle d. only give water in the bottle

d. only give water in the bottle

which hormone moves milk through the mammary ducts a. prolactin b. progesterone c. estrogen d. oxytocin

d. oxytocin

a post-surgical patient with prolonged ileus should be treated with a. a high fiber diet b. enteral nutrition with added fiber c. enteral nutrition d. parenteral nutrition

d. parenteral nutrition (ileus is lack of propulsive ability of GIT, hypomotility of GIT, lack of digestive propulsion)

which of the following best reflects protein status a. serum albumin b. serum transferrin c. retinol-binding protein d. prealbumin

d. prealbumin

the temperature of a beef casserole drops below 140 within an hour. what should you do a. throw it out b. return it to the hot-holding cabinet to reheat it to a temperature of 165 c. return it to the oven to reheat it to a minimum of 140 d. return it to the oven to reheat it to a minimum of 165

d. return it to the oven to reheat it to a minimum of 165

which of the following components is projected first during the budgeting process a. cash b. labor c. operating expenses d. revenue

d. revenue

which job would you not give to a consultant dietitian a. planning future goals b. writing policies and procedures c. providing staff in service d. supervision of dietary employees

d. supervision of dietary employees

a patient reports decreased intake, nausea, vomiting and decreased functional capacity. to complete a subjective global assessment evaluate a. his lab values over the past 6 months b. current body weight compared to ideal body weight c. usual dietary intake using a food frequency questionnaire d. the presence of edema and wasting or dehydration

d. the presence of edema and wasting or dehydration (SGA looks at history, intake, GI symptoms, functional capacity, physical appearance, edema and weight change)

why are open ended questions better during an interview a. they provide the same info on all applicants b. there is less personal bias c. they are less time-consuming d. they provide more info

d. they provide more info

which anthropometric measurements are useful in assessing adults a. triceps skinfold, head circumference, arm muscle circumference b. head circumference, arm muscle circumference, height, weight c. serum albumin, triceps skinfold d. triceps skinfold, arm muscle circumference, height weight

d. triceps skinfold, arm muscle circumference, height, weight

In the alanine cycle, alanine is released from muscle, taken up by the liver, and: a. excreted in the urine b. deaminated to release glucose c. converted into adipose d. oxidized to nitric acid

deaminated to release glucose

What is the process by which the water content of a vegetable is replaced by a concentrated salt solution? a. gelling b. osmosis c. diffusion d. retrogradation

diffusion

Frozen chicken has been thawed in warm water. You should: a. prepare it immediately b. discard it c. refreeze it d. refrigerate it

discard it

A research study was done on plate waste. "Plate waste for breakfast is more than for dinner. This was due to the familiarity and popularity of the food." What section of the research report would that statement be found? a. methodology b. results c. implications d. discussion

discussion

When writing a research paper, a statement including "this is probably due to..." should be placed in which section? a. discussion b. results c. implications d. methodology

discussion

Where would this be placed in a research article: "most of the changes seen were probably due to..." a. introduction b. discussion c. results d. summary

discussion

What gives the structure to quick breads? a. flour and milk b. flour and water c. egg and flour d. egg

egg and flour

Which is the best source of emulsification? a. eggs b. milk c. salt d. flour

eggs

NADPH is: a. essential in the synthesis of fatty acids b. essential in the synthesis of glycogen c. involved in the catabolism of protein d. essential in transamination

essential in the synthesis of fatty acids

After you review the patient's medical records, the first step in counseling is: a. assess the patient's nutritional status b. collect all relevant data c. establish a climate of trust d. establish an overall evaluation plan for the patient

establish a climate of trust

meats cured with whey won't be tolerated by someone with a. lactose intolerance b. diabetes c. post-gastrectomy of 3 months d. duodenectomy e. a, b, c f. b, c, d g. a h. a, c, d

g. a (lactose intolerance)

Which enzymes are involved in protein digestion? a. ptyalin, lipase, sucrase b. amylase, lipase, trypsin c. pepsin, trypsin, chymotrypsin, carboxypeptidase d. trypsin, lipase, amylase

pepsin, trypsin, chymotrypsin, carboxypeptidase

Fats high in polyunsaturated fatty acids include: a. olive and canola b. sunflower and corn c. coconut and palm d. soy and peanut

sunflower and corn

When teaching a non-English speaking patient about serving sizes: a. show pictures of proper serving sizes b. use food models to allow choice of recommended serving sizes c. show slides of various foods and their serving sizes d. show a videotape of serving sizes

use food models to allow choice of recommended serving sizes

What color do unions turn when cooked in an aluminum pan? a. purple b. blue c. red d. yellow

yellow

228. Older adults are more vulnerable to toxicity of vitamin [NTR:522] a) A b) C c) D d) K

a) A

62. Which of the following foods should a pregnant woman avoid? [NTR:457] a) King mackerel b) Tuna c) Cottage cheese d) Mozzarella cheese

a) King mackerel

freezing will a. inhibit bacterial growth b. destroy toxins c. destroy bacteria d. destroy enzymes

a. inhibit bacterial growth

how many grams does a low birth weight infant weigh a. less than 2500 b. between 500 and 1500 c. between 3000 and 4000 d. between 100 and 500

a. less than 2500

what is a formal study to verify that documentation and processes have been followed a. the audit b. a quality assurance program c. a verification study d. statistical analysis

a. the audit

goals for a community nutrition program should be based on a. the dietary guidelines for Americans b. the nutrition screening initiative c. the healthy people report d. the NHANES III report

a. the dietary guidelines for Americans

the nutritional label on food utilizes the DRIs which replace the old a. RDAs b. USRDAs c. USRs d. OSHAs

b. USRDAs

which is a common complication of AIDS a. constipation b. diarrhea c. hypoglycemia d. hyperkalemia

b. diarrhea

iron status is best monitored by a. dietary and clinical methods b. dietary and lab studies c. dietary studies d. anthropometric studies

b. dietary and lab studies

anasarca is extreme generalized edema and swelling of the skin associated with a. uncontrolled diabetes b. heart, liver or renal failure c. short bowel syndrome d. pancreatitis

b. heart, liver or renal failure

the catecholamines released during stress cause a. hypertension b. hyperglycemia c. diuresis d. elevated BUN

b. hyperglycemia

quickly stopping a 25% dextrose solution leads to a. hyperglycemia b. hypoglycemia c. hypophosphatemia d. hyperkalemia

b. hypoglycemia

esophageal varices may be caused by a. portal hypotension b. portal hypertension c. a high fiber diet d. a high fiber, high residue diet

b. portal hypertension

which of the following is least valuable in a specification a. grade and brand b. quality tolerance limits c. count per container or number per pound d. the unit on which price will be based

b. quality tolerance limits

milk is packaged in opaque containers to avoid UV destruction of a. thiamin b. riboflavin c. niacin d. ascorbic acid

b. riboflavin

which best applies to nutritional screening a. JCAHO standards must be followed b. should include nutritional parameters c. done after nutritional problems are identified d. only done on at risk patients

b. should include nutritional parameters

NHANES III increased the number of participants in which age group a. infants and children b. the elderly c. adolescents d. pregnant women and infants

b. the elderly

285. How many pounds should an overweight woman gain during her pregnancy? [NTR:579] a) 0 b) 5-10 c) 15-25 d) 28-40

c) 15-25

109. Which of the following is a disadvantage of a cohort study? [NTR:503] a) It is difficult to select a suitable comparison group. b) It is difficult if not impossible to validate data. c) It is rarely possible to study the mechanism of a disease. d) Incidence rates among participants cannot be calculated.

c) It is rarely possible to study the mechanism of a disease.

85. Which food would not be an example of a functional food? [NTR:480] a) Garlic b) Yogurt c) Tomato soup d) Gluten-free pasta

c) Tomato soup

224. Children on strict vegan diets may be deficient in: [NTR:518] a) vitamin B6. b) Magnesium. c) vitamin B12. d) Iron.

c) vitamin B12.

the above patient should receive how many grams of protein per Kg a. 0.5-0.6 b. 0.6-0.8 c. 0.8-1.0 d. 1.0-1.4

c. 0.8-1.0

the #12 scoop measures a. 1/4 cup b. 1/2 cup c. 1/3 cup d. 1 cup

c. 1/3 cup

a PN regimen that containes 1L of d50W and 1L of 7% aminosyn provides how many non-protein calories a. 300 b. 1400 c. 1700 d. 2000

c. 1700

the elderly need how many ml of water per KG a. 35 b. 45 c. 30 d. 25

c. 30

in interstate shipping of shellfish is monitored by the a. USDA b. HHS c. FDA d. FTC

c. FDA

the number of work movements between pieces of equipment is indicated on a. a flow diagram b. a Gantt chart c. a cross chart d. Johari's window

c. a cross chart

which of the following most affects osmolarity a. protein b. sugar c. electrolytes d. fat

c. electrolytes

when given as the sole source of energy, isocaloric amounts of which energy substrate would most effectively reduce starvation ketosis a. corn oil b. MCT oil c. glucose d. essential amino acids

c. glucose

an infant has just begun to drink from a cup and uses the pincer grasp. which of the following should you recommend be added to his diet a. finely cut meat b. unflavored yogurt c. graham crackers d. pureed vegetables

c. graham crackers

what is the appropriate diet for heart failure a. low potassium b. low calcium c. low sodium d. low fat

c. low sodium

what nutrients most affect the renal solute load a. protein and carbohydrate b. sodium and carbohydrate c. protein and sodium d. sodium and fat

c. protein and sodium

iatrogenic malnutrition is a. vitamin and mineral malabsorption b. protein malabsorption c. protein, calorie malnutrition d. kwashiorkor

c. protein, calorie malnutrition

a new patient with a BMI of 32 and hypertension comes to you for counseling. what is your first recommendation a. achieve DBW b. maintain weight and exercise c. reduce sodium intake d. reduce sodium, reduce fluid, reduce caloric intake

c. reduce sodium intake

you are conducting a nutrient analysis of a recipe that uses 10 lbs of 80/20 raw ground beef. to best ensure analysis accuracy a. use data for the proportioned cooked ground beef b. use data for the edible portion weight of the raw ground beef c. use data for the edible portion weight of the cooked ground beef d. cooking and draining the ground beef, weighing the fat, then reducing the quantity of beef in the recipe by this amount

c. use data for the edible portion weight of the cooked ground beef

to increase the number of patrons served in the cafeteria during the lunch meal a. increase the number of cafeteria employees b. add a second tray line c. use the scramble system d. add another cashier

c. use the scramble system

which of the following would provide the most useful data when conducting nutrition screening on preschool children at a health fair a. triceps skinfold thickness b. weight for age c. weight for stature d. stature for age

c. weight for stature

An 8 month old baby contracts botulism. What is the likely cause? a. cereal and evaporated milk b. cereal and honey c. cereal and table food d. cereal and fruit juice

cereal and honey

Cholecystokinin causes: a. contraction of the gallbladder b. peristalsis c. secretion of pancreatic juice d. contraction of the liver

contraction of the gallbladder

77. Which step in the Nutrition Care Process has as its purpose to quantify progress made by the patient/client in meeting nutrition care goals? [NTR:472] a) Nutrition Assessment b) Nutrition Diagnosis c) Nutrition Intervention d) Nutrition Monitoring and Evaluation

d) Nutrition Monitoring and Evaluation

30. Which of the following foods does not contain carbohydrates? [425] a) tortilla b) broccoli c) lentils d) eggs

d) eggs

287. When working with disabled adults, it is suggested that the dietetics professional _____. [NTR:581] a) use the children's My Pyramid because it contains pictures b) speak initially to the guardian rather than the client c) limit options presented to prevent confusing the client d) teach rather than test

d) teach rather than test

303. The breakdown of fatty acids to yield acetyl-CoA is called [NTR:597] a) Glycolysis b) lipolysis. c) ketogenesis. d) ß-oxidation.

d) ß-oxidation.

which infant is at greatest risk a. 20 pounds hemoglobin 10 b. 22 pounds hemoglobin 10.5 c. 19 pounds hemoglobin 11 d. 30 pounds hemoglobin 9

d. 30 pounds hemoglobin 9

the kitchen floor will be changed. what do you recommend a. asphalt tiles b. terrazzo c. concrete blocks d. clay tiles

d. clay tiles

the Ouchi Theory Z management theory states that a. decision making is a centralized function b. committed employees can exercise self control c. employee potential is only partially utilized d. employee involvement is the key to productivity

d. employee involvement is the key to productivity

for which of the following diseases is diet therapy the primary treatment a. peptic ulcer b. diabetes c. Wilson's disease d. galactosemia

d. galactosemia

a Hispanic woman needs more calcium in her diet. what do you recommend a. milk b. plantains c. avocados d. pinto beans

d. pinto beans

the dietitian is asked to determine the prevalence of overweight children within a local school district. the last step of the community needs assessment would be to a. share the findings with the school's health team b. collect data on soda consumption c. measure the BMIs of all of the school children d. set a plan of action

d. set a plan of action

when does a breakeven point increase a. when sales decrease b. when profits decrease c. when profit increases d. when costs increase

d. when costs increase

To define performance objectives for a class for patients, you must: a. review the literature for the latest research b. determine what the learner should know or be able to do c. review diet prescriptions and nutrition care plans d. refer to the hospital's standards of care

determine what the learner should know or be able to do

Children's diets are found to be low in iron due to emphasis on: a. vegetables b. milk c. fruits d. cereals

milk

A patient who is iron-deficient should consume _____ with a good iron source. a. apple juice b. orange juice c. milk d. cheese

orange juice

Customer and employee satisfaction are both: [MGT:376] a. functions of management. b. linking processes. c. environmental factors. d. outputs of the foodservice system.

outputs of the foodservice system.

A serum sodium level of 115 mEq/L may be due to: a. overhydration b. dehydration c. a very high sodium intake d.a sodium intake of 5.5 grams per day

overhydration

Pie crust is moist the day after cooking. Why? a. overcooking of the crust b. retrogradation of filling c. too much egg in the recipe d. inadequate flour

retrogradation of filling

If patients in a counseling session become restless, a. shorten the sessions b. use a series of guest speakers c. realize that this is to be expected d. cancel the session

shorten the sessions

Protein digestion begins in the: a. mouth b. stomach c. small intestine d. ileum

stomach

How can you encourage preschool children to eat more fruits and vegetables? a. Have them participate in preparing fruits and vegetables b. Have the draw pictures of fruits and vegetables c. Give examples of good choices using a slide show d. Give examples of good choices using pictures

Have them participate in preparing fruits and vegetables

An example of formative evaluation is: a. IQ test b. RD exam c. Pretest quiz d. Comparative evaluation

Pretest quiz

What elements comprise the marketing mix? [MGT:384] a. Product, price, promotion, and place b. Price, product, popularity, and process c. Product, promotion, position, and popularity d. Product, place, position, and process

Product, price, promotion, and place

In the systems model, transformation is dependent on the requirements of the functional subsystems. Which resource would be most important for the distribution and service subsystem of a school cafeteria? [MGT:374] a. Steam table b. Storeroom c. School nutrition staff d. Vendor delivery schedule

Steam table

Which exhibits a definite positive correlation? a. .05 - .07 b. 1.2 - 1.3 c. 0.8 - 1.0 d. 0.2 - 0.9

0.8 - 1.0

Which would you give to a person who is mildly hypokalemic? a. 1 cup blueberries b. 1 cup cranberries c. 1/2 cup applesauce d. 1 piece cantaloupe

1 piece cantaloupe

The toxic level of vitamin A has been identified as: a. 5000 IU b. 10000 IU c. 15000 IU d. 20000 IU

10000 IU

Which oil is highest in monounsaturated fatty acids? a. olive b. canola c. peanut d. sunflower

olive

Fats high in monounsaturated fatty acids include: a. olive and canola b. sunflower and corn c. coconut and palm d. soy and peanut

olive and canola

Green beans cooked in tomato sauce turn _____ due to _____. a. bight green; chlorophyllin b. olive green; phenophytin c. brown; carotenoids d. black; anthoxanthins

olive green; phenophytin

Considering the following smoke points of various fats, which would be the best to use for frying? a. 300 'F b. 325 'F c. 375 'F d. 400 'F

400 'F

How many mEq of sodium are in 1 gram of sodium? a. 25 b. 43 c. 65 d. 110

43

Which of the following is a current asset? [MGT:346] a. Accounts receivable b. China c. Small equipment d. Retained earnings

Accounts receivable

French fries made from stored potatoes would have changes in: I. taste II. appearance III. texture a. I, II, III b. II, III c. I d. I, III

I, II, III

In the Nutrition Care Process, the Nutrition Diagnoses should be _____during Nutrition Intervention. [MGT:395] a. Prioritized b. Promulgated c. Investigated d. linked

Prioritized

The results of a glucose tolerance test would be best plotted on: a. a linear regression graph b. a line graph c. a pie chart d. a mean and standard deviation chart

a line graph

When training new employees, what should always be followed? a. a planned orientation and training program b. government orientation regulation c. OSHA orientation regulations d. county orientation regulations

a planned orientation and training program

290. What enzyme is responsible for starch digestion? [NTR:584] a) Amylase b) Lipase c) Lactase d) Insulin

a) Amylase

which of the following provides 60g of CHO a. 1 English muffin, 1 c skim milk, 1/2 cup apple juice b. 1 fat-free granola bar, 1/2 cup raisins, 1 c fruited yogurt with aspartame c. 2 small waffles with 2T lite syrup and 1T peanut butter d. 1 four oz. bagel with 2T lite cream cheese and 1T jelly

a. 1 English muffin, 1 c skim milk, 1/2 cup apple juice

it takes _____ FTEs to fill one 8 hour, 7 day per week position a. 1.4 b. 1.6 c. 1.8 d. 2

a. 1.4 (relief workers can cover 2 days off plus a five day shift, since need # of FTEs calculate 1 FTE for the 5 day work week and then # of FTEs for the remaining 2 days (2/5=.40) so 1 FTE +.40 FTE = 1.4 FTEs)

you are planning the budget for next year. labor costs will increase by 10%. food costs will increase by 5%. operating costs will increase by 2%. this year, sales totaled $1000000. labor costs were 40% of income, food cost was 40% of income and operating costs were 20% of income. what is the projected budget a. 1064000 b. 1640000 c. 1840000 d. 1170000

a. 1064000

how much free water is in 1.5 liters of an enteral feeding that has 1.5 calories per cc a. 1155 cc b. 1057 cc c. 926 cc d. 901 cc

a. 1155 cc (1.5kcal/cc formulas are 76-78% water; 1500ml formula x 1.5 kcals = 2250kcals; 1500 x 76-78% = 1140-1170ml water)

according to the diabetic exchanges, 2T peanut butter is a. 2 high fat meats b. 1 high fat meat c. 2 fats d. high fat meat plus 2 fats

a. 2 high fat meats

a sandwich with two pieces of white bread and two ounces of turkey provides how many calories a. 250 b. 145 c. 198 d. 290

a. 250

how many 4 ounce servings can you get from 1 gallon of ice cream a. 32 b. 26 c. 22 d. 44

a. 32 (4 quarts in a gallon = 32x4=128ounces/4=32 servings)

what is the food cost percentage if the cost of food is $325 and the income is $865 a. 38% b. 36% c. 34% d. 32%

a. 38% (325/865=37.58%)

according to the diabetic exchange lists, which of the following is equivalent to one medium fat meat a. 4 ounces tofu b. 2 ounces pork c. 2 eggs d. 2T peanut butter

a. 4 ounces tofu

if a diabetic patient reduces his calorie intake from 2300 to 1800 calories per day, how much will he lose in one month a. 4 pounds b. 6 pounds c. 2 pounds d. 8 pounds

a. 4 pounds

the cafeteria serves lunch from 11am -230pm with a seat turnover rate of 1.5/hour. they need to seat 250 customers. how many chairs do they need a. 48 b. 56 c. 72 d. 125

a. 48 (1.5 x 3.5 = 5.25; 250/5.25 = 47.61)

which level of HgA1c suggests good compliance with dietary recommendations in a diabetic after 3 months of counseling a. 6.5 mg/dl b. 4.0 mg/dl c. 8.2 mg/dl d. 10.3 mg/dl

a. 6.5 mg/dl

the minimum recommended temperature for a chemical sanitizer is a. 75F b. 140F c. 100F d. 170F

a. 75F

responsibility, recognition by management, and participation in decision making are all considered true motivators by a. Herzberg b. Taylor c. Maslow d. Likert

a. Herzberg (two-factor theory; motivation and maintenance approach; satisfiers are good dissatisfiers are negative - interfere with work and do not motivate; motivators are fair wage, insurance, retirement benefits, supervision, schedule, working conditions, relationships on job)

who has the greatest water, protein and calorie needs per unit of body weight a. a 6 month old infant b. a 50 year old female c. a 16 year old male d. a 25 year old male

a. a 6 month old infant

a food service director uses a 4 month moveable average to determine how many chicken breasts are needed for August given the number served in April, May, June and July. how would they forecast the need for September a. add august and delete April b. add august c. add august and delete july d. look at last year's September production tally

a. add august and delete april

the immobilized state can lead to low levels of a. albumin and calcium b. albumin and sodium c. calcium and potassium d. potassium and zinc

a. albumin and calcium

a 35 year old female with Type 1 diabetes received nutrition counseling and started NPH therapy one month ago. today her fasting blood glucose level is 290mg/dl and cholesterol level is 185mg/dl. she eats one meal per day. which need do you address first a. instruction about the relationship of diet to blood glucose level b. instruction about maintaining current body weight c. advice to include various food groups d. advice about controlling cholesterol levels

a. instruction about the relationship of diet to blood glucose level

southeast Asian populations would likely consume the following foods a. rice, vegetables and small amounts of meat b. fruits, cheese and small amounts of rice and meat c. large amounts of rice and milk, and small amounts of meat and fruit d. large amounts of meat and milk, small amounts of fruits and vegetables

a. rice, vegetables and small amounts of meat

on your meal rounds you notice that elderly patients are not fed properly. what is the best solution a. speak with the director of nursing b. discuss it at a discharge planning meeting c. confer with the nurse's aide responsible d. assign your diet aide to help feed the patient

a. speak with the director of nursing

which of the following has the quickest onset of food borne illness a. staphylococcus aureus b. clostridium botulinum c. clostridium perfringens d. campylobacter jejuni

a. staphylococcus aureus

which is a part of the nutritional therapy for IDDM in a lean person a. time meals b. decrease caloric intake c. it is not crucial to maintain consistency of calories if average intake is low d. decrease the frequency of feedings

a. time meals

the best use of performance appraisals is a. to improve performance b. to motivate the employee c. to weed out the employees who perform poorly d. to reward those who do well

a. to improve perforamnce

muscle mass is determined using the a. triceps skinfold and arm circumference b. skinfold caliper c. BEE d. urinary creatinine measurement

a. triceps skinfold and arm circumference

Glucocorticoids are: a. thyroid hormones b. pituitary hormones c. adrenal hormones d. parathyroid hormones

adrenal hormones

A client says: "I want to change my eating habits." You ask why. The learning domain is: a. affective b. cognitive c. psychomotor d. psychosocial

affective

Which amino acid is especially glucogenic? a. alanine b. theonine c. valine d. methionine

alanine

Which is the best flour when making cakes? a. graham flour b. all-purpose flour c. whole wheat flour c. durum wheat flour

all-purpose flour

What causes egg yolk color to change? a. alteration in the feed provided b. storage length c. shell color d. storage conditions

alteration in the feed provided

Which of the following is a digestive enzyme secreted by the pancreas? a. pepsin b. amylase c. lactase d. maltase

amylase

Which enzymes are secreted by the pancreas? a. amylase, lipase, cholesterol esterase, trypsin, chymotrypsin b. sucrose, maltase, lactase, aminopeptidase c. ptyallin d. aminopeptidase, dipeptidase

amylase, lipase, cholesterol esterase, trypsin, chymotrypsin

Which of the following will stabilize the egg in a recipe? a. a base b. an acid c. sugar d. salt

an acid

open tube feedings should be disposed of after a. 48 hours b. 24 hours c. 6 hours d. 3 hours

b. 24 hours

WIC encourages women with babies to a. formula feed b. breast feed c. feed solid foods early d. feed a mixture of formula and solids foods only

b. breast feed

following a cholecystectomy, bile is produced by the a. pancreas b. liver c. spleen d. portal vein

b. liver

a fecal excretion of ___ grams of fat/24 hours is considered fat malabsorption a. less than 1 b. more than 7 c. 3-5 d. 2-4

b. more than 7 (normal stool fat is 2-5g; >7g is indicative of malabsorption)

a patient with liver disease needs adjustments in the intake of a. protein, fat and sodium b. protein, fluids and sodium c. protein, calories, sodium d. fluids, calories, fat

b. protein, fluids, sodium

patients with decubitus ulcers will likely demonstrate low a. serum sodium b. serum albumin c. serum potassium d. serum glucose

b. serum albumin

quality standards for fruits assure you of a. wholesomeness b. specifications of minimum quality characteristics c. safety from pesticides and residues d. good taste

b. specifications of minimum quality characteristics

a child has been measured several times before the age of two years. results of the measurements were not accurate because the child was measured a. first thing in the morning b. standing up c. after breakfast d. with light clothing

b. standing up

spaghetti sauce prepared on Monday is to be served on Wednesday. what should you recommend to the cook a. cool the sauce and store it in a deep pan b. store the sauce in shallow pans in the refrigerator c. store the sauce in a deep pan in the refrigerator quickly d. cool completely on the counter, then refrigerate

b. store the sauce in shallow pans in the refrigerator

the year 2010 healthy people goals are an outgrowth of a report by a. ADA b. the Surgeon General, under HHS c. WHO d. FAO

b. the Surgeon General, under HHS

what is not shown on an organizational chart a. relationships of positions to one another b. the degree of authority and informal relationships c. advisory responsibilities d. lines of communication

b. the degree of authority and informal relationships

why might a child with PKU have an increased incidence of dental caries a. phenylalanine is not converted to tyrosine b. the diet is high in carbohydrates, low in protein c. pyruvate is not converted to alanine d. the diet is devoid of protein

b. the diet is high in carbohydrates, low in protien

if you worked overtime and did not get paid, which labor law would help your claim a. the equal opportunity act b. the fair labor standards act c. the equal pay act d. the civil rights act

b. the fair labor standards act

to find out about the new regulations developed by the FDA you would check a. the congressional record b. the federal register c. current newspapers d. the congressional index

b. the federal register

why is an affluent couple (the man with diabetes, 67 years old, and the wife with a broken arm, 50 years of age) not eligible for Meals on Wheels a. she is too young b. they are affluent c. this program does not cater to special diets d. they can leave the house

b. they can leave the house

which is the most useful for evaluating recent under-nutrition or over-nutrition a. height for age b. weight for height c. weight for age d. length for weight

b. weight for height

Which of the following are synthesized by intestinal bacteria? a. biotin, pantothenic acid, vitamin K b. vitamin E, vitamin K, biotin c. pyridoxine, vitamin E d. ascorbic acid, pyridoxine, vitamin K

biotin, pantothenic acid, vitamin K

346. How many dietary folate equivalents (DFE) are provided in a supplement containing 150 mcg of folate? [NTR:641] a) 150 DFE b) 225 DFE c) 255 DFE d) 270 DFE

c) 255 DFE *1 DFE = 1 μg food folate = 0.6 μg folic acid from supplements and fortified foods

295. When counseling patients diagnosed with celiac disease, which of the following would you recommend they avoid? [NTR:589] a) Hominy grits b) Sherbet c) Cheerios d) Fresh ground beef

c) Cheerios

280. Infants of mothers who practiced a vegan diet while pregnant, and who are breastfed, would benefit from supplementation of what micronutrient? [NTR:574] a) Niacin b) Calcium c) Vitamin B12 d) Vitamin A

c) Vitamin B12

47. A 2-year old child who drinks large quantities of milk and very little solid food is at risk for [NTR:442] a) diabetes mellitus. b) food allergies. c) iron-deficiency anemia. d) short stature.

c) iron-deficiency anemia.

271. A patient status post gastric bypass surgery has an increased risk for deficiency of which micronutrient? [NTR:565] a) Vitamin C b) Potassium c) vitamin B12. d) Omega 3 fatty acids

c) vitamin B12.

a patient with a GFR of 20ml/min should receive how many grams of protein/kg a. 1.1 b. 0.8 c. 0.6 d. 0.2

c. 0.6

how many actual relief employees are needed to cover 20 fulltime employees a. 12 b. 15 c. 11 d. 8

c. 11 (multiply # of FT employees by .55 to get # of relief workers needed; 20 x .55=11)

which program teaches good nutrition practices and provides foods with certain nutrients a. EFNEP b. NET c. WIC d. MCP

c. WIC

if you were an employee and member of the union, and were involved in initial labor dispute talks, you would be called a. an arbitrator b. a mediator c. a shop steward d. a union agent

c. a shop steward

an obese person with type 2 diabetes should a. time meals b. decrease the frequency of feedings c. achieve DBW d. increase the frequency of feedings

c. achieve DBW

which is not a fiber in the diet a. cellulose b. hemicellulose c. amylose d. pectin

c. amylose

the immune system reacts to foods that are normally harmless. this is a. food intoxication b. an inflammatory response c. an immunoglobulin E (IgE) mediated reaction d. food intolerance

c. an immunoglobulin E (IgE) mediated reaction

which would you recommend to a Mexican American on a low fat diet a. low fat cottage cheese b. skim milk c. beans and rice d. low fat yogurt

c. beans and rice

catecholamines have what physiological effect a. induce diuresis b. cause hypoglycemia c. cause sodium retention d. cause potassium retention

c. cause sodium retention

underwriters laboratory does which of the following a. inspects facilities for safety b. inspects facilities for cleanliness c. checks safety of electrical equipment d. inspects gas equipment

c. checks safety of electrical equipment

how does a new supervisor gain respect a. show employees different ways to do their job b. work with a different employee each day c. communicate regularly with them to keep them informed d. smile a lot

c. communicate regularly with them to keep them informed

you discover an incorrect diet order entered in the patient's chart a. chart the order as incorrect b. file the order because it was written by a physician c. discuss the order with the physician and make suggestions d. change the order in the chart

c. discuss the order with the physician and make suggestions

the data that is most useful in evaluating children is a. sex and age b. iron and vitamin intake c. energy intake and health history d. height and weight

c. energy intake and healthy history

the most effective nutrition assessment method for large groups is the a. food diary b. 24 hour recall c. food frequency list d. diet history

c. food frequency list

before discontinuing a parenteral feeding, provide enteral support to prevent a. development of a fatty liver b. ketoacidosis c. hypoglycemia d. hepatic encephalopathy

c. hypoglycemia

an 11 month old infant is constipated. the most likely nutrition diagnosis would be a. GI distress b. lack of fruits c. inadequate fluid intake d. imbalanced intake

c. inadequate fluid intake

a patient has been hospitalized for 17 days with multiple injuries following an accident. when should the discharge plan begin a. as soon as it is known when the patient will be discharged b. once the patient is stable c. on the first day of admission d. at least two days prior to discharge

c. on the first day of admission

the diet for galactosemia should exclude a. high fat foods b. high cholesterol foods c. organ meats, MSG d. simple carbohydrates

c. organ meats, MSG (missing enzyme that converts galactose-1-PO4 into glucose-1-PO4, treated with a lactose and galactose free diet, NO: organ meats, MSG extenders, milk, lactose, galactose, whey, casein, dry milk solids, curds, calcium or sodium caseinate, dates, bell peppers), OK: soy, hydrolyzed casein, lactate, lactic acid, lactalbumin, pure MSG

which of the following is an example of a secondary prevention program a. changing vending machine options available in a workplace b. a billboard that encourages fruit and vegetable consumption c. osteoporosis education classes for postmenopausal women d. weight-management classes in a cardiac rehabilitation program

c. osteoporosis education classes for postmenopausal women (Primary - aims to prevent disease before it occurs; Secondary - aims to reduce the impact of a disease/injury that has already occurred; Tertiary - aims to soften the impact of an ongoing illness/injury that has lasting effects)

which nutrients are involved in energy metabolism a. pyridoxine and cyanocobalamin b. copper and zinc c. pantothenic acid and biotin d. folic acid and iron

c. pantothenic acid and biotin

what type of leadership style is shown with a circular organizational chart a. democratic b. autocratic c. participative d. bureaucratic

c. participative

which condition benefits from the use of indirect calorimetry a. post-gastrectomy b. patient receiving chemotherapy c. patient with third degree burns d. post-surgical patient

c. patient with third degree burns

one of the most important strategies a dietitian may use to evaluate quality patient care is to a. determine criteria for consultations b. monitor treatment for a nutrition problem c. periodically assess the outcomes of nutrition therapy d. assign individuals responsibility for monitoring activities

c. periodically assess the outcomes of nutrition therapy

which term relates to scientific management a. psychological b. intellectual c. physical d. sociological

c. physical

which of the following extinguishers is best to use on a grease fire a. triangle symbol b. circle symbol c. square symbol d. inverted triangle symbol

c. square symbol (square (B) = flammable liquids, gases, greases; triangle (A) = ordinary combustible materials, wood, paper, cloth; circle (C) = live electrical fires)

the cleanliness of on-line steam is not an issue when purchasing a a. convection steamer b. combination oven c. steam jacketed kettle d. compartment steamer

c. steam jacketed kettle

which of the following would be classified as NC in the NCP a. inadequate vitamin intake b. limited access to food c. swallowing difficulty d. inappropriate intake of food fats

c. swallowing difficulty (NC=nutritional findings/problems that relate to medical/physical condition; physical or mechanical change that interferes/prevents desired nutritional results - swallowing difficulty, altered GI function-change in capacity to metabolize nutrients due to medications, surgery or indicated by lab values; chronic or changed weight status when compared with UBW, underweight, involuntary weight loss, overweight)

if a female is hired and does not receive the same pay as a male in the same position, she can seek help from a. the equal opportunity act b. the fair labor standards act c. the equal pay act d. the fair wage act

c. the equal pay act

which would be the best tool for rating foods in a pediatric hospital a. the triangle test b. paired comparison test c. the facial hedonic scale d. the paired preference test

c. the facial hedonic scale

a hospital administrator would be most concerned with a. the increase in the number of elderly admissions b. the increasing growth of outpatient services c. the increase in patient's length of stay d. the increase in diabetic admissions

c. the increase in patients length of stay

which of the following is a paid employee who represents other union workers a. a mediator b. the department head c. the shop steward d. the bargaining agent

c. the shop steward

a homeless patient who is deficient in all nutrients is to be discharged. what organization may help a. WIC b. EFNEP c. the social workers group d. the shelter

c. the social workers group

Asian, Hispanic and native americans have the following in common a. they consume few green vegetables b. they consume significant amounts of dairy products c. they consume little milk d. they consume few root vegetables

c. they consume little milk

what best measures excess calorie reserves a. weight b. creatinine c. triceps skinfold d. arm circumference

c. triceps skinfold

which of the following indicates protein calorie malnutrition a. a creatinine height index of 93% a. height for weight ratio of 89% c. triceps skinfold and arm muscle circumference of 79% d. serum albumin of 4.5; transferrin of 410

c. triceps skinfold and arm muscle circumference of 79%

what should a Jewish patient include in his diet to increase the intake of zinc a. fresh green vegetables b. pork c. turkey d. wheat bread

c. turkey (red meat and poultry provide majority of zinc in American diet, also in beans, nuts, breakfast cereals, seafood, whole grains, dairy)

a Rabbi is admitted to the hospital. what question will he ask the dietitian a. what time are meals served b. who cooks the food c. what is the mixture of foods in combination dishes d. where will he eat

c. what is the mixture of foods in combination dishes

when can a citizen have impact on a bill a. during debate in the Senate b. when the bill goes to committee c. when the bill is in public hearings d. during debate in the House

c. when the bill is in public hearings

a patient with chronic pancreatitis should consume oral pancreatic enzymes a. with large meals and fatty foods b. only if steatorrhea increases c. with all meals and snacks d. only if consuming a high fat, low protein diet

c. with all meals and snacks

iron preparations should be taken a. before meals b. on an empty stomach c. with meals d. before bedtime

c. with meals

What is the difference between cake flour and bread flour? a. cake flour has less gluten, weaker gluten, less protein b. bread flour has less gluten, weaker gluten less flour c. cake flour has weaker gluten, but has more protein d. bread flour has stronger gluten and less protein

cake flour has less gluten, weaker gluten, less protein

Which mineral is involved in blood clotting? a. calcium b. phosphorus c. iron d. selenium

calcium

Which pigment does not change color in an acid or an alkaline solution? a. anthocyanin b. chlorophyll c. carotene d. flavones

carotene

Which pigment can be converted into a vitamin? a. chlorophyll b. carotenoids c. anthocyanin d. anthoxanthins

carotenoids

Which oil is highest in saturated fat? a. coconut b. palm c. cocoa butter d. butter

coconut

Decreases in labor costs and overtime can be accomplished by: [MGT:389] a. decreasing part-time employees. b. controlling absenteeism. c. evaluating employees. d. all of the above.

controlling absenteeism.

Kidneys help control pH by: a. excreting sodium and calcium b. controlling the ratio of hydrogen ions to bicarbonate excretion c. excreting carbon dioxide and carbonic acid d. excreting NADPH

controlling the ratio of hydrogen ions to bicarbonate excretion

Excess amounts of zinc may lead to a deficiency of: a. copper b. magnesium c. iron d. cobalt

copper

Which mineral is absorbed through the intestine and transported with ceruloplasmin in the blood? a. copper b. iodine c. sodium d. calcium

copper

The following oils can be winterized: a. corn, soy, cottonseed b. olive, corn, soy c. cottonseed, olive d. olive, corn

corn, soy, cottonseed

What must be fortified with folic acid? a. skim milk b. cornmeal c. orange juice d. canned vegetables

cornmeal

To determine the reading level of a test: a. count the number of words more than 1 syllable b. count the number of paragraphs c. count the average number of letters in the words d. count the number of sentences in a paraphraph

count the number of words more than 1 syllable

Gluconeogenesis is the: a. catabolism of carbohydrate b. creation of glucose from glycerol and amino acids c. breakdown of glycogen releasing glucose d. creation of glycogen

creation of glucose from glycerol and amino acids

282. The biologically active hormone form of vitamin D is: [NTR:576] a) 7-dehydrocholesterol b) 25-hydroxyvitamin D c) D3 d) 1, 25-dehydroxyvitamin D

d) 1, 25-dehydroxyvitamin D

21. How many times might a parent need to expose a child to a food before acceptance of that food occurs? [NTR:416] a) 1 to 2 b) 3 to 5 c) 6 to 7 d) 8 to 10

d) 8 to 10

9. According to the CDC growth chart in what percentile is a 1-year-old boy measuring 82 cm? [NTR:404] a) <5% b) 25% c) 75% d) >95%

d) >95%

323. Which of the following symptom(s) often characterizes niacin deficiency? [NTR:617] a) Diarrhea b) Dermatitis c) Dementia d) All of the above

d) All of the above

321. Which of the following foods is likely to be consumed by a lacto-vegetarian? [NTR:615] a) Scrambled eggs b) Tuna fish salad c) Hamburger d) Grilled cheese sandwich

d) Grilled cheese sandwich

75. Which of the following is completed as part of the third step in the Nutrition Care Process? [NTR:470] a) Identifying whether or not intake is inadequate or excessive compared to actual or estimated needs. b) Assessing nutrition problems that relate to medical or physical conditions. c) Interpreting lab data and tests. d) Implementing appropriate nutrition interventions that are tailored to the patient/client's needs.

d) Implementing appropriate nutrition interventions that are tailored to the patient/client's needs.

334. Currently, which of the following nutrients is determined to be the most deficient among children and adolescents in the United States? [NTR:628] a) Calcium b) Fiber c) Vitamin A d) Iron

d) Iron

279. Phenylalanine is a precursor to which amino acid? [NTR:573] a) Arginine c) Glutamine c) Cysteine d) Tyrosine

d) Tyrosine

a patient receives 4 ounces whole milk and 6 graham crackers as a snack. which of the following substitutes would lower the glycemic index a. skim milk in place of whole milk b. fruit juice in place of whole milk c. 12 saltines in place of 6 graham crackers d. 1 T peanut butter in place of 3 graham crackers

d. 1 T peanut butter in place of 3 graham crackers

which of the following would be appropriate for a hemodialysis patient on a 60g protein diet, with 75% from HBV protein a. 1 egg, 2 ounces chicken, 3 ounces beef, 1/2 cup milk b. 1 egg, 2 ounces chicken, 3 slices bread, 3 ounces beef c. 2 eggs, 3 ounces chicken, 3 ounces beef, 1 cup milk d. 1 egg. 3 ounces chicken, 2 cups milk, 5 slices bread

d. 1 egg, 3 ounces chicken, 2 cups milk, 5 slices bread (59g PRO means 44g from HBV; HBV are meats, eggs, milk)

how many FTEs can you hire with a weekly budget of $8640 if the salary is $12.00 per hour a. 10 b. 13 c. 16 d. 18

d. 18 (8640/12=720/40=18)

what is the inventory turnover rate net profit $41137 cost of goods sold $145350 average inventory value $7330 total meals served per day 200 a. 205.68 b. 726.75 c. 5.6 d. 19.83

d. 19.83 (inventory turnover rate = cost of sales (food cost)/average inventory cost)

a standing height should first be obtained when an infant is how many months old a. 18 b. 20 c. 22 d. 24

d. 24

what is the most important advice to give a patient with a peptic ulcer a. drink milk every 1-2 hours b. consume small portions c. eat everything in balance d. abstain from spicy food

d. abstain from spicy food

what function is served by glycerol monostearate a. add to the taste of the food b. acts as an emulsifier c. acts as an antioxidant d. acts as a humectant

d. acts as a humectant

what can be done to lower the respiratory quotient of a tube fed pulmonary patient a. add MCT oil to decrease energy required for growth b. change the type of fat to improve oxygen exchange c. add carbohydrate to decrease the respiratory quotient d. add a fat emulsion to replace some of the carbohydrate

d. add a fat emulsion to replace some of the carbohydrate

which statement applies to nutritional screening a. only a physician makes the referral b. only a dietitian makes the referral c. it is exclusively done by nursing d. all health team members can participate according to policy

d. all health team members can participate according to policy

which of the following statements about cow's milk are true a. it is high in protein b. it creates a high solute load c. it is low in iron and vitamin C d. a only e. a and c f. all of the above

d. all of the above, it is high in protein, creates a high solute load and is low in iron and vitamin C

which will cause the most delay in gastric emptying a. lean baked meat b. broiled cod c. shrimp d. corned beef

d. corned beef (higher in fat so delays gastric emptying)

the most important prenatal supplement is a. calcium b. vitamin D c. iron d. folic acid

d. folic acid

Medicaid benefits apply to a. healthy adults b. a 60 year old female c. those at nutritional risk d. low income individuals

d. low income individuals (133% of federal poverty level including children, pregnant women, parents, seniors and individuals with disabilities)

the organized gathering of data and information for the purpose of analyzing a service for clients is a. an experimental design b. a relationship analysis c. product development d. market research

d. market research

the NHANES study looks at which parameters a. nutritional b. nutritional, clinical c. nutritional, biochemical d. nutritional, clinical, biochemical

d. nutritional, clinical, biochemical

the Prader Willi syndrome is associated with a. hyperactivity b. deficiency of glucose 6 phosphatase c. macrocytic anemia d. obesity

d. obesity

a nutritional care plan for overweight children should include a. restrict intake to maintain current weight b. maintain body weight c. reduce weight with caloric reductions of 500 per day d. reduce rate of body weight gain while allowing growth

d. reduce rate of body weight gain while allowing growth

unions usually advance employees based on a. age b. skill c. merit d. seniority

d. seniority

in order to influence a legislator one day before a vote, the dietitian should a. visit the legislative aide b. write a letter to the legislator c. send a postcard to the legislator d. telephone the legislator

d. telephone the legislator

the main reason for keeping an inventory is a. to plan next year's budget b. to prevent thievery c. to see how well you are following the budget d. to determine when and how much to reorder

d. to determine when and how much to reorder

Which of the following is a common cause of salmonella in a salad bar? a. fresh fruit b. egg salad c. lettuce and tomatoes d. pickles

egg salad

A natural plant hormone that is used to ripen fruits is: a. methanol b. ethylene c. acetyl acid d. ethanol

ethylene

Which of the following has the least amount of saturated fat? a. 2% milk b. half and half c. non-dairy creamer d. evaporated milk

evaporated milk

Fat produces more calories than carbohydrates because: a. fat has more carbon and hydrogen in relation to oxygen b. fat has less carbon and hydrogen in relation to oxygen c. fat is a larger molecule d. fat is a smaller molecule

fat has more carbon and hydrogen in relation to oxygen

The statement "The county's infant mortality rate is 12.8, and the state's infant mortality rate is 10.5" means that: a. more infants died in the county than in the state b. for every 1000 live births in the county, 12.8 infants died c. two more infants died in the county than in the state d. 12.8% of the infants born in the country died before the age of 1 year

for every 1000 live births in the county, 12.8 infants died

Pernicious anemia is likely to occur following a: a. colectomy b. colostomy c. gastrectomy d. esophagostomy

gastrectomy

In the fed state, the brain uses which nutrient as a source of energy? a. fatty acid b. lactic acid c. glucose d. ketone bodies

glucose

Lactic acid is produced by: a. glycolysis b. gluconeogenesis c. glycogenesis d. lactolysis

glycolysis

Which of the following has the most bacteria? a. roast chicken b. ground beef c. roast veal d. roast pork

ground beef

Which of the following is the most useful for evaluating long term nutrition in a community assessment program? a. height for age b. weight for height c. weight for age d. length for weight

height for age

An advantage of using focus groups to collect data is that they: a. help you generalize to large populations b. help the researcher understand participants' perspectives c. help participants stay focused on the people d. are inexpensive because many can be interviewed at once

help the researcher understand participants' perspectives

A commercial muffin mix is described as a "rich formula," which means it contains: a. high proportions of sugar and shortening b. high-fructose corn syrup instead of crystalline sugar c. high amounts of whey instead of nonfat dry milk solids d. more chemical leavening than is used in a "lean formula"

high proportions of sugar and shortening

A serum sodium level of 150 mEq/L is a sign of: a. very high sodium intake b. very low sodium intake c. hyponatremia d. hypernatremia

hypernatremia

A serum sodium level of 150 mEq/L may be due to: a. in increase in the intake of sodium-rich foods b. decreased steroid formation c. in increased level of aldosterone d. a decrease in the intake of sodium-rich foods

in increased level of aldosterone

Beef bottom round should be cooked: a. at a high temperature for a short time b. in water for an extended time c. in dry heat d. in moist heat for a short time

in water for an extended time

The number of individuals in a population diagnosed with a specific condition during a specified time is the: a. prevalence b. incidence c. rate of the conditions d. duration of the conditons

incidence

The two financial statements primarily used by foodservice managers to analyze operational effectiveness are the [MGT:343] a. inventory valuation record and budget. b. balance sheet and inventory valuation record. c. budget and income statement. d. income statement and balance sheet.

income statement and balance sheet.

How can you preserve thiamin when cooking pork? a. use the drippings after the fat has been removed b. insert a meat thermometer and cook it to 160 'F in an oven at 325 'F c. increase the cooking temperature to lessen cooking time d. add a small quantity of water during cooking

insert a meat thermometer and cook it to 160 'F in an oven at 325 'F

Which of the following inhibits gluconeogenesis? a. glycogen b. insulin c. catecholamines d. glucocorticoids

insulin

How do you begin an interview with a patient? a. obtain height and weight b. get a nutritional history c. introduce yourself and make them feel comfortable d. give the diet instruction

introduce yourself and make them feel comfortable

A client's failure to become involved in a breast-feeding program may be due to: a. lack of funds b. lack of social support c. lack of knowledge about benefits d. lack of medical care

lack of social support

The Cori Cycle converts: a. glucose into galactose b. lactate into pyruvate c. lactic acid into glycogen d. glycogen into glucose

lactate into pyruvate

Which of the following best encourages a client to take responsibility for making appropriate dietary changes? a. "Do you think you will be able to do that?" b. "Will you have family support for these changes?" c. "What change will you be comfortable trying first?" d. "These changes should not be too difficult for you, should they?"

"What change will you be comfortable trying first?"

Which of the following responses by a counselor indicates "reflections" ? a. "Yes, you can do that." b. "You said, 'You want to lose weight?'" c. "Explain what you mean by that, please." d. "I can help you."

"You said, 'You want to lose weight?'"

A child is at the 85th percentile for BMI. He is: a. overweight b. tall for his age c. muscular for his age d. normal weight

overweight

A behavioral strategy for weight control intended to limit exposure to food would require an individual to: a. record daily food intake b. portion the food before serving at the table c. clear the table immediately after eating d. put the fork down between bites

portion the food before serving at the table

What type of learning is used when you learn certain physical skills that require physical manipulation? a. affective b. cognitive d. psychomotor d. ambidextrous

psychomotor

When you measure functional and manipulative skills, which domain of learning are you analyzing? a. conceptual b. affective c. human d. psychomotor

psychomotor

Which vitamin acts as a coenzyme in transamination? a. pyridoxine b. thiamin c. riboflavin d. vitamin B12

pyridoxine

In the Cori cycle, lactate is converted into: a. ribose b. NADPH c. pyruvate d. acetyl CoA

pyruvate

The main function of glycolysis is to provide: a. pyruvate for the TCA cycle b. lactic acid for energy use by muscles c. muscle and liver glycogen d. a substrate for the Cori cycle

pyruvate for the TCA cycle

Which of the following is an achievable outcome for a community weight-control program? a. promote desirable body weight among young adults b. increase by 25% the number of physicians who refer patients for weight-management counseling c. screen and refer for treatment within two years those whose body weight is >10% desirable d. reduce by 10% the number of obese adults, ages 25 - 49 years, within 2 years of starting the program

reduce by 10% the number of obese adults, ages 25 - 49 years, within 2 years of starting the program

The food and nutrition director and the food production manager discuss productivity of the trayline setup. The food production manager agrees to reorganize the trayline's hot and cold food stations. This interaction is an example of [MGT:378] a. linking processes. b. control c. coordinating elements. d. functional subsystems.

linking processes.

Which is not absorbed directly into portal blood? a. monosaccarhides b. amino acids c. medium chained fatty acids d. long chained fatty acids

long chained fatty acids

The most likely source of Clostridium perfringens is: a. egg b. meat c. fruit d. vegetable

meat

Clostridium perfringens is often found in: a. meat and gravied mixtures in large quantities b. fresh fruits and vegetables c. dairy mixtures d. uncured cheese

meat and gravied mixtures in large quantities

In interpreting statistical data, if half the observations are above the number 8, and half are below the number 8, the number 8 is the: a. mode b. median c. mean d. averge

median

Acid-base abnormalities in an uncontrolled diabetic are likely due to: a. respiratory alkalosis b. metabolic acidosis c. respiratory acidosis d. metabolic alkalosis

metabolic acidosis

Riboflavin, thiamin, and niacin are all involved in the: a. metabolism of carbohydrate b. metabolism of fat c. metabolism of protein d. production of amino acids

metabolism of carbohydrate

The incidence of xeropthalamia in children between the ages of 1 - 5 is 5:1000. This means: a. morbidity rate of xeropthalmia is 5 in every 1000 children ages 1 - 5 b. mortality rate of xeropthalmia is 5000 in this age group c. morbidity rate of xeropthalmia is 5000 in this age group d. mortality rate is 5 in every 1000 children aged 1 - 5

morbidity rate of xeropthalmia is 5 in every 1000 children ages 1 - 5

Which pasta contains egg? a. noodles b. spaghetti c. macaroni d. linguini

noodles

Which of the following can be reversed with vitamin A? a. xerophthalmia b. nyctalopia c. Wilson's disease d. homocysteinuria

nyctalopia

What should you do with leftover sliced beef from lunch? a. throw it out b. let the meat cool at room temperature and then serve within two days c. refrigerate immediately and then serve within two days d. keep in the warmer until dinner time

refrigerate immediately and then serve within two days

Fresh meat contamination comes mainly from: a. botulism b. staphylococcus c. clostridium d. salmonella

salmonella

Salmonella contamination in a foodservice can be best controlled by focusing on: a. meat delivery procedures b. sanitizing of the meat slicer c. serving less meat d. meat storage procedures

sanitizing of the meat slicer

A patient relates a story to the RD. The RD says: "The is happened to me, too." This is: a. reflection b. clarification c. attribution d. self-disclosure

self-disclosure

The amino acid tryptophan can be converted into: a. serotonin b. catecholamines c. phenylalanine d. tyrosine

serotonin

All baking powders contain: a. sodium bicarbonate b. tartrate c. phosphate d. acetate

sodium bicarbonate

Which best describes sorbitol and glucose? a. sorbitol is sweeter than glucose b. they each provide 7 calories per gram c. sorbitol is absorbed more slowly d. they convert into fructose

sorbitol is absorbed more slowly

Which data is needed to convert weight into volume? a. mass b. temperature c. specific gravity d. pH

specific gravity

Why do egg whites beat more easily at room temperature? a. the pH is increased b. the protein is denatured c. the surface tension is lower d. the surface tension is higher

the surface tension is lower

When you check overrun, you are checking: a. excess stock in the storeroom b. surplus soap and rinse aids used in the dishmachine c. the weight of ice cream d. total costs of unexpected meals served

the weight of ice cream

If carbohydrate intake is increased, which vitamin needs to be increased? a. thiamin b. niacin c. riboflavin d. pyridoxine

thiamin

Which hormone greatly influences the BMR? a. the endrocrine hormone b. thyroxine c. the growth hormone d. insulin

thyroxine

Meat graded "US Standard" is the grade given at the: a. time of purchase b. time of delivery c. storage facility d. time of slaughter

time of slaughter

Which of the following amino acids is a precursor for serotonin? a. phenylalanine b. methionine c. tryptophan d. tyrosine

tryptophan

The flakiness of pie crust depends on: a. type of fat b. oxidation of fat c. temperature of the fat d. oven temperature

type of fat

An example of a survey with negative correlation is: a. when folic acid intake increases prior to pregnancy, the incidence of neural tube defects declines b. when LDL levels increase, the rate of heart disease increases c. when a person becomes more obese, their LDL levels likely rise d. as the intake of peanut butter and potato chips increases, dental caries increase

when folic acid intake increases prior to pregnancy, the incidence of neural tube defects declines

Which of the following must be reduced in simpler components before it can be transported across epithelial cells of the intestine? a. dipeptides b. whey c. casein hydrolysate d. crystalline amino acidss

whey

Which is most likely to be a cause of salmonella? a. fresh fruit salad b. canned fruit salad c. Casear salad d. avocado salad

Casear salad

store frozen fruits and vegetables at a. 0- -10F b. -10-0-20F c. <32F d. 0-10F

a. 0- -10F

A client returns for a follow-up visit after diet instruction and states: "I have not lost any weight on this diet." You should respond with: a. "Did you follow my advice?" b. "What is your typical day like?" c. "Did you follow the exercise plan?" d. "Have you been eating out?"

"What is your typical day like?"

Which denotes a high correlation? a. .2 b. .4 c. .6 d. .8

.8

How much fluid is lost through insensible losses? a. 0.5 liters b. 1 liter c. 2 liters d. 3 - 4 liters

1 liter

Fresh eggs in the shell can be stored in a refrigerator at 40 degrees for: a. 3 weeks b. 2 weeks c. 10 days d. 4 - 6 days

3 weeks

A #10 can holds how many cups of product? a. 10 b. 13 c. 12 d. 5

13

A #10 can holds how many cups? a. 10 b. 11 c. 13 d. 15

13

Normal saline has ______ mEq sodium/L. a. 154 b. 100 c. 50 d. 200

154

In a normal curve, what % of data falls between +1 and -1 standard deviation? a. 52 b. 93 c. 35 d. 68

68

What ready level is appropriate for written materials for a population with low literacy skills? a. 4th grade b. 6th grade c. 8th grade d. 9th grade

6th grade

The formula for producing the active form of vitamin D is: a. vitamin D3 converting into 7-dehydrocholesterol b. ergosterol converting into calciferol c. cholesterol converting into calciferol d. 7-dehydrocholesterol converting into cholecalciferol

7-dehydrocholesterol converting into cholecalciferol

Looking at the following data, what is this person's total insensible fluid loss? Input (ml) Output (ml) Oral fluids 2000 Urine 2100 Solid fluids 800 Intestinal 250 Metabolic water 250 Lungs and skin 700 Total 3050 Total 3050 a. 700 ml b. 950 ml c. 2100 ml d. 3050 ml

700 ml

What reading level is appropriate for educational material for the general population? a. 4th grade b. 5th grade c. 8th grade d. 10th grade

8th grade

Which of the following elements is NOT part of The Institute of Medicine's (IOM) framework for health care quality? [MGT:300] a. Patient-centered b. Equitable c. Timely d. Choose the process to improve

Choose the process to improve

Which food-borne pathogen would most likely contaminate cooked rice or pasta in open pans at room temperature? a. Clostridium perfringens b. Bacillus cereus c. Staphylococcus, Bacillus cereus d. Campylobacter jejuni, Bacillus cereus

Bacillus cereus

Food was held at 70 'F for a long time in anaerobic conditions. Eight hour after eating it you became ill. What is the culprit? a. Salmonella b. Staphylococcus c. Clostridium perfringens d. Campylobacter jejuni

Clostridium perfringens

What word best describes doing things correctly or getting the most output from the least amount of input? [MGT:318] a. Efficient b. Effective c. Economical d. Expedient

Efficient

Which of the following financial terms includes accumulated depreciation? [MGT:348] a. Fixed assets b. Owner's equity c. Liability d. Gross profit

Fixed assets

Which of the following is true? a. Fructose has sucrose b. Fructose is less concentrated than glucose c. Fructose take longer to digest than glucose d. Fructose is sweeter than glucose

Fructose is sweeter than glucose

Which of the following is correct? a. Glucose is less sweet then sucrose b. Lactose is sweeter than glucose c. Glucose is sweeter than fructose. d. Lactose is sweeter than fructose

Glucose is less sweet then sucrose

A dietitian is working with a client to reduce blood cholesterol levels. The client is knowledgeable about a hyperlipidemia diet, is able to devise healthful meal plans, but has moderate readiness in being independent in nutrition care. To be most effective, what would be the dietitian's approach? [MGT:332] a. Continue to provide dietary instructions to the client and closely monitor compliance. b. Help the client gain confidence through listening, advising, and coaching. c. Listen to the client. Encourage and support the skills the client has developed. d. Turn over responsibility for nutrition care to the client.

Listen to the client. Encourage and support the skills the client has developed.

Which "leadership powers" appear to be most important for compliance? [MGT:329] a. Expert, referent b. Legitimate, expert c. Referent, reward d. Reward, legitimate

Legitimate, expert

Which of the following ratios indicates an organization's ability to meet current financial obligations? [MGT:350] a. Liquidity b. Operating c. Activity d. Profitability

Liquidity

Which ratio is designed to examine how effectively an organization is utilizing its assets [MGT:352] a. Assets to liabilities b. Debt to equity c. Inventory turnover d. Break-even point

Inventory turnover

If linoleic acid replaces carbohydrates in the diet: a. LDL decreases, HDL increases b. LDL and HDL decrease c. LDL and HDL increase d. LDL and HDL increase

LDL decreases, HDL increases

Which statement best describes the "A" component of the PDSA performance improvement model? [MGT:305] a. Determine how changes will be implemented b. Implement the intervention c. Determine the impact of the intervention d. Maintain and continue improvement

Maintain and continue improvement

In a foodservice operation, which of the following is the most critical point of control? [MGT:391] a. Menu b. Overtime c. Quality d. Spoilage

Menu

Which of the following is NOT an element of Chronic Care Model (CCM)? [MGT:298] a. Self-management support b. the community c. Clinical information systems d. Motivation

Motivation

Dry heat cookery should be used for which cut of meat? a. shoulder b. neck c. brisket d. near the backbone

NEAR THE BACKBONE

What term is used to indicate the monetary value of a property or business beyond any amounts owed? [MGT:349] a. Total assets b. Owner's equity c. Long-term liabilities d. Gross profit

Owner's equity

____________ should be up to date and reinforced with strong supervision. [MGT:386] a. Personnel policies b. Employee evaluations c. Schedules d. Employee morale

Personnel policies

The average contribution margin (CM) per item on a menu is $6.74 and the menu mix (MM) percentage that is considered to be high is 12. A menu item has a CM of $6.93 and a MM percentage of 11. What menu engineering term would be used to describe this menu item? [MGT:362] a. Plowhorse b. Star c. Dog d. Puzzle

Puzzle (puzzle=high CM, low MM, dog=low CM low MM, star=high CM, high MM, plowhorse=low CM, high MM)

Which statement best describes the first "S" component of the FOCUS-PDSA performance improvement model? [MGT:310] a. Identify a process to improve b. Develop a team c. Explain the current process d. Recognize sources of variation

Recognize sources of variation

Systems thinking is the management approach to considering the entire organization when making decisions or allocating resources. Which is an example of systems thinking? [MGT:392] a. Select a produce vendor who could deliver high-quality produce twice daily. b. Reschedule kitchen staff to prepare more foods from scratch to meet customer expectations. c. Develop a catering menu for department meetings to generate revenue for equipment purchases. d. Select cleaning chemicals that do not harm the environment.

Reschedule kitchen staff to prepare more foods from scratch to meet customer expectations.

Butter has fatty acids in this order of predominance: a. PUFA, MUFA, SFA b. SAT, MUFA, PUFA c. MUFA, SAT, PUFA d. MUFA, PUFA, SAT

SAT, MUFA, PUFA

The formula that counts the number of polysyllabic words in text to calculate the reading level is the: a. SMOG index b. BACM index c. Smith Reading index d. Text Index

SMOG index

A cutting board was used to cut up raw chicken. It was then used to cut up raw vegetables. Guests eating the meal became ill most likely from: a. Clostridium botulinum b. Clostridium perfringens c. Salmonella d. Streptococcus

Salmonella

After reviewing the results of menu engineering, what is the best way to manage a plowhorse? [MGT:363] a. Remove the item from the menu. b. Decrease the menu item's selling price. c. Serve smaller portions of the menu item. d. Give the item top priority on the menu.

Serve smaller portions of the menu item.

An employee finds job tasks to be too demanding. To enhance the employee's performance, which of the following solutions should a manager consider first? [MGT:325] a. Provide assistance, equipment, and raw materials as deemed appropriate b. Simplify tasks, lessen the work load, and reduce time pressures c. Provide guidance concerning expected goals and standards d. Redesign the job to match the employee's needs

Simplify tasks, lessen the work load, and reduce time pressures

Which of the following is a grade for fresh produce? a. US Select b. US Grade A c. US No. 1 d. US Standard

US No. 1

Which of the following methods is most effective for marketing a nutrition program to an inner-city, Hispanic population? a. Spanish-language brochures written at high school reading level b. public service announcements on TV c. radio messages d. Spanish food demonstrations at local grocery stores

Spanish food demonstrations at local grocery stores

Which government agency inspects and grades meat? a. FDA b. DHHS c. USDA d. Department of Commerce

USDA

The amount of ceruloplasmin is affected by what disease? a. Addison's disease b. Pheochromocytoma c. Cushing's syndrome d. Wilson's disease

Wilson's disease

The DTR receives information from the patients about food temperatures. The line cooks receive information from the RD about recipe modifications. The dishroom provides information for the health inspector. Are these interactions examples of feedback? [MGT:370] a. No, because feedback is only from the external environment. b. Yes, because feedback can be from the internal and external environments. c. It cannot be determined from the information given. d. No, because feedback flows from the bottom to the top of an organization.

Yes, because feedback can be from the internal and external environments.

how many quarts of batter are needed to make 144 muffins using a #12 scoop a. 12 b. 16 c. 22 d. 6

a 12 (32/12 x 144/32)

where can a pregnant teen learn to cook a. EFNEP b. Child Nutrition Services c. UNICEF d. WIC

a. EFNEP

a poor, healthy 19 year old mother is unsure how to feed her 4 year old child. which program may offer her the most help a. EFNEP b. WIC c. HHS d. CACFP

a. EFNEP (improves food practices of low income homemakers with young children, works with small groups, teaches skills needed to obtain a healthy diet)

when goods are received, the delivery slip included is a. an invoice b. a standing order c. the receiving record d. a purchase requisition

a. an invoice

which survey is repeated every few years a. NHANES b. WHO c. Ten state d. FAO

a. NHANES

you are developing a heart disease prevention program for a worksite and need to locate several nutrition research articles. the best database to use is: [NTR:537] a) PubMed b) Merck Index Online c) Scopus c) The Cochrane Library

a. PubMed

which of these work center designs discourages through traffic a. U-shaped b. back to back parallel c. straight line d. square

a. U-shaped

which government agency would you contact regarding food distribution a. USDA b. FDA c. HHS d. Foods Stamps

a. USDA

the dietary intake component of NHANES is a. WWEIA b. BRFSS c. Ten State Nutrition Survey d. PNSS

a. WWEIA (What We Eat In America - the dietary intake interview portion of NHANES)

Ground meat will spoil faster than sliced meat because: a. it has more enzyme activity b. it has more fat c. more surface ares is exposed d. it has less fat

more surface ares is exposed

Cured meat is pink due to: a. sugar b. salt c. vinegar d. nitrites

nitrites

A 5 month old is diagnosed with salmonella. What is the likely cause? a. fruit juice b. milk c. table food d. evaporated milk

table food

An appropriate purpose of analytical research is to ______. a. test the hypothesis b. investigate the... c. formulate a hypothesis... d. distinguish a hypothesis...

test the hypothesis

approximately how many patient trays per minute must a centralized tray assembly unit produce in order to serve 400 patients during 90 minutes a. 3 b. 3.5 c. 4 d. 4.5

d. 4.5 (400/90)

you have determined that an infant should take 340 calories/24 hours. how many ounces of milk should be given to the baby every 6 hours a. 2 oz b. 3 oz c. 3.5 oz d. 4.5 oz

d. 4.5 oz (approximately 20kcal/ounce)

the width of a main kitchen aisle should be a. 24" b. 36" c. 48" d. 60"

d. 60"

with a CHO:insulin ratio of 16:1, how much insulin is needed with this meal. 1 1/3 cups spaghetti, 1 cup spaghetti sauce, 1 cup green beans, 1 slice bread, 1 teaspoon butter, 1/2 cup ice cream a. 4 units b. 5 units c. 6 units d. 8 units

d. 8 units

which of the following is not a governmental agency a. Health and Human Services b. Food and Drug administration c. Food and Nutrition Board d. American Heart Association

d. American Heart Association

which of the following is lowest in fat a. hamburger b. salami c. goat cheese d. Canadian bacon

d. Canadian bacon

a family receiving food stamps often runs out of food by the middle of the month. obtain an emergency food voucher for them and refer them to a. a congregate meal site b. a social services agency c. WIC d. EFNEP

d. EFNEP (teaches clients how to improve food practices, how to budget, meal plan, shop and cook)

what organization regulates the irradiation of foods a. DHHS b. USDA c. department of commerce marine fisheries service d. FDA

d. FDA

which of the infants in the above question have a normal hemoglobin level a. a, b, d b. b, d c. c, d d. a, b, c

d. a, b, c (infant hemoglobin levels range from 9.5-20g/dL dependent on their age)

employees complain the performance appraisal process is highly subjective and unfair. the manager should prepare objectives that a. meet the mission and goals of the department b. are developed with input from the employees c. emphasize the employee's best skills d. are clear and reasonable

d. are clear and reasonable

which of the following would be classified as NI in the NCP a. not ready for diet change b. altered GI function c. involuntary weight loss d. increased energy expenditure

d. increased energy expenditure

quality circles are useful in a. employee evaluations b. job selection and placement c. policy development d. increasing productivity

d. increasing productivity

which of the following forms of pudding would be purchased in an assembly/serve foodservice system a. frozen bulk b. instant c. canned d. individual portions

d. individual portions

Results of a three day food record were obtained in June and again three months later. What is this called? a. validity b. reliability c. sensitivity d. specificity

reliability

A frozen meal contains steak, mashed potatoes, BHA, BHT and TVP. The TVP is used to: a. add color b. improve taste c. retain water and add protein d. act as an emulsifier

retain water and add protein

If linoleic acid replaces saturated fat in the diet: a. total cholesterol including HDL decreases b. HDL increases c. LDL increases c. no change in lipid levels

total cholesterol including HDL decreases

Which amino acids must be present in parenteral solution? a. valine, leucine, glycine, alanine b. tryptophan, phenylalanine, threonine, isoleucine c. phenylalanine, methionine, arginine, tyrosine d. lysine, glutamic acid, alanine, glycine

tryptophan, phenylalanine, threonine, isoleucine

Which of the following should be avoided on a low sodium diet? a. pork loin b. turkey roll c. beef sirloin d. lamb chop

turkey roll

When a white sauce tastes starchy and grainy, the likely cause is: a. too much starch b. improper proportions of ingredients c. starch was added before the fat melted d. uncooked flour

uncooked flour

To decrease sodium intake, substitute: a. carrots for celery b. unsalted margarine for regular margarine c. diet margarine for regular margarine d. carrots for pepper

unsalted margarine for regular margarine

When counseling patients who have limited ability to speak and understand your language, you should: a. provide only verbal instructions b. use demonstrations and food models c. instruct the patient's family d. refer the patient to a language class

use demonstrations and food models

When substituting butter for lard in pastry: a. use the same amount b. use less c. use more d. use less and decrease the water content

use more

Homemade rolls are crumbly and fall apart easily. The most likely cause of the poor texture in the yeast bread is: a. use of a high-protein flour b. use of a lower-protein flour c. insufficient proofing time d. excessive proofing time

use of a lower-protein flour

You are a Food Service Dietitian and would like to make lower fat brownies. What is the best way to cut the fat? a. use margarine instead of butter b. cut out all the fat c. cut out half the fat d. use prune puree in place of half the fat

use prune puree in place of half the fat

when a supervisor says to a subordinate, do it because I am asking you to; i'm the boss, they are using what type of power a. position b. referent c. personal d. incumbent

a. position

Carbohydrate, fat and protein are all converted into: a. pyruvic acid b. acetyl CoA c. oxaloacetic acid d. ketoglutamic acid

acetyl CoA

The oxidation of fatty acids forms: a. acetyl CoA b. pyruvic acid c. lactic acid d. oxaloacetate

acetyl CoA

What is the product of pyruvate breakdown? a. acetyl CoA b. glycogen c. NADPH d. glucose-6-phosphate

acetyl CoA

A food is bacteria-safe if it is: a. neutral and moist b. neutral and dry c. acidic and dry d. acidic and moist

acidic and dry

Iron absorption is enhanced by: a. acidic chyme in the duodenum b. the alkalinity of the duodenum c. oxalates and phyates d. the abscence of ascorbic acid

acidic chyme in the duodenum

The statement "After class, the patient will be able to write a day's menu following the guidelines" is.... a. a goal b. an objective c. a plan d. a research hypthesis

an objective

The management functions of planning, organizing, and staffing: [MGT:379] a. are linked directly to transformation of inputs to outputs. b. are used to coordinate subsystems to achieve objectives. c. are linking processes for communication and decision making. d. express how objectives are achieved.

are used to coordinate subsystems to achieve objectives.

What should you do with a new client who sits scowling at you during the session? a. presume she understands what you are saying b. ask her if she is uncomfortable about the session c. recognize that new clients are seldom relaxed d. presume that she just would rather be somewhere else

ask her if she is uncomfortable about the session

A client refuses to follow the diet after you have explained the plan and its rationale. You should then: a. reinforce the rationale and benefits of the diet b. review the dangers of non-compliance c. ask what changes he is willing to make d. restate what the client needs to do to meet the goals of the diet

ask what changes he is willing to make

343. The tricarboxylic cycle begins with the conversion of pyruvate to [NTR:638] a) acetyl CoA. b) oxaloacetate. b) ATP. d) hydrogen ions.

b) oxaloacetate.

94. Overeating and poorer self-regulation of energy intake in preschool age children is associated with [NTR:489] a) excessive soft drink consumption b) parents assuming control of food portions and coercing children to eat c) family style meal service d) excessive food choices and family style meals

b) parents assuming control of food portions and coercing children to eat

15. In the Nutrition Care Process a PES statement consists of: [NTR:410] a) possibilities, effects, solutions. b) problem, etiology, sign/symptoms. c) people, efforts, satisfaction. d) probabilities, end products, significant information.

b) problem, etiology, sign/symptoms.

241. One of the primary goals of WIC food supplements is [NTR:535] a) increasing income for participating grocery stores. b) reducing or preventing iron deficiency anemia in infants and children. c) providing low-cost lunches to school-age children. d) increasing the number of women who get prenatal care.

b) reducing or preventing iron deficiency anemia in infants and children.

59. The loss of muscle mass that occurs with aging is termed: [NTR:454] a) cachexia. b) sarcopenia. c) dystrophy. d) myotonia.

b) sarcopenia.

311. Body Mass Index (BMI) is a better indicator of obesity than body weight because BMI is_____. [NTR:605] a) the ratio of height to weight b) the ratio of weight to height c) determined by indirect calorimetry d) determined by direct calorimetry

b) the ratio of weight to height

An example of a conditionally indispensable amino acid is: [NTR:572} a) tryptophan. b) tyrosine. c) alanine. d) leucine.

b) tyrosine.

45. Children and adolescents consuming a vegetarian diet may be at risk for a deficiency of [440] a) vitamin D b) vitamin B12 c) vitamin E d) vitamin B6

b) vitamin B12

adulterated food is regulated by the a. HHS b. FDA under the food, drug, cosmetic act c. Department of commerce d. USDA

b. FDA under the food, drug, cosmetic act

a mother with 2 daughters, ages 8 and 11, does not earn enough money to feed her family. which program would you suggest a. WIC b. Food Stamps c. Child Care Food Program d. Medicaid

b. Food Stamps

a single man with two daughters, ages 10 and 11, is looking for aid. which program might provide assistance a. WIC b. Food Stamps c. Child Care Food Program d. Headstart

b. Food Stamps

levodopa is used in the treatment of a. rheumatoid arthritis b. Parkinson's disease c. diabetic ketoacidosis d. nephrotic syndrome

b. Parkinson's disease

to learn about prenatal problems in the community, go to a. center for disease control b. WIC c. NHANES d. FDA

b. WIC

skinfold thickness measurements are best for assessing a. an 18 month old with growth failure b. a 36 year old athlete c. a 54 year old woman with significant weight loss d. an 82 year old man with diabetes

b. a 36 year old athlete

if you increase the size of the kitchen, what effect will there be on worker efficiency a. an increase b. a decrease c. no change d. depends on the menu

b. a decrease

trauma can place the patient in a. an anabolic state b. a hypermetabolic state c. a hypothermic state d. a hypometabolic state

b. a hypermetabolic state

who is the most appropriate candidate for parenteral nutrition a. a 12 year old girl coming in for an appendectomy b. a middle aged woman who will begin chemotherapy c. an elderly lady with emphysema d. a young man going for gallbladder surgery

b. a middle aged woman who will begin chemotherapy

what is the major reason for installing automated equipment a. easier to use b. rising labor costs c. easier to clean d. greater flexibility

b. rising labor costs

the ketogenic diet is used to help treat a. cystic fibrosis b. seizures c. chronic renal failure d. hypotension

b. seizures

methylene is added to some types of coffee to a. retain the flavor b. retain the color c. decaffeinate it d. destroy the bacteria

c. decaffeinate it

work simplification studies a. looks at the job as a whole b. increase worker responsibility c. decrease costs and increases productivity d. increase worker interest in the job

c. decrease costs and increases productivity

the appropriate diet therapy following a Billroth II is a. high carbohydrate, low fat, high protein b. high fat, high protein, high carbohydrate c. decrease simple carbohydrates, high protein, moderate fat, B12 supplements d. low fat, low protein, high carbohydrate

c. decrease simple carbohydrates, high protein, moderate fat, B12 supplemetns

a female has a 36" waist and a 40" hip measurement. she is at risk for a. nothing b. cancer c. heart disease d. malnutrition

c. heart disease

which bacteria may cause gastric ulcers a. salmonella b. streptococcus aureus c. helicobacter pylori d. clstridium perfringens

c. helicobacter pylori

which fiber is insoluble in water a. pectin b. gums c. hemicellulose d. polysaccharides

c. hemicellulose

the diet for hepatic coma is low in protein due to a. low ammonia levels b. high cholesterol levels c. high ammonia levels d. high LDL

c. high ammonia levels

a patient is diagnosed with iatrogenic malnutrition. nutritional therapy should be a. high calorie b. high protein c. high calorie, high protein d. low calorie, high protein

c. high calorie, high protein

compared to human milk, commercially prepared infant formula is a. similar in nutrient content b. equivalent to human milk in nutrient content and antibodies c. higher in protein and iron but lacks antibodies d. lower in protein and iron and lacks antibodies

c. higher in protein and iron but lacks antibodies

an alcoholic patient has a low serum albumin, low hemoglobin, high ammonia, abnormal liver function tests and ascites. why is he on a 500mg sodium diet a. secondary renal failure b. hypoaldosteronism c. hypoalbuminemia d. hyperammonemia

c. hypoalbuminemia (low sodium with ascites, hypoalbuminemia is often caused because of dilution due to fluid and sodium retention)

a PN patient is receiving 45 calories/kg. what may be the result a. hypocalcemia b. hyperkalemia c. hypokalemia d. hyperphosphatemia

c. hypokalemia (refeeding syndrome may occur which results in starved cells taking of nutrients, P and K shifting into intracellular compartments causing hypokalemia, hypophosphatemia and hypomagnesemia)

the first step in planning nutrition services for a new community health center should be to a. survey the area to determine what services are readily available b. establish goals and objectives for the services to be offered c. identify nutrition related health problems in the community d. develop program protocols for client care

c. identify nutrition related health problems in the community

what is the most effective tool for providing nutrition education to the community a. the RDIs b. the report healthy people c. my pyramid d. the healthy eating index

c. my pyramid

the essential amino acid tryptohan is most closely related to a. folic acid b. riboflavin c. niacin d. pantothenic acid

c. niacin

which food should someone on an MAOI not eat a. cottage cheese b. herbal tea c. pepperoni d. butter

c. pepperoni

349. Canned corn has more sodium and less potassium than fresh corn. This is the result of [NTR:644] a) processing b) enrichment c) fortification d) disassociation

a) processing

A foodservice operation's Net Profit is $41,137, Cost of Goods Sold is $145,350, Average Inventory Value is $7,330, and Total Meals Served Per Day is 200. What is the inventory turnover rate? [MGT:354] a. 3.53 b. 5.61 c. 19.83 d. 36.65

19.83 (145350/7330)

What is the mean of the following set of numbers? 3, 2, 2 a. 3 b. 2.3 c. 3.2 d. 2.5

2.3

What number of productive FTEs do you need for two 7-day-per-week positions, working 6:30 am to 3:00 pm? [MGT:388] a. 1.4 b. 2 c. 2.8 d. 2.975

2.8 (40+16 = 56/40 = 1.4x2 = 2.8) shift is an 8 hours shift if you subtract a 30min lunch break

Food service employees should wash their hands for at least: a. 30 seconds b. 25 seconds d. 20 seconds c. 15 seconds

20 seconds

A study showed the morbidity of children with anemia to be 5:1000. This means: a. 5 children out of 1000 died of anemia b. 5 children out of 1000 had anemia c. 1000 children had anemia d. 1000 children died of anemia

5 children out of 1000 had anemia

How many calories are needed by a full-term infant weighing 11 pounds at 3 months of age? a. 108 b. 365 c. 540 d. 720

540

What percent of protein and fat converts to glucose? a. 58% protein, 10% fat b. 10% protein, 60% fat c. 30% protein, 30% fat d. 20% protein, 40% fat

58% protein, 10% fat

expected creatinine clearance is estimated from the client's a. weight and height b. TSF, AMC c. height d. waist circumference

a. weight and height

39. Estimated energy needs for a client should be based on: [NTR:434] a) resting metabolic rate. b) basal metabolism. c) physical activity level. d) body composition.

a) resting metabolic rate.

Which statement best describes the "F" component of the FADE performance improvement model? [MGT:311] a. Choose a specific problem or process to improve b. Review data to better understand the current process c. Create a plan d. Complete a pilot study

Choose a specific problem or process to improve (Focus - define a process to be improved, Analyze - Collect and analyze data to establish baselines, identify root causes and point toward possible solutions, Develop - Based on the data, develop action plans for improvement, including implementation, communication, and measuring/monitoring, Execute/Evaluate - Implement the action plans, on a pilot basis as indicated, and Install an ongoing measuring/monitoring (process control) system to ensure success.)

Which of the following collection of words represents marketing? [MGT:358] a. Emphasis on product, sales-volume-oriented, needs of seller b. Customer wants, profit-oriented, future growth c. Sales-volume-oriented, customer wants, planning for today d. Profit-oriented, future growth, needs of seller

Customer wants, profit-oriented, future growth

A serum sodium level of 150 mEq/L indicates: a. sepsis b. dehydration c. hypertension d. hyperglycemia

DEHYDRATION

During a snow storm, the assistant cook (AC) can't get to work. The person working in the "floater" position is unable to take over all AC responsibilities but is somewhat confident and is willing to assist the head cook in preparing items on the daily production schedule. To be most effective, what would be the manager's approach? [MGT:331] a. Provide specific instructions (i.e., do this) to the willing employee and closely supervise performance. b. Define roles and tasks for the willing employee, provide opportunity for role clarification and questions, and give support. c. Listen to the willing employee, share ideas, facilitate in decision making, encourage, and praise. d. Turn over responsibility for decisions and implementation to the willing employee and head cook. Provide occasional recognition.

Define roles and tasks for the willing employee, provide opportunity for role clarification and questions, and give support.

Which market segmentation variable consists of characteristics such as age, gender, race, ethnicity, income, education, and occupation? [MGT:360] a. Geographic b. Demographic c. Psychographic d. Sociographic

Demographic

Which statement best describes the "P" component of the PDSA performance improvement model? [MGT:302] a. Determine how changes will be implemented b. Implement the intervention c. Determine the impact of the intervention d. Implement and monitor the plan

Determine how changes will be implemented

When using the plan-do-study-act (PDSA) process, which of the following occurs during the plan phase? [MGT:301] a. Determine how to collect data b. Train staff on new process c. Evaluate the effectiveness of the project d. Reduce wait time for appointments

Determine how to collect data

Which statement best describes the "S" component of the PDSA performance improvement model? [MGT:304] a. Determine how changes will be implemented b. Implement the intervention c. Determine the impact of the intervention d. Maintain and continue improvement

Determine the impact of the intervention

an 18 month old child should receive a. whole milk b. 2% milk c. 1% milk d. skim milk

a. whole milk

Which statement best describes the "O" component of the FOCUS-PDSA performance improvement model? [MGT:307] a. Identify a process to improve b. Develop a team c. Explain the current process d. Recognize sources of variation

Develop a team (Find a process to improve, Organize an effort to work on improvement, Clarify currant knowledge of the process, Understand process variation and capability, Select a strategy for continued improvement)

Which "leadership powers" tend to be related to subordinate satisfaction and performance? [MGT:328] a. Expert, referent b.Legitimate, expert c.Referent, reward d. Reward, legitimate

Expert, referent

Which of the following substitutions will most decrease the sodium content of a biscuit? I. Low sodium milk for whole milk II. Egg substitute for egg III. Low sodium baking powder for baking powder a. I b. I, II c. all of the above d. I, III

I, III

Which statement best describes the "D" component of the PDSA performance improvement model? [MGT:303] a. Determine how changes will be implemented b. Implement the intervention c. Determine the impact of the intervention d. Maintain and continue improvement

Implement the intervention

Which set of financial management terms has the correct relationship? [MGT:344] a. Income statement, profit and loss, dynamic statement b. Balance sheet, profit and loss, static statement c. Income statement, statement of financial condition, static statement d. Balance sheet, statement of financial condition, dynamic statement

Income statement, profit and loss, dynamic statement

The Joint Commission outlines patient safety goals annually. Which of the following is NOT a patient safety goal in an outpatient setting? [MGT:299] a. Reconcile medication list b. Identify patients correctly c. Prevent infections d. Individual competence

Individual competence

Which of the following is NOT a main component of continuous quality improvement (CQI)? [MGT:313] a. Customer satisfaction b. Data collection c. Team involvement d. Multidisciplinary rounds

Multidisciplinary rounds

What name is given to the chronological sequence of activities to carry out organizational behavior? [MGT:322] a. Policy b. Procedure c. Method d. Rule

Procedure

Yogurt manufacturers have branched out into Greek yogurt, high-fiber yogurt, parfait yogurt, bacteria-enhanced yogurt, and more because they believe customers prefer yogurt products. By doing so, which marking concept are these manufactures applying? [MGT:357] a. Product b. Selling c. Manufacturing d. Societal

Product

During a snow storm, the assistant cook (AC) can't get to work. The person working in the "salads" position is unable to take over all AC responsibilities but is somewhat confiÞnt and willing to assist the head cook in preparing items on the daily production schedule. To be most effective, which leadership style should the foodservice manager use? [MGT:330] a. Telling b. Selling c. Participating d. Delegating

Selling

A storeroom staffed with a full-time employee would be considered a(n) ___________ of a university residence hall foodservice. [MGT:373] a. Input b. Subsystem c. Interface d. technical core

Subsystem

What name is given to standard methods of accounting and presentation of financial statements that provide comparability of financial data within an industry? [MGT:342] a. Balance sheet b. Common-size statements c. Uniform systems of accounts d. Cost accounting

Uniform systems of accounts

Nitrogen equilibrium is associated with: a. a 25 year old pregnant female b. a 25 year old male c. a 15 year old girl d. a 4 year old child

a 25 year old male

Glycerol, amino acids and free fatty acids will have what effect on the hypothalamus gland? a. an increase in satiety b. a decrease in satiety c. an increased activity level d. increased lethargy

a decrease in satiety

If you replace half and half with whipping cream in ice cream, the effects on the ice crystals will be: a. an increase in size because there is less fat b. a decrease in size because there is more fat c. no change d. an increase in size because there is more fat

a decrease in size because there is more fat

To increase potassium intake, consume a snack of: a. cheese and crackers, plums, cranberry juice b. a bagel, milk and an apple c. a grilled cheese and tomato sandwich and strawberries d. tuna salad, a wheat roll, and cola

a grilled cheese and TOMATO sandwich and strawberries

Synergistics is: a. two people working together b. two people working in separate areas c. a person working alone d. a group of people working together to solve a problem

a group of people working together to solve a problem

A provitamin is: a. a nutrient that can be converted into a vitamin b. a vitamin that can be converted into a nutrient c. the endogenous form of a vitamin d. a man-made copy of a vitamin

a nutrient that can be converted into a vitamin

234. Protein needs for patients with Stage II pressure ulcers can be calculated as [NTR:528] a) 1.2 - 1.5 g of protein/kg body weight b) 1.1 - 1.2 g of protein/kg body weight c) 1.5 - 1.7 g of protein/kg body weight d) 0.8 - 1.0 g of protein/kg body weight

a) 1.2 - 1.5 g of protein/kg body weight

Full-time employees generally work 236 days per year due to vacation and benefit days. Approximately how many employees are necessary for everyday coverage of full-time positions? [MGT:339] a) 1.55 b) 1.75 c) 0.75 d) 0.55

a) 1.55

31. Approximately how many grams of protein does one cup of raw vegetables contain? [NTR:426] a) 2 grams b) 3 grams c) 7 grams d) 8 grams

a) 2 grams

28. Which of the following breakfast meals contains approximately 60 grams of carbohydrate? [NTR:423] a) 2 slices of toast, 1 egg, 1 small orange, 1 cup milk b) 2 slices of toast, 1 cup apple juice, 1 large banana c) 1 -1/2 cups of oatmeal, 1/4 cup blueberries, 1 cup orange juice, 1/2 cup milk d) 2 eggs, 1 slice bacon, 2 slices of toast, 1/2 cup orange juice

a) 2 slices of toast, 1 egg, 1 small orange, 1 cup milk

23. How many kilocalories per ounce does a standard infant formula provide? [NTR:418] a) 20 b) 22 c) 25 d) 28

a) 20

Guidelines to treat disorders of lipid metabolism in patients with diabetes include: [NTR:396] a) 25-35% of total energy from fat; less than 7% of total energy from saturated and trans fat. b) less than 25% of total energy from fat; less than 10% of total energy from saturated and trans fat. c) less than 15% of total energy from all fat. d) less than 200 mg of cholesterol per day; greater than 10% of total energy from saturated and trans fat.

a) 25-35% of total energy from fat; less than 7% of total energy from saturated and trans fat.

360. After protein digestion, what products predominately are absorbed into circulation? [NTR:655] a) Amino acids b) Amino acids and oligopeptides c) Amino acids and dipeptides d) Amino acids, dipeptides, and tripeptides

a) Amino acids

11. Which of the following growth charts is currently recommended by the Centers for Disease Control to assess weight in relation to stature in children ages 2 to 20 years? [NTR:406] a) BMI-for-age b) Length-for-age c) Weight-for-age d) Stature-for-age

a) BMI-for-age

338. The Emergency Food Assistance Program and The Disaster Feeding Program are two examples of Food Distribution Programs. Which of the following statements best describes the similarities of the two programs? [NTR:632] a) Both programs serve children and adolescents and provide commodity foods to local food distribution and service agencies. b) Both programs serve low-income populations and provide commodity foods to local food distribution and service agencies. c) Both programs serve low-income populations and supplemental food supplies are either distributed to individual households or as meals served in congregate dining centers. d) Both programs serve children and adolescents and food supplies are either distributed to individual households or as meals served in congregate dining centers.

a) Both programs serve children and adolescents and provide commodity foods to local food distribution and service agencies.

4. Women with gestational diabetes typically require less carbohydrates at which meal compared to the others? [NTR:399] a) Breakfast (morning) b) Lunch (afternoon) c) Dinner (evening) d) Bedtime snack

a) Breakfast (morning)

87. Which is a functional food and has been labeled to have a qualified health claim for essential and gestational hypertension? [NTR:482] a) Calcium supplement b) Green tea c) Selenium supplement d) Dried tomatoes

a) Calcium supplement

315. Which of the following conditions would most likely result in a slightly elevated albumin? [NTR:609] a) Dehydration b) Cirrhosis c) Inadequate protein intake d) Stage IV pressure ulcer

a) Dehydration

98. What is the correct order of digestion, proximal to distal, of the small intestine? [NTR:492] a) Duodenum - Jejunum - Ileum b) Ileum - Jejunum - Duodenum c) Jejunum - Duodenum - Ileum d) Duodenum - Ileum - Jejunum

a) Duodenum - Jejunum - Ileum

225. Which of the following foods must a child with phenylketonuria avoid? [NTR:519] a) Egg b) Banana c) Rice d) French fries

a) Egg

57. Which of the following situations best describes why an older adult may become dehydrated? [NTR:452] a) Fear of incontinence b) Living alone c) Low income d) Poor intake of fruit

a) Fear of incontinence

Which agency within the US Department of Agriculture (USDA) participates in the review and update of the Dietary Guidelines for Americans [NTR:536] a) Food, Nutrition, and Consumer Services (FNCS) b) Research, Education, and Economics (REE) c) Center for Nutrition Policy and Promotion (CNPP) d) Expanded Food and Nutrition Education Program (EFNEP)

a) Food, Nutrition, and Consumer Services (FNCS)

301. The hormone released by gastric cells in response to food in the stomach is ______. [NTR:595] a) Gastrin b) Secretin c) Cholecystokinin d) chymotrypsin

a) Gastrin

291. Given identical amounts of cow's milk and human breast milk, which of the following is true? [NTR:585] a) Human milk contains less protein. b) Human milk provides more calcium. c) Cow's milk provides 50% of the calories. d) Cow's milk contains a lower proportion of casein compared to whey protein.

a) Human milk contains less protein.

232. A 72 year-old woman with chronic heart failure and symptomatic edema and who is on enteral feeding, is treated with thiazide medication. What would you expect her labs to reveal as a result of this medication and its delivery method? [NTR:526] a) Hypokalemia b) Hypernatremia c) Hypocalcemia c) Hypoglycemia

a) Hypokalemia

256. For Chinese Americans, challenges to dietary compliance typically include which of the following? [NTR:550] a) Inability to control portion sizes b) Reliance on non-nutritive herbs for flavor and color c) Size of portions is dictated by status in the family and community d) Frequent fasting in association with religious holidays

a) Inability to control portion sizes

262. In the Campniha-Bacote Model, cultural competence originates in the _____. [NTR:556] a) Individual b) Family c) Community d) society

a) Individual

71. According to the Nutrition Care Process, if a client takes self-prescribed therapeutic doses of Vitamin C, this information should be recorded in the _____ section of the Nutrition Diagnosis. [NTR:466] a) Intake b) Clinical c) behavioral-environmental d) nutrition knowledge

a) Intake

277. Which fatty acids are essential and must be obtained by eating foods containing these fatty acids? [NTR:571] a) Linoleic and α-linolenic fatty acids b) Cis monounsaturated fatty acids c) Trans fatty acids d) Saturated fatty acids

a) Linoleic and α-linolenic fatty acids

355. Where does carbohydrate digestion begin? [NTR:650] a) Mouth b) Stomach c) Small intestine d) Pancreas

a) Mouth

12. After completing this step in the Nutrition Care Process the dietetics professional can decide whether or not a nutrition diagnosis/problem exists. [NTR:407] a) Nutrition Assessment b) Nutrition Diagnosis c) Nutrition Intervention d) Nutrition Monitoring and Evaluation

a) Nutrition Assessment

13. Which of the steps in the Nutrition Care Process is a systematic method for obtaining verifying, and interpreting data needed to identify nutrition-related problems? [NTR:408] a) Nutrition Assessment b) Nutrition Diagnosis c) Nutrition Intervention d) Nutrition Monitoring and Evaluation

a) Nutrition Assessment

81. In the Nutrition Care Process, what should determine the dietetics practitioner's selection of a Nutrition Intervention? [NTR:476] a) Nutrition diagnosis and etiology b) Time and other available resources c) Client's history of compliance d) Practitioner's level of education and experience

a) Nutrition diagnosis and etiology

332. Cultural competence has been described as a set of congruent attitudes, behaviors, and _____. [NTR:626] a) Policies b) Models c) Values d) interventions

a) Policies

56. Which of the following are appropriate for your patient with celiac disease to eat? [NTR:451] a) Quinoa and tapioca b) Couscous and corn c) Oatmeal and buckwheat d) None of the above

a) Quinoa and tapioca

114. After having defined the nutritional problem, which of the following is the correct sequence of steps a community nutritionist should follow in assessing the needs of the community? [NTR:508] a) Set the parameters of the assessment, collect data, analyze and interpret the data, share the findings, and set priorities. b) Set priorities, collect data, set the parameters of the assessment, analyze and interpret the data, and share the findings. c) Choose a plan of action, collect data, set the parameters of the assessment, share the findings, and analyze and interpret the data. d) Collect data, analyze and interpret the data, set the parameters, share the findings, and set the priorities.

a) Set the parameters of the assessment, collect data, analyze and interpret the data, share the findings, and set priorities.

316. A key factor in reaching special needs groups is _____. [NTR:610] a) Simplicity b) Repetition c) providing examples d) providing scenarios

a) Simplicity

289. Which of the following foods is most appropriate for a level 2 dysphagia, mechanically altered diet? [NTR:583] a) Soft pancakes with extra syrup b) Pineapple c) Licorice candy d) Sausage

a) Soft pancakes with extra syrup

359. Which of the following dietary lipids is most heart healthy? [NTR:654] a) Vegetable oil in a squeezable bottle b) Margarine in a tub c) Vegetable oil in a stick d) Whipped margarine

a) Vegetable oil in a squeezable bottle

292. Resting energy expenditure (REE) is most affected by [NTR:586] a) body composition. b) physical activity. c) time of day. d) food intake.

a) body composition

53. When reviewing 2 years of serial data on a Centers for Disease Control and Prevention growth chart, a pediatric patient is identified to be at risk on nutritional screening when experiencing a [NTR:448] a) change in rate of growth. b) continuation on his or her line of growth. c) growth spurt. d) plateau in growth.

a) change in rate of growth.

255. The first step toward attaining cultural competence is for the dietetics professional to _____. [NTR:549] a) examine his/her own cultural background b) study the relevant cultural norms, values, and practicalities of the daily lives of clients c) identify cultural variations in family relationships d) note culturally-specific health beliefs and practices

a) examine his/her own cultural background

251. A community nutritionist can best elicit opinions as to why a particular nutrition program is not working by [NTR:545] a) forming a focus group. b) examining the results of the pre- and post-tests. c) sharing the problem with similar agencies. d) redefining the target population.

a) forming a focus group.

20. Which of the following foods is least likely to pose a choking risk for toddlers? [NTR:415] a) Ground beef b) Nuts c) Popcorn d) Round candy

a) ground beef

231. The best way to address the mortality rate associated with low-birthweight (LBW) and preterm infants is by [NTR:525] a) improving the birth weight of newborn infants. b) reducing the incidence of pica among pregnant women. c) improving the infant formulas for LBW and preterm infants. d) increasing the emphasis on breastfeeding in hospitals.

a) improving the birth weight of newborn infants.

80. When preparing a PES statement in situations where there are diagnoses from multiple domains, the dietetics professional should give preference to the diagnosis related to: [NTR:475] a) intake. b) physical conditions. c) knowledge. d) food safety.

a) intake.

238. Public health policy is generally created by [NTR:532] a) legislators. b) judges and magistrates. c) political action committees. d) the President and his cabinet.

a) legislators.

239. A group of dietitians meet with a member of the legislature to encourage her support of an important nutrition issue. This activity is termed: [NTR:533] a) lobbying. b) advocacy. c) sponsorship. d) petitioning.

a) lobbying.

265. Metabolic acidosis is characterized by what physiological changes? [NTR:559] a) low pH and negative bicarbonate (HCO3-) b) low pH and bicarbonate c) pH and bicarbonate d) pH and Bicarbonate

a) low pH and negative bicarbonate (HCO3-)

92. Successful family-based group intervention programs that target reducing overweight in school-age children include: [NTR:487] a) parent training, dietary counseling/nutrition education/physical activity counseling, and behavioral counseling. b) parent training, individual psychotherapy, dietary counseling/nutrition education. c) dietary counseling/nutrition education, physical activity counseling, behavioral counseling.

a) parent training, dietary counseling/nutrition education/physical activity counseling, and behavioral counseling.

240. A group that is organized for the purpose of raising and spending money to elect or defeat political candidates is called a/an: [NTR:534] a) political action committee. b) health advocacy organization. c) alliance. d) lobbying group.

a) political action committee.

61. Inadequate weight gain and a low birth weight infant are concerns among: [NTR:456] a) pregnant adolescents. b) pregnant women over the age of 35. c) overweight women who become pregnant. d) women with a history of gestational diabetes.

a) pregnant adolescents.

244. A community dietitian develops an educational and awareness program to teach employees of a local industry to reduce the sodium in their diets. This is an example of: [NTR:538] a) primary prevention. b) secondary prevention. c) tertiary prevention. d) workplace rehabilitation.

a) primary prevention.

108. A study that looks back in time to reconstruct exposures and the resulting health outcomes for a group of people free of the disease or condition being studied is termed a: [NTR:502] a) retrospective cohort study. b) correlational study. c) prospective cohort study. d) prevalence study.

a) retrospective cohort study.

3. The following micronutrients should be supplemented in a patient with history of alcoholism: [NTR:398] a) thiamin and folic acid. b) folic acid and magnesium. c) calcium and thiamin. d) calcium and magnesium.

a) thiamin and folic acid.

120. To avoid contracting Listeria monocytogenes, pregnant women should not consume [NTR: 514] a) unpasteurized milk and cheese b) any processed meats c) large ocean fish d) unwashed fruits and vegetables

a) unpasteurized milk and cheese

the proper portion size of a dessert item is 1/3 cup. which scoop should you use a. #12 b. #8 c. #16 d. #10

a. #12 (8ounces=1 cup/3=2.667, 32/12=2.667)

processed chopped lettuce costs $0.69/lb. whole head lettuce costs $16.49/50lb case. if the head lettuce can be processed in two hours at $5.25/hour for labor, what is the relative value of processed and head lettuce if the yield is 75% a. $.69 processed; $.72 head lettuce b. $.69 processed; $.89 head lettuce c. $.72 processed; $.69 head lettuce d. $.72 processed; $.89 head lettuce

a. $.69 processed;$.72 head lettuce

to sanitize, immerse dishes in a chemical solution for __ minutes a. 1 b. 2 c. 3 d. 4

a. 1

a recipe requires 1 1/2 pints of cleaned, sliced strawberries. the yield for strawberries is 91%. one cup of clean, sliced strawberries weighs 7 ounces. at a minimum, how man pounds of strawberries need to be ordered a. 1 1/2 b. 1 c. 1 1/4 d. 2

a. 1 1/2 (2 cups in a pint so 14 oz = 1 pint/.91% yield = 15.3847 oz x 1.5 pints = 23.0771 oz/16 oz = 1.44 pounds)

what would be the next number on this list with a moving average of 5 1400, 1200, 1300, 1250, 1350, 1400, 1425 a. 1345 b. 1275 c. 1434 d. 1405

a. 1345

a 7 year old on a ketogenic diet requires 1500 calories. which is the appropriate order a. 145g fat, 25g protein, 25g carbohydrate b. 95g fat, 40g protein, 25g carbohydrate c. 102 g fat, 56g protein, 12g carbohydrate d. 75g fat, 32g protein, 43g carbohydrate

a. 145g fat, 25g protein, 25g carbohydrate (1305kcal fat, 100kcal protein, 100kcal CHO = 1505kcals total, 86-90% of kcals from fat = 1294-1354kcals fat; usually a ratio of 4g fat for every 1g PRO/CHO, energy calculated at 80-90% of normal EEN)

A foodservice establishment requires thirteen 8-hour positions, four 4-hour positions, and one 3-hour position to meet daily demands. How many FTEs does the foodservice establishment maintain? [MGT:341] a. 15.38 b. 13 c. 10.38 d. 18

a. 15.38 (13x8 + 4x4 + 3x1 = 123/8 = 15.375)

if implementing a program costs $3 for each patient and the return is $6 per patient, the cost/benefit ratio is a. 1:2 b. 2:1 c. 1:1 d. 3:1

a. 1:2

Mr. Jones has heart failure but no CAD or increased CHD risk. his nutritional prescription should be a. 2 gram sodium b. 2 gram sodium, low potassium c. low cholesterol, low sodium d. low cholesterol, low saturated fat, low sodium

a. 2 gram sodium (diet is low sodium, such as DASH diet and fluid restriction if needed; evaluate thiamin status due to loss with loop diuretics)

in the dietary guidelines, what represents the fat included a. 20-35% calories, <10% saturated fat b. 30% calories, <6% saturated fat c. 35% calories, <15% saturated fat d. 35% calories, <10% fat

a. 20-35% calories, <10% saturated fat

children and adolescents need what level of physical activity a. 60 minutes every day b. 30 minutes every day c. 60 minutes at least three days each week d. 90 minutes at least three days each week

a. 60 minutes every day

using the above formula, determine the protein intake per day in grams a. 78 b. 80 c. 84 d. 86

a. 78 (500x1.560=780x10%)

a female, 5'7" tall, weights 115 pounds. six months ago she weighed 130 pounds. what is her percent of ideal body weight a. 85% b. 83% c. 81% d. 87%

a. 85% (IBW = 7x5+100, 135lb; 115/135=85%)

a TPN solution is to be delivered at 50ml per hour at 3/4 strength. what is the appropriate order a. 900 ml formula, 300 ml water b. 600 ml formula, 600 ml water c. 800 ml formula, 400 m l water d. 300 ml formula, 900 ml water

a. 900 ml formula, 300 ml water (50ml x 24hrs x 75% = 900ml formula)

which patient has marasmus patient A patient B patient C male Female Male serum albumin 3.5 2.6 4.0 transferrin 275 125 450 triceps skinfold 3 16.8 17 MAMC 15 24.1 28.1 TLC 1000 3500 a. A b. B c. C d. A, B

a. A

48. Centers for Disease Control and Prevention growth charts are appropriate for [NTR:443] a) American infants. b) children with cerebral palsy. c) children with Down syndrome. d) premature infants weighing less than 1500 g at birth.

a. American infants

who is most at risk from protein calorie malnutrition a. a cancer patient on chemotherapy b. a patient with gastric ulcers c. a patient with Crohn's disease d. a patient with megaloblastic anemia

a. a cancer patient on chemotherapy

the line type of organizational chart shows a. a direct flow of authority from top to bottom b. authority flows from side to side c. all advisory positions d. informal relationships

a. a direct flow of authority from top to bottom

quality assurance is a. a measurement of the quality of care provided to patients b. food service oriented c. control of the distribution and service of food d. control of the budget

a. a measurement of the quality of care provided to patients

a written record of items ordered, with quantities and specifications is a. a purchase requisition b. a standing order c. an invoice d. a receiving record

a. a purchase requisition

what is not a likely cause of failure to thrive in a child a. a vegetarian diet b. a chronic illness c. parental food restrictions due to them being overly concerned with obesity d. lack of fiber in the diet

a. a vegetarian diet

which formula is high in calories and low in protein a. aminaid b. osmolite c. sustacal d. jevity

a. aminaid

which would be a sentinel event a. an NPO patient receiving a regular tray b. a low sodium patient receiving a regular tray c. a weight reduction patient receiving a regular tray d. a high potassium patient receiving a regular tray

a. an NPO patient receiving a regular tray

the family and medical leave act guarantees a. an equivalent position with equal benefits and pay status b. the same position you held before you left c. the first open position after you return d. a similar position in a different division

a. an equivalent position with equal benefits and pay status

lactic acid is a result of a. anaerobic glycolysis b. aerobic glycolysis c. TCA cycle d. Cori cycle

a. anaerobic glycolysis

formularies are descriptions of a. approved treatment regimens b. pharmacy drug specifications c. capital budget items d. semi-variable costs

a. approved treatment regimens

the final step in a union dispute is a. arbitration b. mediation c. injunction d. check-off

a. arbitration

what procedure is binding and cannot be changed a. arbitration b. medication c. an injunction d. collective bargaining

a. arbitration

which of the following is a hearing between labor and management during an impasse to aid in settling disputes a. arbitration b. mediation c. injunction d. collective bargaining

a. arbitration (hearing to dissolve a dispute during an impasse; decision is usually binding; final step in bargaining)

the first function of the public health nutritionist is a. assess the needs of the community b. counsel patients with special dietary needs c. write pamphlets for educational purposes d. expand the role of Dietitians

a. assess the needs of the community

the dietitian finds that the ice cream freezer is not working and everything inside has melted. she decides to throw it all out and substitute another item for dessert. she is demonstrating what type of leadership a. autocratic b. bureaucratic c. democratic d. participatory

a. autocratic

which of the following foods are safe a. beefstew at 160 b. potato salad at 75 c. tuna casserole at 100 d. egg salad at 80

a. beef stew at 160

a patient on warfarin should avoid a. black cohosh b. thiamin supplements c. simple carbohydrates d. B vitamin supplements

a. black cohosh

gastric irritants include a. black pepper, chili powder, caffeine, alcohol, cocoa b. seeds, nuts, chili powder, black pepper, alcohol c. caffeine, black pepper, nuts, alcohol d. orange juice, black pepper, chili powder, alcohol

a. black pepper, chili powder, caffeine, alcohol, cocoa

creatine supplements may be beneficial for the a. body builder b. marathon runner c. soccer player d. hockey player

a. body builder

a Billroth II procedure is most likely to affect a patient's absorption of a. calcium and iron b. vitamins A and D c. folic acid and B12 d. potassium and sodium

a. calcium and iron (Ca and Fe absorption are adversely affected because the duodenum is bypassed; may also have B12 and folate deficiency; need small frequent dry feedings with fluid before/after meals, restrict hypertonic concentrated sweets; lactose may not be tolerated)

a job description a. can be used for employee appraisal b. studies and analyzes all aspects of the job c. lists what steps to do and how to do them d. lists time limits on all aspects of the job

a. can be used for employee appraisal

research suggests that in order to guard against cancer you should ingest a. carotene and vitamin C b. vitamins C and B12 c. protein and vitamin A d. vitamins E and K

a. carotene and vitamin C

the kitchen layout below is characteristic of which foodservice system deck ovens/range/tilting skillet/refrigerator/freezer/preparation counter/bulk hot carts/bulk cold carts a. commissary b. conventional c. ready-prepared d. assembly-serve

a. commissary (key is the "bulk" since commissary kitchens prepare foods in bulk or individual portions)

cost effectiveness analysis is used to a. compare two or more alternatives to achieve the same objective b. evaluate direct, indirect benefits c. place a dollar amount on the outcome of the procedure d. calculate a profit margin

a. compare two or more alternatives to achieve the same objective

when using anthropometric measurements, the most important factor is a. competence of the practitioner and calibration b. number of participants done at one sitting c. that the patient be sitting d. that they be done early in the morning

a. competence of the practitioner and calibration

a strict Muslin would not eat a. congealed fruit salad b. egg salad c. tuna salad d. citrus fruit salad

a. congealed fruit salad

PERT is a tool that will help you a. control large projects and calculate the time needed to get the job done b. assign workers to various tasks c. determine productivity levels d. determine the average check

a. control large projects and calculate the time needed to get the job done

when preparing a budget, which management function is involved a. controlling b. organizing c. evaluating d. projecting

a. controlling (purpose of a budget is to give a manager a basis for control)

which oven is most energy efficient a. convection oven b. rotary oven c. deck oven d. microwave oven

a. convection oven

which system is not labor efficient a. conventional b. vending c. cafeteria d. assembly serve

a. conventional

a cholesterol level of 240mgs needs treatment first with a. diet b. diet and drugs c. drugs d. exercise, diet and drugs

a. diet

after a gastrectomy, the diet order should indicate a decrease in a. disaccharides b. fats c. protein d. calories

a. disaccharides (to prevent dumping syndrome and delay gastric emptying)

infants born to mothers with diabetes may be a. larger than normal, and be hypoglycemic shortly after birth b. smaller than normal and be hyperglycemic shortly after birth c. larger than normal and be hyperglycemic shortly after birth d. smaller than normal and be hypoglycemic shortly after birth

a. larger than normal and be hypoglycemic shortly after birth

according to HACCP, which would be the recommendation when preparing spaghetti and sauce a. divide the prepared sauce into several small pans before chilling b. rinse spaghetti in cold water before mixing with sauce c. cook the meat thoroughly before mixing with sauce d. stir occasionally throughout the holding time

a. divide the prepared sauce into several small pans before chilling (HACCP - identify preventative approaches to quality control, identify potential dangers/food hazards, areas where a loss of control would result in an unacceptable safety risk)

how do you determine the average check in the cafeteria a. divide total sales by the number of customers b. divide entrée sales by the number of customers c. divide the number of customers by total sales d. divide the total sales by the cost of an average priced meal

a. divide total sales by the number of customers

which of the following would lower serum cholesterol levels a. dried beans b. green beans c. yellow beans d. summer squash

a. dried beans

a patient has a rare urea disease where the body is unable to synthesize urea. what might be the result a. elevated ammonia levels b. elevated BUN c. elevated glucagon levels d. compromised visceral protein levels

a. elevated ammonia levels

an elderly obese patient has been diagnosed with type 2 diabetes. which of the following is an appropriate nutrition diagnosis a. elevated serum triglycerides b. high HDL cholesterol c. low blood pressure d. low serum insulin

a. elevated serum triglycerides

a facility with a labor turnover rate of 50% also experiences numerous equipment malfunctions. both problems might be resolved through improved a. employee training b. repair standards c. equipment quality d. ergonomics

a. employee training

a dietitian who wishes to delegate work to a subordinate should first a. evaluate employee skills and task difficulty b. give the employee the authority to make decisions c. give the employee the information needed to complete the task d. ensure that the employee has the necessary materials and funds

a. evaluate employee skills and task difficulty

why may a patient with ascites be dehydrated a. fluid has moved to interstitial spaces b. fluid has moved to intravascular spaces c. fluid has moved to vascular spaces d. fluid has moved to the lungs

a. fluid has moved to interstitial spaces (interstitial - space between structures or objects i.e. between cells etc.)

to restore hydration after continuous endurance activity of 2 hours, consume a. fluid, carbohydrate, sodium b. non-CHO electrolyte solutions c. fluids d. fluids and sodium

a. fluid, carbohydrate, sodium

a dietary deficiency of which of the following causes a macrocytic anemia resembling pernicious anemia a. folacin b. pyridoxine c. vitamin B6 d. vitamin C

a. folacin

which is a quick diet assessment tool for a large group a. food frequency lists b. diet history forms c. weighing food samples and recording intake d. food records

a. food frequency lists

phytochemicals, which may influence the development of tumors, are found in a. fruits, vegetables, grains b. lean poultry and fish c. low fat dairy products d. polyunsaturated fats

a. fruits, vegetables, grains

before adding new items to the menu a. give samples to a few patients and ask for their comments b. ask patients in writing if they would like the new items c. have your managers decide which ones to offer d. ask for employee comments on new suggestions

a. give samples to a few patients and ask for their comments

as a public health Nutritionist, you notice an increase in the percentage of obese Hispanic females in the community. what is the best way to educate them a. have a cooking class that they can participate in the cooking and then eat the food they prepared b. hold classes that teach low fat cooking c. develop posters with low fat cooking tips d. develop low fat recipes and instructional materials that they can follow at home

a. have a cooking class that they can participate in the cooking and then eat the food they prepared

nutritional therapy for hepatitis includes a. high protein, high carbohydrate, moderate fat, high calories b. high protein, low carbohydrate, low fat, moderate calories c. low protein, high carbohydrate, low fat, high calories d. low protein, high carbohydrate, low fat, low calories

a. high protein, high carbohydrate moderate fat, high calories (50-55% CHO to spare protein and replenish liver glycogen, 1-1.2g/kg protein, moderate to liberal fat if tolerated - sometimes have steatorrhea, small frequent feedings, multivitamin with B complex, C, K, zinc, increase fluid to prevent dehydration)

you recently interviewed a top candidate for an administrative position, and are about to offer him the job. you then learn that he is planning on adopting a child. what should you do a. hire him b. confront him about the possibility c. do not hire him d. wait two weeks to see if he does adopt the child

a. hire him

a patient with nephrosis will likely exhibit a. hypercholesterolemia, albuminemia, edema b. hypertension, hypocholesterolemia c. hypotension, azotemia d. malnutrition, hyperalbuminemia

a. hypercholesterolemia, albuminemia, edema

a concern with parenteral feeding is a. hypokalemia b. hypercalcemia c. hyponatremia d. hypernatremia

a. hypokalemia

a patient complains of abdominal distension, mouth lesions, and a 10% weight loss over the past 6 months. the nutrition diagnosis would be a. impaired nutrient utilization b. inadequate vitamin intake c. involuntary weight loss d. altered GI function

a. impaired nutrient utilization

an employee evaluation should be given a. in a quiet place with privacy b. in the diet office c. at the end of the day d. at the beginning of the day

a. in a quiet place with privacy

employees of a congregate feeding program serving an orthodox jewish community are not allowed to bring food items into the facility. this is related to jewish dietary laws that state a. kosher foods should not be allowed to mix with non kosher foods b. consumption of fruit and vegetables are restricted to the noon meal c. only unleavened bread or matzo products are allowed in a kosher kitchen d. no pork or shellfish are allowed in a kosher kitchen

a. kosher foods should not be allowed to mix with non kosher foods

which menu item generates the highest profit menu item/units sold/selling $/raw food $/prime cost lasagna 1500 $3.00 $1.35 $2.00 veg. chili 100 $1.25 $0.57 $1.00 hamburger 400 $2.00 $0.90 $1.50 enchilada 100 $3.00 $1.14 $1.75 a. lasagna b. vegetarian chili c. hamburger d. enchilada

a. lasagna (prime cost = raw food cost + labor cost)

which of the following weight gains puts pregnant women at risk a. less than 4 pounds/month in the last half of pregnancy b. less than 5 pounds/month in the last half of pregnancy c. less than 6 pounds/month in the last half of pregnancy d. more than 2 pounds/month in the last half of pregnancy

a. less than 4 pounds/month in the last half of pregnancy (2-4lb weight gain in first trimester and then 1lb/week in last two trimesters)

a patient with diabetes has not eaten for more than three days. you should a. liberalize the diet b. provide multivitamin/mineral supplements c. provide increased fluids d. stay with her plan and encourage small frequent feedings

a. liberalize the diet

a customer who has eaten lunch in the cafeteria becomes ill one hour after eating a chicken sandwich. what should the dietitian do first a. listen carefully, take the complaint seriously and conduct an investigation b. accept responsibility and offer a coupon for a complimentary meal c. report the complaint immediately to the risk management department d. ask where the person ate the night before and deny any responsibility

a. listen carefully, take the complaint seriously and conduct an investigation

a work schedule a. lists both tasks and time b. lists only tasks to be performed c. is used for employee appraisal d. studies all aspects of the job

a. lists both tasks and time

what would the lab values above have likely reflected if the patient had been nauseated and had averaged only 400 calories from primarily starchy foods over the weekend a. little or no rise in potassium, BUN/creatinine ratio <10 b. increased phosphorus, BUN/creatinine ratio >15 c. typical rise in potassium, BUN/creatinine ratio>15 d. metabolic alkalosis, BUN/creatinine ratio<10

a. little or no rise in potassium, BUN/creatinine ratio <10

what is the diet for hypertension a. low sodium b. low sodium, high potassium c. low sodium, high calcium d. low sodium, high potassium, high calcium

a. low sodium

compared to whole milk, breast milk is a. lower in protein, higher in carbohydrate and fat b. higher in protein and fat, lower in carbohydrate c. lower in protein, carbohydrate and fat d. has the same nutrient composition

a. lower in protein, higher in carbohydrate and fat

an appropriate menu for a Kosher meal is a. macaroni and cheese, green beans b. roast beef, potato, vegetable, cream soup c. veal parmesan, noodles, green beans d. crabmeat salad, lettuce milk

a. macaroni and cheese, green beans

a female who eats a wide variety of foods and takes supplemental iron and birth control pills may be a. marginally deficient in folic acid b. low in calcium c. low in vitamin A d. low in zinc

a. marginally deficient in folic acid

to implement an efficient staffing plan and adhere to budget constraints, the dietitian can ensure labor costs are controlled by a. master schedules b. job descriptions c. compensation changes d. cross training

a. master schedules

when meat is irradiated, the best flavor comes from a. meat that has been frozen b. fresh meat c. aged meat d. young meat

a. meat that has been frozen

a tube fed patient develops diarrhea. what is the most likely cause a. medications b. the osmolality is too low c. the strength is too weak d. it is being given at an improper rate

a. medications

a patient with low hemoglobin and low MCV might have a. microcytic anemia b. macrocytic anemia c. megaloblastic anemia d. normochromic anemia

a. microcytic anemia

within 2-3 hours of continuous exercise at 60-80% of maximum: a. muscle glycogen is depleted b. muscle glycogen is the main energy source c. amino acids begin to contribute a source of energy d. fatty acids are the favored substrates

a. muscle glycogen is depleted

a substitute is legally differentiated from another food by a. nutrient content b. calorie content c. fat content d. color

a. nutrient content

what should most be encouraged in a Headstart program a. nutrition activities related to food b. participation for at least one year c. class instructions d. healthy snacks

a. nutrition activities related to food

84. During which step in the Nutrition Care Process is it important to continually reevaluate the patient's/client's status compared to specified criteria? [NTR:479] a) Nutrition Assessment b) Nutrition Diagnosis c) Nutrition Intervention d) Nutrition Monitoring and Evaluation

a. nutrition assessment

hospitals use formularies to a. obtain volume purchasing discounts b. limit the use of certain products c. develop process charts d. organize departments in a concentric pattern

a. obtain volume purchasing discounts (formularies: descriptions of approved products than an organization would like used; volume discounts can be obtained)

which is the least common symptom in the elderly a. osteomalacia b. osteoporosis c. obesity d. hypertension

a. osteomalacia

a patient with short bowel syndrome will likely receive as initial nutrition therapy a. parenteral nutrition b. elemental formula through an enteral feeding c. predigested nutrients through a gastrostomy tube d. lactose free, low osmolar, jejunal feeding

a. parenteral nutrition

a child has been identified as being at high risk for food allergies. what should be withheld for the first few years a. peanut butter and milk b. eggs and strawberries c. oats and fish d. citrus fruits and chocolate

a. peanut butter and milk

what method is used when you log in and out movement from the storeroom a. perpetual inventory b. mini-max inventory c. physical inventory d. stage three inventory

a. perpetual inventory

a patient admitted to the ICU is diagnosed with HIV and progressive infection. which nutrients would be helpful a. phosphorus and zinc b. sodium and potassium c. sodium and chromium d. potassium and magnesium

a. phosphorus and zinc

excess food soil in a dish machine wash tank will result in foam that interferes with satisfactory dishwashing. the most appropriate corrective action is to a. pre-rinse dishes before washing b. increase the amount of detergent c. increase water pressure in the wash tank d. drain the wash tank during meals and refill

a. pre-rinse dishes before washing

what is the goal of the dietary guidelines a. prevention of chronic disease b. provide standards for food stamp allocation c. provide standards for WIC d. provide standards for EFNEP

a. prevention of chronic disease

which is restricted in a diet for hepatic encephalopathy a. protein, sodium, fluid b. protein, fluid, calories c. calories, protein, fat d. sodium, fat, calories

a. protein, sodium, fluid (encephalopathy is damage to the brain often resulting in altered mental status, treated with a low protein diet, low sodium due to often accompanying ascites and restricted fluid due to ascites as well)

sugar and salt act as preservatives because they a. provide unfavorable osmotic pressure b. prevent microbial contact c. disguise flavors d. reduce the alkalinity

a. provide unfavorable osmotic pressure

which of the following about nutrition screening in a hospital is true a. provides direction for patient care b. cost-effective for all patients c. it has universal criteria for all age groups d. it should be performed only when ordered

a. provides direction for patient care

high doses of which vitamin will interfere with levodopa a. pyridoxine b. cyanocobalamin c. thiamin d. riboflavin

a. pyridoxine

which of the following diets is appropriate for reactive hypoglycemia a. reduce intake of concentrated carbohydrates, 5-6 small meals b. increased complex carbohydrate, low fat, high protein c. decreased complex carbohydrate, low fat d. high protein, high fat, high carbohydrate

a. reduce intake of concentrated carbohydrates, 5-6 small meals

what does a fecal fat of 12 grams indicate a. severe fat malabsorption b. normal fat absorption c. high normal fat absorption d. nothing

a. severe fat malabsorption

which foods are most likely to cause an allergic reaction a. shellfish, peanuts, wheat b. wheat, chocolate, rice c. citrus fruits, milk, corn d. chocolate, milk, corn

a. shellfish, peanuts, wheat

chemically defined formulas are used for a. short bowel syndrome b. post chemotherapy treatment c. hypermetabolism d. galactosemia

a. short bowel syndrome

enteral feeding products that contain protein in the form of small peptides and amino acids are recommended for patients who have a. short-bowel syndrome b. undergone gastrectomy c. undergone colectomy d. undergone jejunostomy

a. short-bowel syndrome (significant resection of intestine - can impair digestion and absorption)

a hospital cafeteria that currently serves 500 meals at noon must increase service to 800 without adding food production staff. the best action would be a. simplify the menu b. redesign the kitchen c. rearrange employee work schedules d. replace current equipment with fast-performance equipment

a. simplify the menu

a child has chronic non-specific diarrhea without malabsorption. an increased intake of which of the following might be the cause a. sorbitol b. fat c. protein d. fruit

a. sorbitol

what is an example of a program that provides food to the needy a. the Food Bank b. the Food Stamp Program c. the Child Care Program d. WIC

a. the Food Bank (food donation/distribution programs provide foods to help meet nutritional needs of children and adults)

large volume purchasing, centralized production, service areas in separate facilities are features of a. the commissary system b. the ready-prepared system c. the assemble serve system d. the conventional system

a. the commissary system

which law sets the minimum wage a. the fair labor standards act b. the national labor relations act c. the equal opportunity act d. the labor management act

a. the fair labor standards act (also called minimum wage or wage hour law; sets minimum wage; donated time must be compensable; child labor laws; prohibits discrimination on basis of sex)

if dishes are not cleaned properly in the dish machine, the first thing to check is a. the temperature of the various cycles b. the number of items being serviced in a short time c. the length of time between a meal and dishwashing d. the number of employees assigned to the process

a. the temperature of the various cycles (lower temperatures lead to greasy dishes, pre-rinse temp liquefies fat and removes food before hot water coagulates protein)

which type of nutrition support ensures the maximum function of the GI surface area a. tube feeding with an isotonic formula b. tube feeding with BCAA c. peripheral parenteral nutrition d. parenteral nutrition

a. tube feeding with an isotonic formula

when measuring a child's somatic protein mass, in addition to the BMI, use a. upper arm muscle area b. triceps skinfold c. mid-arm muscle circumference d. knee to height ratio

a. upper arm muscle area

the transfer of heat when cooking with an induction burner is the a. use of electrical magnetic fields to energize the metal cooking surface b. movement of heat through direct contact c. distribution of heat by movement of liquid or vapor d. generation of heat by wave action within an object

a. use of electrical magnetic fields to energize the metal cooking surface

costs which are proportionately responsive to changes in volume are a. variable b. sunk c. hidden d. fixed

a. variable

a patient with rough dry scaly skin and Bitot's spots has signs of a deficiency in: a. vitamin A b. vitamin E c. thiamin d. niacin

a. vitamin A (Bitot's spots are build of keratin in conjunctiva)

which supplements should you give to a breast-fed baby a. vitamin D and fluoride b. fluoride c. vitamin D d. vitamins A and C

a. vitamin D and fluoride

dishes are greasy when unloaded from the dish machine. what should you check first a. wash temperature b. rinse temperature c. amount of detergent d. amount of rinse agent

a. wash temperature

A muffin has long tunnels from top to bottom. Next time you make this recipe you should: a. add more baking soda b. avoid over-mixing the batter c. add less butter d. cook it for a shorter time

avoid over-mixing the batter

298. Which of the following is an example of a positive correlation? [NTR:592] a) "The amount of weight gained by pregnant women during the first trimester has been found to decrease as the number of episodes of nausea and vomiting increase." b) "The incidence of type 2 diabetes mellitus increases as central adiposity increases." c) "Consuming calories in excess of those needed results in an increase in generalized adiposity." d) "Blood pressure in certain genetic groups appears to decrease as the consumption of dietary sodium is reduced."

b) "The incidence of type 2 diabetes mellitus increases as central adiposity increases."

42. A calorie deficit of 500 kcals below estimated energy needs should result in the loss of: [NTR:437] a) 1/2 pound per week. b) 1 pound per week. c) 2 pounds per week. d) 2-1/2 pounds per week.

b) 1 pound per week.

29. Which of the following foods contains approximately 15 grams of carbohydrate? [NTR:424] a) 1 English muffin b) 1/2 cup mashed potatoes c) 4 cups popped popcorn d) 1 large banana

b) 1/2 cup mashed potatoes

60. The recommended weight gain during pregnancy for a woman with BMI between 19.8 and 26 is [NTR:455] a) 15-20 pounds b) 25-35 pounds c) 28-40 pounds d) 35-45 pounds

b) 25-35 pounds

267. The body mass index range for Class I obesity is defined as [NTR:561] a) 25-29.9. b) 30-34.9. c) 35-39.9. d) ≥40.

b) 30-34.9.

266. An isotonic enteral formula would have an osmolality of: [NTR:560] a) 0 mOsm/kg. b) 375 mOsm/kg. c) 550 mOsm/kg. d) 700 mOsm/kg.

b) 375 mOsm/kg.

19. All breast fed infants should receive how many IU's of vitamin D per day beginning in the firsts 2 months of life? [414] a) 200 b) 400 c) 600 d) 800

b) 400

362. What would be the approximate protein requirement, in grams, for a male, non-athlete weighing 87 kg? [NTR:657] a) 37 b) 70 c) 239 d) 174

b) 70

361. What is the upper limit of fat, in grams, that should be consumed by a healthy person requiring 2500 kcals per day? [NTR:656] a) 875 b) 97 c) 218 d) 71

b) 97

327. What is the requirement to label a food "fat free"? [NTR:621] a) 0 grams of saturated fat per serving b) <0.5 grams total fat per serving c) <3 grams total fat per serving d) 100% less fat than in the comparable product

b) <0.5 grams total fat per serving

363. Which grouping of words contains all the major energy-yielding substances? [NTR:658] a) Carbohydrates, triglycerides, protein b) Alcohol, carbohydrates, triglycerides, protein c) Vitamins, minerals, carbohydrates, triglycerides, protein d) Carbohydrates, triglycerides, protein, vitamins, minerals, water

b) Alcohol, carbohydrates, triglycerides, protein

344. Which one of these individuals is most likely to present with Wernicke-Korsakoff syndrome? [NTR:639] a) A premature infant b) An alcoholic man c) A postmenopausal woman d) A disabled child

b) An alcoholic man

284. Which statement correctly describes the relationship between calcium and the other substance indicated? [NTR:578] a) Calcium may interfere with zinc absorption. b) Calcium may decrease iron absorption. c) Calcium may increase magnesium absorption. d) Calcium intake may decrease phosphorus absorption.

b) Calcium may decrease iron absorption.

33. Which of the following eating pattern changes does research show most positively contributes to a decrease in energy consumption among children? [NTR:428] a) Participating in family-style meals at home b) Children serving themselves on individual plates at mealtimes c) Parents or caregivers serving children on individual plates d) Providing scheduled meals and snacks in the home

b) Children serving themselves on individual plates at mealtimes

97. Which of the following oils is an adequate source of medium chain triglycerides (MCT)? [NTR:491] a) Corn oil b) Coconut oil c) Safflower oil d) Soybean oil

b) Coconut oil

112. While conducting a community needs assessment, after defining the nutritional problem, which of the following is your next step? [NTR:506] a) Identify community groups and their leaders. b) Determine the purpose of the needs assessment. c) Collect information on health and nutrition resources available in the community. d) Search the Census Bureau records to determine the percentage of single-family households in the community with children in the 2- to 5-year-old age range.

b) Determine the purpose of the needs assessment.

302. Which of the following statements about lipids is true? [NTR:596] a) Steroid hormones are made from one or more fat-soluble vitamins. b) Digestion of triglycerides requires bile and lipases. c) Triglyceride digestion is completed in the large intestine. d) Insulin stimulates lipolysis and the mobilization of triglycerides.

b) Digestion of triglycerides requires bile and lipases.

The Food and Drug Administration (FDA) provides a strict definition for functional foods. [NTR:481] a) TRUE b) FALSE

b) FALSE

246. The largest hunger-relief organization in the United States is: [NTR:540] a) UNICEF. b) Feeding America. c) SHARE. d) The Hunger Project.

b) Feeding America.

37. Which of the following conditions best describes when an elderly person does not have enough to eat? [NTR:432] a) Food insecurity b) Food insufficiency c) Food insecurity with hunger d) Food security with hunger

b) Food insufficiency

296. Which of the following foods would you recommend decreased intake of if your patient has a malodorous ileostomy? [NTR:590] a) Yogurt b) Garlic c) Banana d) Peanut butter

b) Garlic

308. Which of the following gastrointestinal hormones has the effect of increasing the consumption of food? [NTR:602] a) Cholecystokinin b) Ghrelin c) Enterostatin d) Peripheral hormone peptide YY

b) Ghrelin

326. Which of the following does not provide a significant source of vitamin D? [NTR:620] a) Egg yolk b) Kale c) Sardines d) Fortified milk

b) Kale

17. Initially, a dietetics professional undertaking Nutrition Diagnosis as part of the Nutrition Care Process should identify and _____ a specific nutrition problem that he or she is responsible for treating independently. [NTR:412] a) Manage b) Label c) Measure c) coordinate

b) Label

283. Where in the body is vitamin D hydroxylated to 25-hydroxyvitamin D? [NTR:577] a) Kidney b) Liver c) Skin d) Blood

b) Liver

258. Muslims avoid eating _____. [NTR:552] a) Yogurt b) Pork c) Legumes d) goat

b) Pork

110. The nutritionist employed by the local county wants her nutrition objectives for the next 5-7 years to reflect the goals of Healthy People 2020. Which of her goals listed below most closely reflects the goals of Healthy People 2020? [NTR:504] a) Reduce among residents the severity of chronic diseases such as diabetes and heart disease. b) Promote, across every stage of life, healthy behaviors that positively impact the nutritional status of residents. c) Improve the nutrition of residents by eliminating disparities in treatment plans. d) Eliminate social and physical environments that threaten the nutrition of residents.

b) Promote, across every stage of life, healthy behaviors that positively impact the nutritional status of residents.

82. In the Nutrition Care Process, a Nutrition Diagnosis differs from a medical diagnosis in that it can be _____. [NTR:477] a) Treated b) Resolved c) Generic d) Monitored

b) Resolved

78. When using the Nutrition Care Process, what three things do dietetics practitioners do in completing Nutrition Monitoring and Evaluation? [NTR:473] a) Prioritize nutrition diagnoses, consult ADA's Evidence-based Nutrition Practice Guidelines, and determine patient-focused expected outcomes. b) Scrutinize, measure, and assess the changes in nutrition care indicators. c) Confer with patient/client/caregivers, define a nutrition plan and strategies, and determine time and frequency of care. d) Identify resources needed, communicate the nutrition care plan, and implement the plan.

b) Scrutinize, measure, and assess the changes in nutrition care indicators.

236. Which of the following is a contraindication to enteral nutrition support in an elderly patient? [NTR:530] a) Dysphagia b) Severe acute pancreatitis c) Advanced osteoporosis c) Oncologic disease

b) Severe acute pancreatitis

103. Chin has written the following objective for her community nutrition program — "Women who are at least 60 years of age will perform at least 30 minutes of weight-bearing exercise three or more times per week." Which of the required components is missing from Chin's objective? [NTR:497] a) Target population b) Time frame c) Action d) Measure of success

b) Time frame

10. Which of the following are the correct CDC growth charts to use in a child care center for assessing the growth of a 6 year-old boy? [NTR:405] a) Weight-for-age and Length-for-age charts b) Weight-for-age and Stature-for-age charts c) Stature-for-age and Weight-for-stature charts d) Length-for-age and Weight-for-stature charts

b) Weight-for-age and Stature-for-age charts

89. Tertiary prevention efforts in childhood weight management target: [NTR:484] a) children living within low-income urban or rural populations. b) children who are already overweight. c) children who typically live in areas classified as "food deserts". d) Children who participate in childcare environments or after-school programs.

b) children who are already overweight.

102. Rosa wants to plan a community nutrition program to encourage children ages 11-15 to consume healthier afterschool snacks. She has reviewed the results of the community needs assessment, defined program goals and objectives, and developed a program plan. Next, she should [NTR:496] a) identify funding sources. b) define the management system. c) implement the program. d) seek support from the parents and schools of intended participants.

b) define the management system.

41. An appropriate client to recommend the use of meal replacements for weight loss to is one who: [NTR:436] a) has no more than 20 pounds to lose. b) has difficulty with portion control. c) habitually eats late at night. d) does not engage in physical activity.

b) has difficulty with portion control.

272. _________ is an essential amino acid [NTR:566] a) alanine b) histidine c) proline d) tyrosine

b) histidine

304. Glycogenolysis is the breakdown of glycogen into glucose that occurs in the cells of the __________. [NTR:598] a) stomach and small intestine b) liver and muscle c) kidney and brain d) small intestine and kidney

b) liver and muscle

294. Vision problems in older adults may be due to dietary factors such as a/an [NTR:588] a) excess intake of sodium. b) low intake of antioxidant nutrients. c) excess intake of fat and cholesterol. d) low intake of B vitamins.

b) low intake of antioxidant nutrients

348. Excessive intake of phytates and oxalates can interfere with the absorption of [NTR:643] a) cholesterol. b) minerals. c) vitamins. d) fiber.

b) minerals.

34. What is the first step in the nutrition care process? [429] a) nutrition screening b) nutrition assessment c) nutrition diagnosis d) goal setting

b) nutrition assessment

24. The 2007 Expert Committee Recommendations for the Prevention, Assessment, Treatment, and Recommendations for Pediatric Childhood and Adolescent Obesity defines BMI greater than the 95th percentile as: [NTR:419] a) at risk for overweight. b) obese. c) Overweight d) severe childhood obesity

b) obese.

91. Studies suggest that the most serious and prevalent long-term consequences of childhood overweight is: [NTR:486] a) dyslipidemia and glucose intolerance. b) obesity and morbidity in adulthood. c) lowered self-esteem and increased depression as an adolescent. d) excessive body weight and hormonal imbalance before puberty leading to growth plate injuries.

b) obesity and morbidity in adulthood.

226. Julia is a 10-year-old fifth grader with ADHD who takes Adderall. Her BMI-for-age is less than the 5th percentile. The most appropriate nutrition intervention for her is to [NTR:520] a) provide snacks for her to eat during recess. b) offer her a high-calorie bedtime snack. c) send a lunch consisting of her favorite foods. d) have the teacher monitor her intake.

b) offer her a high-calorie bedtime snack.

what is the labor cost per meal total food purchased $32123 total cafeteria sales $23231 total labor expenses $25790 meal equivalent factor $2.15 total patient meals served 1923 a. $1.85 b. $2.03 c. $3.98 d. $4.36

b. $2.03 (23231/2.15 = 10805+ 1923=12728 25790/12728=2.0263)

how much will it cost to prepare these scrambled eggs amount to be made: 25 cost of eggs $.83 per dozen 2 ounces margarine at $2.29 per pound 2 cups milk at $2.50 per gallon a. $1.47 b. $2.35 c. $1.02 d. $1.79

b. $2.35 (0.83/12x25=1.73 + 2.29/16x2=.2863 + 2.50/16x2=.3125 = 2.3288)

the hospital budget for the first 6 months is set for an 80% occupancy rate. budget figures for this level are listed below. the second six months will have an occupancy rate of 60%. what would the food cost budget be for the second six months. food cost: $30000 labor cost: $25000 operating costs: $24000 a. $20000 b. $22500 c. $30000 d. 32000

b. $22500 (30000/.80=37500x60%=22500)

if a patients GFR is <50 ml/minute, what should their protein intake be (in g/kg) a. 1 b. .8 c. .50-.6 d. .2-.4

b. .8 (GFR < 60 = .8g pro (stage 2-3), <25 = .6g pro (stage 4-5)

how many meals were served per labor minute if 34 FTE produced 3800 meals in one 40 hour week a. 2.79 b. 0.05 c. 1.12 d. 0.25

b. 0.05 (3800 meals/34 FTEx40hr/wkx60min = 0.0466)

what is a guide for calculating water needs with tube feedings a. 1 cc water/0.5 calories b. 1 cc water/1.0 calories c. 1 cc water/2.0 calories d. 1 cc water/3.0 calories

b. 1 cc water/1.0 calories

which contains more total dietary fiber a. 1/2 cup raw spinach b. 1 oz raisin bran cereal c. 1 slice whole wheat bread d. 2T peanut butter

b. 1 oz raisin bran cereal

if a man normally consuming 2600 calories per day reduces his intake to 1500 calories per day, how much weight will he lose in one week a. 0.5kg b. 1.0kg c. 1.5kg d. 2.0kg

b. 1.0kg

how many calories are in 1.5 liters of a 25% dextrose solution a. 850 b. 1275 c. 1000 d. 1500

b. 1275

you have invested in new hardware and software that will reduce the labor hours needed to complete a task. what is the payback period hardware: $3500, software: $2400; staff instruction: $1000; labor hours saved: 6 per week at $20/hour a. 6 years b. 13 months c. 20 months d. one year

b. 13 months (3500+2400+1000=6900, 6x20=120 6900/120=57.5 weeks)

what is the lowest acceptable holding temperature while serving a. 120F b. 140F c. 160F d. 180F

b. 140F

chicken has a popularity index of 45%. there are 380 total patients, 85% of whom are eating. how many will choose chicken a. 202 b. 145 c. 196 d. 154

b. 145 (380x85%=323x45%=145.35)

previously cooked hazardous food must be reheated to an internal temperature of 165F for fifteen seconds within a. 1 hour b. 2 hours c. 4 hours d. 30 minutes

b. 2 hours

a 1500 calorie diabetic diet with 45% of the calories from carbohydrate is prescribed. one third of the carbohydrate grams is met with 1/2 cup orange juice, 1 cup milk and a. 1 slice toast b. 2 slices toast c. 2 slices toast and 1/2 cup oatmeal d. 1/2 slice toast

b. 2 slices toast

a foodservice uses hamburger patties at an average rate of 6 cases per day. lead time is 3 days. you want to have a safety supply of 2 cases. you need to order them when the supply drops to ___ cases a. 18 b. 20 c. 36 d. 45

b. 20 (6 cases x 3 days lead + 2 safety supply = 20)

in which of the following would you likely see very low albumin if it is drawn a few days after admission to an acute care facility a. anorexia nervosa b. 30% full thickness burn c. patient with newly diagnosed colon cancer d. patient with COPD

b. 30% full thickness burn

forty entrees of fish cost $2.00 per serving. total sales for the fish came to $240.00. what was the food cost percentage a. 25% b. 33% c. 50% d. 60%

b. 33% (80/240=33%)

two liters of a 5% dextrose solution provide how many calories a. 170 b. 340 c. 212 d. 195

b. 340

what temperature range is recommended for a diary refrigerator a. 28-32F b. 36-40F c. 45-50F d. 55-60F

b. 36-40F

in a hospital with 300 patients, it takes 14 minutes to prepare one meal. how many FTEs are needed per week a. 12 b. 37 c. 46 d. 10

b. 37

Employees Work hours/week dietitian 40 nurse 40 doctor 30 diet aides 20 secretary 20 clerk 10 how many FTEs are in the above operation a. 2 b. 4 c. 6 d. 8

b. 4

a newly diagnosed diabetic needs 2200 calories, 50% from carbohydrates. how many CHO choices can he have at each meal if he has 3 snacks with 2 CHO choices each a. 3 b. 4 c. 2 d. 1

b. 4

the normal range for hemoglobin A1C is a. 1-2% b. 4-6% c. 10-12% d. 35-40%

b. 4-6%

the maximum number of calories per serving in a product labeled "low calorie" is a. 20 b. 40 c. 60 d. 80

b. 40 (no more than 40 calories/serving)

how wide should an aisle be for two people with a mobile cart to move through a. 40" b. 50" c. 60" d. 72"

b. 50" (lane with >1 person or where mobile equipment passes through = 48-52")

entrée #servings food cost/serving pot roast 70 $1.50 chicken stew 30 $.75 spaghetti 40 $.60 what is the popularity index for pot roast a. 40% b. 50% c. 60% d. 70%

b. 50% (70/total number of servings)

which of the following types of menus would be most appropriate for a new acute-care facility with an expected length of stay of 2 days a. static b. 7 day cycle c. 14 day cycle d. 21 day cycle

b. 7 day cycle

how many grams of protein would you receive on a 1500 calorie diet which is 20% protein a. 70 b. 75 c. 64 d. 300

b. 75

the most appropriate temperature for a sanitizing solution is a. 50F b. 79F c. 90F d. 140F

b. 79F (1 minute in 75F water with a chemical solution)

infuse a peripheral solution at a maximum rate of how many mOsm/L a. 300 b. 800 c. 1400 d. 2000

b. 800

you are involved in planning a prospectus. this is a. measuring the size of all food service equipment b. a planning guide for a kitchen c. how to arrange the dairy box d. how to document nutrition care

b. a planning guide for a kitchen (prospectus - a document describing the major features of a proposed literary work, project, business venture etc. )

a graphic representation of the separate steps in a procedure using symbols is a. a flow diagram b. a process chart c. an operations chart d. a cross chart

b. a process chart

which would be a sentinel event a. a meat slicer left with the blade unguarded b. a wet floor near the dish machine c. an unclean can opener d. MSDS sheets not posted

b. a wet floor near the dish machine

a dietitian documents their work on time. this is an example of a. responsibility b. accountability c. courtesy d. good time management

b. accountability (n obligation or willingness to accept responsibility or to account for one's actions)

a morbidly obese patient in ICU on a ventilator has end-stage renal disease, diabetes, and sacral decubiti. the primary concern for recommending a tube feeding formula and rate should be a. increased protein b. adequate calories c. decreased carbohydrates d. increased electrolytes

b. adequate calories

an acutely ill patient is receiving propofol and tube feedings. the dietitian should a. recommend the feeding be held 2 hours before and after administration of the medication b. adjust the tube feeding to avoid overfeeding c. monitor the patients lab data for increased BUN levels d. monitor the patients lab data for decreased folate levels

b. adjust the tube feeding to avoid overfeeding

symptoms of nephrosis include a. dehydration, hypocholesterolemia b. albuminuria, hypercholesterolemia c. edema, hypocholesterolemia d. hypotension, hyperalbuminemia

b. albuminuria, hypercholesterolemia

what is the best example of delegation a. tell the employee what to do b. allow the employee to change procedures to achieve desired outcome c. require the employee to provide daily progress reports d. select an employee who is interested in management

b. allow the employee to change procedures to achieve desired outcome

if a patient has a biochemical block in the formation of urea, what will accumulate in his blood a. uric acid b. ammonia c. protein d. glucose

b. ammonia

which is considered a hazardous situation a. a gauge on the coffee urn not working b. an employee washing floors during rush hour c. an employee cleaning the grill with steel wool d. an employee leaving the freezer door ajar

b. an employee washing floors during rush hour

for the past 2 months, a man who is 6'0" tall has been nauseated and anorexic and has consumed fewer than 500cals/day, mostly from sweet foods. his albumin of 5.0g/dl most likely reflects a. adjustment to the new energy intake b. an increase in albumin due to dehydration c. increased production of albumin by the liver due to the catabolic state d. retention of albumin due to the high carbohydrate, low calorie diet

b. an increase in albumin due to dehydration

when a starved patient begins to eat, they might expect a. constipation b. an increased workload on the heart c. lethargy d. diarrhea

b. an increased workload on the heart

which type of anemia is likely to be seen in a patient with arthritis a. microcytic, hypochromic b. anemia secondary to chronic disease c. macrocytic d. megaloblastic

b. anemia secondary to chronic disease

how long should a new mother breast-feed a. at least 6 months b. at least 12 months c. until 6 months, then add cow's milk d. until 4 months, then switch to cow's milk

b. at least 12 months (up until 6 months with only breast feeding then breast feeding plus baby food starting at 5-6 months)

the diet for stomatitis is a. low fat, high carbohydrate, normal protein b. avoid spices, very hot, very cold, sour, tart foods c. high protein, high carbohydrate d .liquids, mostly carbohydrates

b. avoid spices, very hot, very cold, sour, tart foods

the report representing the financial condition of an organization as of a particular date is a a. budget b. balance sheet c. income statement d funds flow sheet

b. balance sheet (shows financial condition as of a particular date; formulas use data from financial statements, compares organization with similar ones, compares ratios with those projected or with preceding rations)

when counseling an anorexic, the best advice is to a. be real strict in their food choices b. be flexible and help them choose reasonable goals c. provide a large variety of menu items d. give them free rein with their intake

b. be flexible and help them choose reasonable goals

several patients receiving pureed foods at a long term care facility have developed salmonella infections. the most likely cause is a. pots and pans were washed and left to air dry b. blender was washed in warm soapy water c. the canned pureed meat was stored at 70 degrees d. the eggs from breakfast were held on the hot line at 165 degrees

b. blender was washed in wark soapy water

traditional diets of Asian populations in the united states are low in a. iron b. calcium c. vitamin C d. essential fatty acids

b. calcium

what is lacking in the traditional diet for Asians living in America a. iron b. calcium c. folic acid d. thiamin

b. calcium

which provides the most vitamin A for a Mexican a. tortillas and refried beans b. cantaloupe and squash c. cheese d. tortillas and milk

b. cantaloupe and squash

a dietitian should consider a client's absorption of which of the following nutrients during peritoneal dialysis and continuous renal replacement therapy before modifying his diet a. protein b. carbohydrate c. fat-soluble vitamins d. water soluble vitamins

b. carbohydrate

what would you do to decrease fat intake for a Hispanic a. change from whole milk to 2% milk b. change beans from refried to boiled c. change to low fat yogurt d. serve low fat cheese

b. change beans from refried to boiled

a child with AIDS needs a high calorie and _____ intake a. megadose vitamins b. high protein c. low fat d. low residue

b. high protein

if the chlorine solution in the dish machine is not working properly, the first action should be a. change to quaternary ammonia b. check the concentration and temperature of the solution c. use disposable dishes d. change to an iodine solution

b. check the concentration and temperature of the solution

which type of power is ineffective in motivating behavioral change and may create resistance a. referent b. coercive c. legitimate d. expert

b. coercive

what method of food service is used when one school delivers meals to another a. ready-prepared b. commissary-satellite c. assembly serve d. cook-chill

b. commissary-satellite

an infant's failure to thrive can be seen in a. their behavior b. comparison with growth charts c. weight for height d. height for age

b. comparison with growth charts

you are hired by a health club to develop a weight control program. there are at least two other similar programs in the area. to determine the potential success of this new program first a. develop promotional materials b. conduct a market analysis c. analyze the success of existing programs d. discuss the idea with other dietitians

b. conduct a market analysis

which of the following tools would be best in determining consumption of vegetables in a high school a. 24 hour dietary recall records b. conduct plate waste studies c. conduct focus groups with students d. track school cafeteria sales records

b. conduct plate waste studies

what would a vegan not eat a. coleslaw made with oil and vinegar b. congealed fruit salad c. salad with tofu dressing d. peanut butter

b. congealed fruit salad

a 75 year old female with adequate income would benefit from a. food stamps b. congregate meals c. EFNEP d. WIC

b. congregate meals

you are the food service director and have received a note saying that last night's meat delivery had been improperly handled by two employees. your first action is to a. contact the delivery driver to see how the meat was delivered b. contact the two employees responsible for handling the meat c. inspect the meat for color and odor d. have the meat cooked immediately

b. contact the two employees responsible for handling the meat

an insulin dependent diabetic living alone calls you with symptoms of nausea, vomiting and anorexia. what do you recommend a. continue with insulin and eat anything he can b. continue with insulin and drink anything he can c. discontinue the insulin until he feels better d. discontinue the insulin and eat whatever he can

b. continue with insulin and drink anything he can

the main goal for a type 2 obese diabetic with persistent hyperglycemia is a. weight loss b. control of serum glucose levels c. consumption of a low fat diet d. consumption of a low protein diet

b. control of serum glucose levels

the kitchen layout below is characteristic of which foodservice system deck ovens/range/tilting skillet/refrigerator/freezer/preparation counter/meal assembly/delivery carts a. commissary b. conventional c. ready prepared d. assembly-serve

b. conventional

the process of comparing a program's benefits to net savings is called a. cost effectiveness analysis b. cost benefit analysis c. formative evaluation d. program evaluation

b. cost benefit analysis

when cleaning ______, use chlorine or quaternary ammonia a. laundry b. countertops c. oven d. floors

b. countertops

which test measures somatic protein a. prealbumin, albumin b. creatinine clearance, serum creatinine c. serum transferrin d. prealbumin, albumin, transferrin

b. creatinine clearance, serum creatinine

which of the following is an outcome indicator a. freezer temperatures are recorded daily b. customers are satisfied with the meals c. tube feeding bags and sets are changed every 24 hours d. food allergies are documented in the medical record

b. customers are satisfied with the meals (information gathered that reflects whether the process achieved desired results)

which piece of food service equipment is the least energy efficient a. convection oven b. deck oven c. steam jacketed kettle d. rotary oven

b. deck oven

you are preparing dinner for a 500 bed facility. the menu includes roast beef, mashed potatoes and carrots. which equipment is best suited for this task a. a convection oven and a tilting braising pan b. deck ovens and pressure steamers c. a microwave oven and a trunion kettle d. a rotary oven and a steamer

b. deck ovens and pressure steamers (deck ovens - stacked to save space, used when production is high and space is limited; pressure steamers - facilitate speedy batch cooking which allows for better quality, reduced holding time)

when switching from a conventional to a convection oven, a. increase the temperature by 15 degrees b. decrease the temperature by 25 degrees c. increase the temperature by 12 degrees d. decrease the temperature by 5 degrees

b. decrease the temperature by 25 degrees

looking at the above question, what else would you expect to see a. increased bicarbonate, increased potassium, decreased phosphorus b. decreased bicarbonate, increased potassium, increased phosphorus c. decreased bicarbonate, decreased potassium, increased phosphorus d. increased bicarbonate, increased potassium, increased phosphorus

b. decreased bicarbonate, increased potassium, increased phosphorus

prior to surgery a patient has a serum albumin level of 3.7mg/dl. post surgery the level is 3.1mg/dl. you recognize that a. an error has occurred b. decreased serum albumin levels are normal after surgery c. the patient is experiencing protein-losing enteropathy d. the patient has visceral protein depletion

b. decreased serum albumin levels are normal after surgery

which is a good method of food handling a. sanitize raw food before preparation b. defrost frozen food in the refrigerator c. refrigerate raw food at 48F d. thaw frozen food quickly under running hot water

b. defrost frozen food in the refrigerator

116. If the community nutritionist needed information about residents who are at least 65 years old, the best source of existing information would be the: [NTR: 510] a) department of health and human services b) department of agriculture c) department of education d) center for disease control and prevention

b. department of agriculture

the purpose of outcomes research is to a. determine the best approach to solving a problem b. determine the most cost-effective way to solve a problem c. determine the most productive outcomes d. determine the best outcome desired

b. determine the most cost-effective way to solve a problem (evaluate the effectiveness(help achieve objectives) and efficiency(minimal use of resources to achieve objective) of an entire process; data collected/analyzed so performance can be adjusted/improved; findings compared with past performance, standards, benchmarks; goal is to improve quality of services offered)

which of the following medications is an appetite suppressant a. prednisone b. dextroamphetamine c. erythromycin d. dronabinol

b. dextroamphetamine

beef stew has been unrefrigerated for 4 hours after preparation. what should you do a. reheat it to 140F and refrigerate it in a shallow pan b. discard the stew c. reheat it to 165F and refrigerate it d. refrigerate the stew in shallow pans

b. discard the stew

you are a supervising dietitian and are very satisfied with one of your diet aides who works very hard. one year after hiring her, you realize the information on her application wasn't true. what should you do a. say nothing and ignore it b. discuss it with her one on one in your office c. inform human resources about the situation d. fire her

b. discuss it one on one in your office

the most important critical control point to monitor in the preparation and service of canned cream soup is the temperature a. at delivery b. during steam table holding c. of storage d. during end-point cooking

b. during steam table holding

the best way to counsel a patient with anorexia nervosa is to a. recommend that the patient increase food intake and attend weekly counseling sessions b. emphasize the health benefits associated with weight gain and increased calorie intake c. maintain a no nonsense attitude to discourage inappropriate behavior d. remain flexible to help the patient set realistic goals

b. emphasize the health benefits associated with weight gain and increased calorie intake

the human relations theory of management stresses a. tasks, structure, and authority b. employee's needs c. direct control by management d. management's needs

b. employee's needs

a patient on lithium decreases his sodium intake to 2grams/day. the effect will be a. decreased activity of the drug b. enhanced activity of the drug c. no change in drug activity d. reduced response to the drug

b. enhanced activity of the drug

you are estimating the prevalence of inadequate intake levels within a group of adults. which of the following dietary reference intake guidelines should be used a. recommended dietary allowance b. estimated average requirement c. tolerable upper intake level d. adequate intake

b. estimated average requirement

the best way for you to follow-up on a patient's application of diet instruction provide is to a. give pre and post tests on recommendations b. evaluate his food diary for one week c. ask him how he is doing with the new plan d. use body language to determine his interest in following the plan

b. evaluate his food diary for one week

an athlete weighs more than the average person of his height. what should you do first a. put him on a slow weight reduction diet b. examine other anthropometric measurements c. have him increase his exercise d. suggest he drink more water

b. examine other anthropometric measurements

a food service manager who supervises 35 employees has a problem with discipline and increased absenteeism. why? a. decreased unity among managers b. excessive span of control c. decreased unity of control d. excessive unity of command

b. excessive span of control

if an IDDM tells you she eats meals on an irregular schedule, you should a. obtain more information about her usual eating habits b. explain that insulin therapy is related to intake c. do nothing if blood sugar levels are okay d. ask her to complete a detailed food record

b. explain that insulin therapy is related to intake

a patient comes to you with a diet that you know is not good to follow. what should you do a. tell him its not good b. explain why it is not good c. tell him to only follow what you have provided d. let him do as he wishes

b. explain why it is not good

a spastic quadriplegic requires a. lower energy intake b. fewer calories, increased fluid and fiber c. increased fluid and fiber, acid ash diet d. lower energy intake, acid ash diet

b. fewer calories, increased fluid and fiber (spastic quadriplegia is a form of cerebral palsy, movement is difficult w/limited activity, often obese; requires low calorie, high fluid, high fiber diet)

what type of food would you feed to a patient who has the athetoid form of cerebral palsy a. milk-based products b. finger foods like a hot dog in a roll c. milk-free products d. galactose-free foods

b. finger foods like a hot dog in a roll

which of the following must be included when preparing a budget a. controlling expenses b. forecasting revenue c. controlling labor costs d. proper cash control

b. forecasting revenue

the test that measures toxic levels of lead on heme synthesis is the a. prothrombin test b. free erythrocyte protopophyrin test c. hemoglobin value test d. MCV

b. free erythrocyte protopophyrin test

which of the following would lacto-ovovegetarian not eat a. eggs b. halibut c. cheese d. beans

b. halibut

why would a man who eats very little protein have an albumin of 5.5g/dl a. carbohydrate keeps albumin levels elevated b. he is dehydrated c. serum albumin falls quickly with protein deprivation d. it is a sign of protein malnutrition

b. he is dehydrated

which of the following best describes the expected outcomes of dietary intervention for a patient who has anorexia nervosa a. he will gain 4lbs in 2 weeks through tube feeding b. he will eat more often and try a larger variety of foods c. his diet will be supplemented with a calorie and protein dense liquid d. he will learn more about the effects of dietary fiber on GI transit time

b. he will eat more often and try a larger variety of foods

a patient with diabetic gastroparesis should avoid foods that are a. high in simple sugars b. high in fat c. low in fiber d. low in protein

b. high in fat

low levels of folate, pyridoxine and cyanocobalamin are associated with a. glycogen storage disease b. homocysteinurias c. phenylketonuria d. galactosemia

b. homocysteinurias (an inherited disorder in which the body is unable to process certain building blocks of proteins (amino acids) properly)

which of the following nutrition diagnoses is most likely to be associated with cyclosporine therapy a. hyperphosphatemia b. hyperlipidemia c. hypoglycemia d. hypokalemia

b. hyperlipidemia (an immunosuppressant that can cause: hyperlipidemia, hyperglycemia, hyperkalemia, hypertension)

a nutrition surveillance program is designed primarily to a. verify data from other surveys b. identify the nutritional needs of the population c. indicate the number of people at increased risk for medical conditions d. indicate the causes underlying the nutrition problems identified

b. identify the nutritional needs of the population

when should you provide nutrition counseling to an AIDS patient a. when nausea and vomiting become problems b. immediately upon diagnosis c. when diarrhea occurs d. no counseling is needed

b. immediately upon diagnosis

an athlete has recently increased his intake of chicken, tuna and beef to increase muscle mass. however he reports decreased energy and poor performance at practice. what is the likely cause a. excessive protein intake b. inadequate carbohydrate intake c. depletion of sodium d. increasing cholesterol level

b. inadequate carbohydrate intake

the most relevant objective for a multiyear project directed at reducing the incidence of colon cancer in a community is to a. increase by 50% in 1 year adult's knowledge about the association between vitamin A and colon cancer b. increase by 50% in 1 year middle-aged men's knowledge about the association between high levels of dietary fat and colon cancer c. decrease by 10% during the next year the intake of alcoholic beverages among those between aged 21 and 55 d. within the next year provide training in diet and cancer prevention to leaders in local health clubs and fitness centers

b. increase by 50% in 1 year middle-aged men's knowledge about the association between high levels of dietary fat and colon cancer

in acute hepatitis, you would likely see a. decreases in SGOT, increase in SGPT b. increase in both SGOT and SGPT c. decrease in both SGOT and SGPT d. increase in SGOT and decrease in SGPT

b. increase in both SGOT and SGPT (Serum Glutamic oxaloacetate transaminase and Serum glutamic pyruvic transaminase - released when liver is damaged)

a patient on megestrol acetate may experience a. weight loss b. increased appetite c. increased diarrhea d. iron-deficiency anemia

b. increased appetite

the immediate post stress release of glucagon a. increases glucose uptake into cells b. increases glucose release into the bloodstream c. increases insulin release d. increases fluid and sodium excretion

b. increases glucose release into the bloodstream

which method of buying includes contacting several vendors to obtain price quotes and placing orders a. prime vendor b. informal c. formal d. contract

b. informal

when a manager updates the director about activities occurring on the patient tray line, he is demonstrating which managerial role a. interpersonal b. informational c. technical d. decisional

b. informational

a long term care patient has normal intestinal function but cannot eat. what tube feeding should you provide a. high protein, high calorie formula b. isotonic lactose free formula c. elemental formula d. low fiber, low fat formula

b. isotonic lactose free formula (an isotonic formula has an osmolality near that of body fluid and is tolerated by most patients; elemental is chemically defined, predigested and doesn't require pancreatic enzymes)

the efficacy of a low sugar, low artificial flavor diet on hyperkinesis is a. it works in two out of three patients b. it has not been scientifically proven effective c. it has never been tested d. it has been proven effective

b. it has not been scientifically proven effective (hyperkinesis is increase in muscular activity that results in uncontrollable/increased muscular movements)

which of the following is true about SGOT a. it is increased with hepatitis and diabetes b. it is increased with hepatitis, decreased with diabetes c. it is decreased in both hepatitis and diabetes d. it is decreased with hepatitis and increased with diabetes

b. it is increased with hepatitis, decreased with diabetes

current research on the relationship between cancer and diet includes all of the following except a. cancer is associated with the intake of vitamin A b. it is related to demographic factors c. it is related to epidemiologic data d. it is associated with ascorbic acid

b. it is related to demographic factors (interrelationship between host, agent, environment in causing disease, some evidence that fruits and vegetables are beneficial in overall cancer prevention (carotenoids, vit C)

which of the following is an example of productivity management a. labor turnover rate b. labor minutes per day c. food cost percentage d. edible portion cost

b. labor minutes per day

following a Billroth II, steatorrhea often results. what is the likely cause a. bacterial overgrowth in the remaining loop b. lack of pancreatic secretion due to less duodenal hormone secretion c. a diet restricting simple carbohydrates d. rapid absorption of simple carbohydrates

b. lack of pancreatic secretion due to less duodenal hormone secretion

a child is lethargic, irritable with diarrhea. the cause may be a. zinc toxicity b. lead poisoning c. iron overload d. vitamin A toxicity

b. lead poisoning

you have a hospitalized renal failure patient who has eaten 0-10% of her meals for the past three days. you should recommend a. tube feeding b. liberalizing the diet c. switching to a liquid diet d. initiating peripheral parenteral feeding

b. liberalizing the diet

a 6'2" male, large frame, 230 pounds with type II diabetes should a. gain 5 pounds to be at desirable body weight b. lose 20 pounds slowly following balanced diet c. lose 20 pounds in 4 weeks on an 800 calorie diet d. lose 10 pounds slowly by following a balanced diet

b. lose 20 pounds slowly following balance diet

325. Which vitamin enhances iron absorption? [NTR:619] a) vitamin A b) vitamin B2 c) vitamin C d) vitamin D

c) vitamin C

which of the following is most important in determining the space required for a food production unit in a new facility a. number of hours the foodservice will be operating b. market form of foods purchased c. number of foodservice employees d. length of menu cycle

b. market form of foods purchased

when counseling a diabetic with limited education, which would be the most appropriate question to ask in assessing her comprehension level a. do you enjoy candy and sweets b. name a food that is a fat c. do you have difficulty swallowing or chewing d. how many meals and snacks to you consume each day

b. name a food that is a fat

which program enables a juvenile justice residential facility to provide nutritious meals to its residents a. supplemental nutrition assistance program b. national school breakfast program c. expanded food and nutrition education program d. the emergency food assistance program

b. national school breakfast program

a 50 bed hospital has limited kitchen space for refrigeration, dry storage and food production. the most appropriate type of menu is a. restaurant style b. nonselective c. selective d. single-use

b. nonselective

a child presents at the 65th percentile height for age what does this signify a. overweight b. normal c. underweight d. small for age

b. normal

the BMI for a woman who is 5'7" tall and weighs 148 pounds indicates a. underweight b. normal weight c. overweight d. obese

b. normal weight

a dietitian who is developing policies and procedures for nourishment distribution and service should first a. evaluate the nourishment distribution schedule b. note how nourishments are being distributed and served c. assign the foodservice supervisor the task of gathering data d. review policies and procedure manuals from other facilities

b. note how nourishments are being distributed and served

what change should be made in the following menu if it is to be served to seventh day Adventists. tomato juice, cottage cheese fruit plate or egg salad sandwich, milk, coffee, tea a. offer fortified soy milk b. offer decaffeinated beverages c. offer tofu as an alternative to cottage cheese d. offer a meat analog as an alternative to egg salad

b. offer decaffeinated beverages

nutrition intervention with Alzheimer's disease should include a. eating with TV on provides stimulation b. offer one food at a time, avoid distractions c. encourage eating meals with others d. several small meals and snacks throughout the day

b. offer one food at a time, avoid distractions

an AIDS patient should select the following fresh fruit a. grapes, orange b. orange, banana, pineapple c. apple, grapes, peach d. grapes, pear

b. orange, banana, pineapple

which of the following is not a guideline for Medicaid a. low income b. over the age of 60 only c. nutritional need d. a e. b and c f. b g. c

b. over the age of 60 only

which of the following kitchen layouts will help prevent other employees from walking through the cook's work area a. plan each work center with its own supply storage areas b. plan work aisles separate from traffic aisles c. position work centers adjacent to the appropriate point of service d. design aisles wide enough to accommodate both

b. plan work aisles separate from traffic aisles

management by objectives refers to a management process which employees a. periodic, preplanned, broad objectives b. precise, measurable objectives c. determination of goals by department superiors d. objectives of instruction set by top management

b. precise, measurable objectives (democratic management that provides control from within; establish performance goals with employees, gives higher incentive value, overall control achieved through self-control by employees, management stresses accomplishments and results; participative leadership)

urea excretion is related to ___, while creatinine excretion is related to ____ a. muscle mass, protein intake b. protein intake, muscle mass c. protein intake, adipose reserves d. protein intake, calcium intake

b. protein intake, muscle mass

a blast freezer and rethermalization are part of the a. commissary system b. ready-prepared system c. assemble serve system d. conventional system

b. ready-prepared system (cook-chill; cook-freeze; foods are prepared on site and then frozen or chilled for later use)

if you are working on a cost benefit analysis you would a. record the number of patient complaints b. record the number of hours the dietitian spent providing nutrition education. c. record the number of patients who were satisfied with the education provided d. keep a record of all who failed to attend

b. record the number of hours the dietitian spent providing nutrition education

when working with patients who have food allergies in a pediatric GI clinic, which of the following outcomes should be measured a. frequency of patient compliance with instructions b. reduction of food related reactions among patients c. level of knowledge about causes of food allergies d. patient satisfaction with dietitians interaction

b. reduction of food related reactions among patients

what would you do if an employee does not wash pots as required a. fire her b. reeducate her as to proper procedures and provide follow-up c. take the job away from the employee d. discipline the employee

b. reeducate her as to proper procedures and provide follow-up

the recommendation to treat an infant with acute diarrhea is to a. use a half strength formula b. rehydrate within 4-6 hours c. limit or restrict use of a formula containing fiber d. administer glucose and electrolytes only for the first 24 hours

b. rehydrate within 4-6 hours

if there is an increase in input and output, productivity will a. increase b. remain constant c. decrease d. modulate

b. remain constant

if a union member repeatedly arrives late, you should a. fire him b. report him to management c. say nothing but make a note of it d. forget it

b. report him to management

a teenager was brought to the hospital with CVA and dysphagia. the dietitian should immediately a. order a nasogastric feeding b. request a swallow test c. order a dysphagia diet d. order a CVA diet and disregard the dysphagia

b. request a swallow test

the organizational chart a. is an organized list of duties b. shows the employee how he fits into the organization c. organizes departmental activities d. is an evaluation tool for productivity

b. show the employee how he fits into the organization

iron deficiency anemia has the following characteristics a. large cells laden with hemoglobin, hypochromic, microcytic b. small, pale cells, hypochromic, microcytic c. large cells laden with hemoglobin, macrocytic, megaloblastic d. small, pale cells, macrocytic, megaloblastic

b. small, pale cells, hypochromic, microcytic

a patient with dehydration exhibits the symptoms of a. high blood pressure, dizziness b. sunken eyes, decreased renal function, increased tachycardia c. decreased BUN, decreased serum creatinine d. low blood pressure, low serum sodium

b. sunken eyes, decreased renal function, increased tachycardia

the national sanitation foundation standard for equipment construction that is most related to microbiological food safety requires that a. outside corners are bull-nosed b. surfaces do not have open seams or cracks c. equipment surfaces are without sharp edges d. bolts are securely fastened

b. surfaces do not have open seams or cracks

a client who takes orlistat may need to a. use MCT as a supplement b. take a multivitamin supplement c. take an omega-3 fatty acid supplement d. avoid caffeinated and carbonated beverages

b. take a multivitamin supplement (Alli - weight loss medication, prevents body from absorbing some of the fat eaten)

you teach food service workers about pot washing temperatures. how do you know if the workers then follow the rules a. ask them to repeat the rules b. test the water temperatures at frequent intervals c. ask them to write down the guidelines d. check to see if the pots are clean

b. test the water temperatures at frequent intervals

an athlete needs 4000 calories. the cornflakes nutrient label says that one serving provides 25% of the RDA for CHO. how many servings of cornflakes does he need to get his 25% a. one b. two c. three d. four

b. two (labels are based on a 2000kcal diet so would need two servings)

if the temperature on the dishmachine rinse cycle is 135F, you should a. add additional sanitizer b. use paper products until the problem has been resolved c. nothing - that is the proper temperature d. increase the length of the rinse cycle

b. use paper products until the problem has been resolved

a vegan is allergic to milk. which nutrients would you expect might be deficient in her intake a. protein, calories b. vitamin B12, D c. vitamin C, calcium d. vitamin A, D

b. vitamin B12, D

following a colectomy, the absorption of which of the following would be of the most concern a. vitamins D and E b. water and sodium c. B vitamins d. vitamin C

b. water and sodium

a man with HIV disease presents with the following data. what should be your primary concern as his dietitian Height: 5'10" Weight: 115lbs Hematocrit: 35% MCV 102fl BUN 11ml/dl serum albumin 2.8g/dl CD4 280 total lymphocytes 1200/cubic mm a. anemia b. weight c. renal failure d. protein deficiency

b. weight

when is breast feeding contraindicated a. when the mother is underweight b. when the mother has AIDS c. when the mother has a breast infection d. when the mother has hyperlipidemia

b. when the mother has AIDS

according to Chinese beliefs, foods and herbs that may be used to treat illnesses are grouped as a. yin (bright, hot)/ yang (dark, cold) b. yin (dark, cold)/yang (bright, hot) c. action/maintenance d. harmful/beneficial

b. yin (dark, cold)/yang (bright hot) (Yin: raw, cold, fish, vegetables, fruits; Yang: bright, hot, hot soup from chicken; rice is neutral)

When counseling an anorexic, the best advice is to: a. be real strict in their food choices b. be flexible and help them choose reasonable goals c. provide a large variety of menu items d. give them free rein with their intake

be flexible and help them choose reasonable goals

If you are on a low phosphorus diet, what food should you most avoid? a. beef b. orange juice c. yogurt d. bread

beef

Which cooking method would best convert collagen to gelatin in bottom round roast? a. grilling it to medium rare b. braising c. broiling d. searing

braising

101. What would the nitrogen balance be for a patient with a 24-hour urinary nitrogen excretion of 14.3 grams and a protein intake of 102 grams? [NTR:495] a) +16.3 grams b) -16.3 grams c) -2 grams d) +2 grams

c) -2 grams Nitrogen Balance = Nitrogen Intake - Nitrogen Output Nitrogen intake = gm Protein consumed over 24 hr/6.25 (divide grams of protein by 6.25 because 1gm Protein = 6.25gm Nitrogen) Nitrogen output = 24 HR UUN result + 4 (losses from sweat, etc) If your result is a zero balance, you need to add 2-4 gm N2 to make that a positive N2 balance. If your result is a negative balance, you need to add as much protein to bring to zero + 2-4gm N2.

32. Which of the following foods contains less than 20 calories? [NTR:427] a) 1 Tbsp peanut butter b) 1 Tbsp sunflower seeds c) 1 Tbsp catsup d) 1 Tbsp reduced-fat mayonnaise

c) 1 Tbsp catsup

339. The 2010 Dietary Guidelines for Americans suggest what percentage of calories comes from saturated fat? [NTR:633] a) 1% b) 5% c) 10% d) 20%

c) 10%

65. Enteral feeding started within _____ hours of admission to the intensive care unit has been shown to improve length of stay and mortality. [NTR:460] a) 2 b) 12 c) 24 d) 48

c) 24

46. When performing a nutritional assessment on a child, head circumference should be measured up to the age of: [NTR:441] a) 12 months. b) 18 months. c) 24 months. d) 36 months.

c) 24 months.

6. The most appropriate nutrition prescription for your patient with cirrhosis includes: [NTR:401] a) 0.6 grams protein/kg body weight b) 40-45% fat c) 4-6 smaller meals per day d) 4-6 grams sodium

c) 4-6 smaller meals per day

104. When talking about program planning for community nutrition, to what would the term management refer? [NTR:498] a) Controlling and providing oversight for financial resources b) Utilizing time effectively and efficiently c) Personnel and data systems d) Coordinating and classifying objectives

c) Personnel and data systems

223. What are the estimated kilocalorie needs of a 4-month-old breastfed infant girl who weighs 14 pounds? [NTR:517] a) 509 b) 636 c) 687. d) 724

c) 687 The weight in kilograms is the number you will use in the calculation. Calculate your child's calorie requirements according to her age. If your infant is age less than 6 months old, the calorie requirements are 103 calories per kilogram. If your infant is between 6 and 12 months old, the calorie requirements are 98 calories per kilogram.

68. In the Nutrition Care Process, where should the dietetics professional note the client's BMI? [NTR:463] a) Food/nutrition-related history b) Biochemical data, medical tests and procedures c) Anthropometric measurements d) Nutrition-focused physical findings

c) Anthropometric measurements

83. For which two steps in the Nutrition Care Process are the terms combined? [NTR:478] a) Assessment and Intervention b) Monitoring and Evaluation and Diagnosis c) Assessment and Monitoring and Evaluation d) Diagnosis and Intervention

c) Assessment and Monitoring and Evaluation

93. The following programs are recommended as primary interventions in school-based child and adolescent overweight prevention programs: [NTR:488] a) Food environment changes, homework/reading/computer use changes, media influences, nutrition education b) Food environment changes, nutrition education, physical activity education c) Behavioral counseling, nutrition education, physical activity education, parental/family involvement

c) Behavioral counseling, nutrition education, physical activity education, parental/family involvement

73. Which of the following is one of the four domains of Nutrition Intervention in the Nutrition Care Process? [NTR:468] a) Client history b) Nutrition-focused physical findings c) Coordination of care d) Food/nutrition-related history

c) Coordination of care (4 domains: food and/or nutrient delivery, nutrition education, nutrition counseling, coordination of nutrition care)

235. Calculating protein needs at 1.5 g/kg body weight in an elderly patient with Stage III pressure ulcer and impaired renal function may cause [NTR:529] a) hyperglycemia. b) dyslipidemia. c) Dehydration d) increased wound healing.

c) Dehydration

320. Which dietary reference intake value (DRI) estimates the nutrient needs to meet the requirements of at least half of individuals in a population? [NTR:614] a) RDA b) AI c) EAR d) UL

c) EAR

254. The Campniha-Bacote Model of Cultural Competence includes the interdependent constructs of cultural awareness, cultural knowledge, cultural skill, cultural desire, and cultural _____. [NTR:548] a) Understanding b) Communication c) Encounter d) integration

c) Encounter

350. Approximately how many servings of milk or yogurt would a woman 53 years of age need to consume to meet her RDA for calcium? [NTR:645] a) Two b) Three c) Four d) Five

c) Four

341. Which of the following amino acids is classified as a nonessential amino acid? [NTR:635] a) Valine b) Lysine c) Glycine d) Threonine

c) Glycine

36. Which of the following population groups of children does research show to be at the highest risk for iron deficiency? [NTR:431]) a) School-age children and children ages 3 to 5 years old b) Infants and school-age children c) Infants and adolescents d) Children ages 3 to 5 years old and adolescents

c) Infants and adolescents

44. During their peak growth spurt, adolescent males are at greatest risk for a deficiency of: [NTR:439] a) protein. b) copper. c) Iron d) phosphorus.

c) Iron

351. Choose the statement that best describes the function of copper. [NTR:646] a) It is involved in taste perception. b) It is a component of the thyroid hormone. c) It is necessary for the formation of hemoglobin. d) It enhances insulin activity.

c) It is necessary for the formation of hemoglobin.

64. Which would be an appropriate food or beverage item to serve to your patient on postoperative day #1 following gastric bypass surgery? [NTR:459] a) Ginger ale b) Sherbet c) Low-fat chicken broth d) Cranberry juice

c) Low-fat chicken broth

317. What have studies in the areas of literacy and special education shown about the use of drawings vs. photographs? [NTR:611] a) Drawings are generally easier to grasp because of their simplicity. b) Photographs are generally preferred because of their accurate presentation of reality. c) Neither is necessarily more effective than the other. d) Both are problematic because they are two-dimensional.

c) Neither is necessarily more effective than the other.

74. Which step in the Nutrition Care Process involves the dietetics professional establishing goals jointly with the patient/client? [NTR:469] a) Nutrition Assessment b) Nutrition Diagnosis c) Nutrition Intervention d) Nutrition Monitoring and Evaluation

c) Nutrition Intervention

76. Which step in the Nutrition Care Process involves the dietetics professional coordinating nutrition care with other health care providers, institutions, or agencies? [NTR:471] a) Nutrition Assessment b) Nutrition Diagnosis c) Nutrition Intervention d) Nutrition Monitoring and Evaluation

c) Nutrition Intervention

69. In the Nutrition Care Process, statements in the format of "Nutrition problem label related to ____ as evidenced by ____" are classified as: [NTR:464] a) intake critiques. b) NCP links. c) PES statements. c) nutrition concerns.

c) PES statements.

247. Which health education planning model demonstrates that lasting behavior change is voluntary and must include input from the client? [NTR:541] a) PATCH b) The 10-step Planning Model c) PRECEDE-PROCEED d) CDCynergy

c) PRECEDE-PROCEED

25. Which food allergy is the leading cause of fatal and near-fatal food allergic reactions in the United States each year? [NTR:420] a) Egg b) Fish c) Peanut d) Wheat

c) Peanut

100. Which enteral feeding access device will deliver nutrition past the ligament of Trietz? [NTR:494] a) Nasogastric b) Nasoduodenal c) Percutaneous jejunostomy d) Percutaneous gastrostomy

c) Percutaneous jejunostomy

What is the KEY element in disciplining employees? [MGT:338] a) Identifying solutions to correct undesirable behavior b) Providing feedback c) Providing consistency d) Identifying causes for unacceptable behavior

c) Providing consistency

233. Which of the following does the Braden Scale measure? [NTR:527] a) Risk factors that may lead to undernutrition b) Medical conditions that affect the ability to feed oneself c) Risk assessment for pressure ulcers d) Appetite stimulants in the patient or client's environment

c) Risk assessment for pressure ulcers

111. Which of the following social or economic trends is thought to have the most important consequences for future community nutrition practice? [NTR:505] a) The segment of the US population that is growing most rapidly is children ages 12 years and younger. b) Different generations in the United States have similar values and attitudes about health. c) The deteriorating global environmental indicators such as climate change, degradation of topsoil, and increased use of many chemical pollutants. d) The food purchasing practices and preferences in the United States that indicate a desire for an extensive variety of "value-added" pre-prepared foods.

c) The deteriorating global environmental indicators such as climate change, degradation of topsoil, and increased use of many chemical pollutants.

257. Which of the following is true of Jewish food practices? [NTR:551] a) Kosher means "blessed by a Rabbi." b) Jews keeping Kosher are permitted to eat dairy and meat items in the same meal. c) The food traditions of Ashkenazi Jews are rooted in those of France, Germany, and Eastern Europe. d) According to kosher laws, Jews may eat rabbit.

c) The food traditions of Ashkenazi Jews are rooted in those of France, Germany, and Eastern Europe.

347. A patient presents with lesions on his gums and pinpoint hemorrhages on his skin. What vitamin is this patient most likely deficient in? [NTR:642] a) Thiamin b) Magnesium c) Vitamin C c) Selenium

c) Vitamin C

324. Rickets is a result of which vitamin deficiency? [NTR:618] a) Vitamin A b) Vitamin C c) Vitamin D d) Vitamin K

c) Vitamin D

312. Which of the following is a good predictor of central adiposity? [NTR:606] a) Hip circumference b) Basal metabolic rate c) Waist circumference d) Body weight

c) Waist circumference

293. Which of the following nutrients is essential for growth and sexual maturation in adolescents? [NTR:587] a) Calcium b) Selenium c) Zinc d) Iron

c) Zinc

40. The best studied and most frequently recommended weight-loss dietary strategy is: [NTR:435] a) a low-carbohydrate high-protein diet. b) a low fat high-protein diet. c) a low-fat reduced-energy diet. d) a low carbohydrate reduced-energy diet.

c) a low-fat reduced-energy diet.

261. Culture, race, and ethnicity_____. [NTR:555] a) are all facets of cultural competency b) are all genetically-predisposed factors c) are distinct concepts d) should not impact the professional-client interactions if the practitioner is culturally competent

c) are distinct concepts

229. A deficiency of vitamin B12 in older adults is most likely due to [NTR:523] a) diverticulosis. b) fat malabsorption. c) atrophic gastritis. c) gallbladder disease.

c) atrophic gastritis.

288. Which of the following foods would expect to empty from the stomach the quickest? [NTR:582] a) French fries b) oatmeal c) chocolate milk d) scrambled eggs

c) chocolate milk

50. A child who plots consistently at the 5th percentile on the weight-for-length/stature Centers for Disease Control and Prevention growth chart [NTR:445] a) demonstrates a negative pattern of growth. b) demonstrates failure to thrive. c) has a thin body habitus. d) reflects a change in growth velocity.

c) has a thin body habitus.

90. The Academy of Nutrition and Dietetics defines tertiary prevention related to pediatric overweight interventions as: [NTR:485] a) overweight prevention efforts that include identifying and intervention efforts targeting children who are at risk for overweight. b) education and prevention programs targeting children and adolescents before they are overweight. c) interventions targeting overweight children designed to slow down or reverse the increase in BMI.

c) interventions targeting overweight children designed to slow down or reverse the increase in BMI.

119. The city manager wants to be sure that the tool the community nutritionist is using to collect data about the use of dietary supplements has validity. In order to ensure that the instrument is valid, the nutritionist needs to choose an instrument that [NTR:513] a) has been used previously many times. b) contains words and phrases familiar to participants. c) measures accurately what it is intended to measure. d) yields primarily quantitative data.

c) measures accurately what it is intended to measure.

314. As the pH of a glass of orange juice decreases, it becomes _____. [NTR:608] a) Hydrophobic b) Hydrophilic c) more acidic d) more basic

c) more acidic

335. Which of the following food program is the largest federally funded child nutrition assistance program? [NTR:629] a) supplemental nutrition assistance program b) child and adult care food program c) national school lunch program d) summer food service program

c) national school lunch program

313. This reaction H2® 2H+ + 2e- represents: [NTR:607] a) redox. b) reduction. c) oxidation. c) oxygenation.

c) oxidation.

113. In a Community Needs Assessment report, the statement; "...the southeast region of Essex County, including the adjoining municipalities of Kardon, Greenwalt, and Addison...," is an example of: [NTR:507] a) the target population. b) the lead organizations. c) the definition of the community. d) the stakeholders.

c) the definition of the community.

54. A typical feeding/eating pattern for infants is [NTR:449] a) to need apple juice at age 4 months to regulate stooling. b) to need cereal at bedtime after age 3 months to be able to sleep through the night. c) to need only breast milk or iron-fortified formula for the first 6 months of age. d) hard to predict since their interest in eating is so variable.

c) to need only breast milk or iron-fortified formula for the first 6 months of age.

245. A family is having difficulty providing food for all its members. The father skips meals occasionally and the mother eats very little to ensure that the children have enough to eat. Using established categories of food security status, this family would be classified as having [NTR:539] a) high food security. b) marginal food insecurity. c) low food security. c) very low food security.

c) very low food security.

the employee turnover rate refers to the following a. total replacement employees divided by average work force b. % of turnover rate divided by number of replacements c. # employees terminated and replaced divided by total number of positions d. # total positions divided by number part time employees

c. # employees terminated and replaced divided by total number of positions

an 8 lb roast beef cost $2.79/lb. the edible portion is 65%. what is the AP price of the roast a. $21.19 b. $22.00 c. $22.32 d. $23.04

c. $22.32

using the following data, the average check for September is: total revenue: $909009; net income: $173010; FTEs: 27.9; number of customers: 32613 a. $5.30 b. $6.29 c. $27.87 d. $33.04

c. $27.87 (total revenue/# of customers)

a business pays 4% of its income as rent ($1200), 70% towards labor and food costs, and $2500 towards other monthly expenses. how much profit does this business make per month a. $6000 b. $4200 c. $5300 d. $7200

c. $5300 (1200/.04=30000 x 70% = 21000; 30000-(1200+21000+2500 )=5300 profit)

if the productivity goal for a patient tray assembly unit is 4 trays per minute, the unit should be able to assemble approximately 350 trays in a. 1 hour b. 1 hour 15 minutes c. 1 hour 30 minutes d. 1 hour 45 minutes

c. 1 hour 30 minutes (350/4 = 87.5 minutes to prepare 350 trays)

which of the following should the patient in the above question have for lunch a. 4oz cottage cheese b. 1 oz cheddar cheese c. 1 oz roast beef d. 1 oz cold cuts

c. 1 oz roast beef

which of the following is the appropriate diet for peritoneal dialysis a. .8 grams protein, 35kcal/kg b. 2.0 grams protein, 30kcals/kg c. 1.5 grams protein, 30kcals/kg d. 1.0 grams protein, 35kcals/kg

c. 1.5grams protein, 30kcals/kg

a 60kg female receives 2700 calories on PN. the calorie to nitrogen ratio is 150:1. How many grams of nitrogen is she getting a. 14 b. 16 c. 18 d. 20

c. 18 (2700/150=18)

if an employee works 5 days each week, how many regular employees can be covered by one relief worker a. 1 1/2 b. 2 c. 2 1/2 d. 3

c. 2 1/2 (hospital food service positions cover a 7 day week. employees work 5 days and have 2 days off; a relief worker covers those 2 days off and can also work a 5 day week; a relief worker can cover the "days off" of a 2.5 full time workers each week. 5/2=2.5)

based on an 8 hour day, the number of hours worked was 55,267 per year. the total number of hours paid was 59,995. calculate the number of productive FTEs a. 2.27 b. 18.90 c. 26.60 d. 28.80

c. 26.60 (55267 # of hours worked / 2080 FTE per year = 26.57 FTEs)

a cafeteria seats 80 customers. lunch runs from 11:15am to 1:00pm. turnover is 2.5 customers/hour. how many can be served during those hours a. 125 b. 216 c. 350 d. 550

c. 350 (2.5 x 1.75(hours open) = 4.375 x 80 seats = 350)

according to the ADA exchange lists, what is the nutrient content of the following: 1 cup cantaloupe, 1/2 cup cereal, 1 cup 1% milk, 1 tsp margarine, 1 tsp low sugar jelly, black coffee, 1 slice toast calories protein CHO fat a. 397 12 61 6.5 b. 346 12 52 9 c. 365 12 57 7 d. 327 13 58 10

c. 365 calories, 12g protein, 57g CHO, 7g fat

what is the benefit: cost ratio of a program intervention that cost $1890000 if the savings as a result of the program were $5.67 million a. 1:1 b. 2:1 c. 3:1 d. 1:3

c. 3:1 (5.67million/1.89million = 3; savings is 3x greater than cost)

there are seven foodservice employees in the tray assembly unit. the patient census is 350. if 1 1/2 hours of assembly time is allowed, a feasible productivity goal for the unit is to assemble how many trays per minute a. 2 b. 3 c. 4 d. 5

c. 4 (350 patients/90minutes = 3.89 trays per minute; number of employees is irrelevant)

infants should be given solid foods at a. 1-3 months b. 3-4 months c. 5-8 months d. 10-12 months

c. 5-8 months

a baker makes a batch of cookies in 18 minutes. what is the maximum number of batches he can make in 2 hours a. 4 b. 5 c. 6 d. 7

c. 6 (60/18=3.3334x2=6.6668)

if the above entrees had been served on Tuesday, how many servings of spaghetti will you need next Tuesday, if you expect to serve 300 total entrees a. 150 b. 63 c. 87 d. 101

c. 87 (40/total # of servings = 28.58% x 300)

determine the calorie contribution of carbohydrate from the following parenteral formula. the amounts stated are given per liter. 300ml 60% dextrose; 500 ml 10% amino acids; 200 ml 20% lipid; total nutrient mixture administered at 65 ml/hour for 24 hours a. 393 calories per day b. 1550 calories per day c. 955 calories per day d. 1572 calories per day

c. 955 calories per day (key is per liter: 65x24=1560ml, 1.560x300=468x60%=280.8x3.4)

who is most at risk for gestational diabetes a. BMI<30 b. age <25, overweight c. BMI>30, history of GSM d. conception within 10 months of last pregnancy

c. BMI>30, history of GSM

a father needs help in feeding his children, ages 6 and 8. which are the best options a. WIC and TANF b. Head Start and Food Stamps c. Food Stamps and School Lunch d. Commodity Foods and WIC

c. Food Stamps and School Lunch

230. Which of the following lists of the side effects of contraceptive injections, such as Depo-Provera, is accurate and complete? [NTR:524] a) Weight loss, decreased blood levels of HDL-cholesterol, and decreased bone density b) Weight gain and increased blood levels of LDL-cholesterol and insulin c) Weight gain, increased blood levels of LDL-cholesterol and insulin, decreased blood levels of HDL-cholesterol, and decreased bone density d) Increased blood levels of LDL-cholesterol and insulin, weight loss, and decreased blood levels of HDL-cholesterol

c. Weight gain, increased blood levels of LDL-cholesterol and insulin, decreased blood levels of HDL-cholesterol, and decreased bone density

a man with a peptic ulcer should consume a. milk every two hours b. a bland diet c. a balanced diet he can tolerate d. a low roughage diet

c. a balanced diet he can tolerate

an example of a healthy people goal is a. a decrease in saturated fat consumption b. an increase in iron intake c. a decrease in infant mortality d. an increase in the intake of complex carbohydrates

c. a decrease in infant mortality

if dishes come out of the dish machine with water spots a. use more detergent b. add a drying agent to the wash cycle c. add a drying agent to the rinse cycle d. use less detergent

c. add a drying agent to the rinse cycle

which is an example of line authority a. a part time consultant for WIC b. a consulting dietitian for an outpatient department c. an administrative dietitian in a 600 bed hospital d. a consulting dietitian for a nursing home

c. an administrative dietitian in a 600 bed hospital

the effectiveness of a group education program for decreasing the number of anemic pregnant women enrolled in a community nutrition program is best indicated by a. improved scores on a nutrition knowledge posttest b. improved scores on a Likert type attitudinal scale c. an increase in hematocrit or hemoglobin levels d. an increase in the intake of dietary iron

c. an increase in hematocrit or hemoglobin levels

in reviewing the patient's medical record, you note a hemoglobin of 10g/dl and an MCV of 110 cu mm. you should document a. anemia due to intrinsic factor b. anemia without nutritional implications c. anemia with larger than normal erythrocytes d. that the iron content of the diet should be increased

c. anemia with larger than normal erythrocytes

the drug methylphenidate may have adverse effect of a. increased excretion of folate requiring supplementation b. type 2 diabetes c. anorexia affecting growth d. involuntary weight gain

c. anorexia affecting growth (Ritalin; may cause anorexia, weight loss, nausea)

which bill requests government funding a. budget bill b. markup bill c. appropriations bill d. spending grant bill

c. appropriations bill

the FDA a. inspects meat, poultry, fish, eggs b. inspects factories c. enforces the Food, drug and Cosmetic Act d. monitors interstate shipping of shellfish e. a, b, c f. c, d g. b, c, d h. a, c

c. b, c, d (inspects factories, enforces the Food, drug and Cosmetic Act, monitors interstate shipping of shellfish)

14. In the Nutrition Care Process which of the following is a category for Nutrition Assessment? [NTR:409] a) Food and/or nutrient delivery b) Nutrition counseling c) Biochemical data, medical tests, and procedures d) Nutrition education

c. biochemical data, medical tests, and procedures

an appropriate source of folate for a female from central America is a. corn b. broccoli c. black beans d. raisins

c. black beans

which additive improves crispness in vegetables a. BHA b. BHT c. calcium d. pectin

c. calcium (additives used as humectants; retain moisture, increase firmness, tenderness)

during an interview, which of the following can you ask a. do you have a car b. how will you get to work c. can you get to work on time without any problem d. do you own a car

c. can you get to work on time without any problem

an athlete complains of weakness, rapid weight gain and muscle spasms. what may be the cause a. lack of adequate rest b. too much exercise c. carbohydrate loading d. not enough iron

c. carbohydrate loading

a child with PKU can eat a. spaghetti with tomato sauce b. vanilla ice cream c. carrots and celery sticks d. aspartame-sweetened gelatin

c. carrots and celery sticks (build up of phenylalanine causes need to limit/avoid: most foods with protein; milk, eggs, cheese, nuts, soybeans, beans, chicken, beef, pork, fish; limited potatoes, grains and vegetables containing protein; avoid aspartame)

which of the following tube feeding ingredients might precipitate an allergic reaction in a child with a milk allergy a. whey b. modified starch c. casein d. carrageenan

c. casein (milk allergy can be to casein or whey or both, the allergy should specify which type of allergy it is)

results of a patient food satisfaction survey indicate that patients think servings are too large. the dietitian should first a. conduct a plate waste survey b. reduce the serving size c. check the serving utensils on the line d. review regulations concerning nutrient requirements

c. check the serving utensils on the line

a five month old baby has changed from the 65th to the 95th percentile weight for height. the dietitian should first a. suggest the mother introduce more solid foods b. change the feeding formula c. check to see if weights have been plotted properly d. increase the amount of formula given

c. check to see if weights have been plotted properly

a child with cystic fibrosis has had a recent weight loss and several large, foul-smelling stools each day over the past two months. to evaluate the situation, the first step should be to assess her a. calorie intake b. total fat intake c. compliance with PERT d. 24 hour fecal fat

c. compliance with PERT

a dietitian ordering canned goods can be assured of a reliable product if they a. buy directly from the manufacturer b. accept the most reasonable bid c. conduct a can cutting analysis d. buy the same brand, grade, size as the last time

c. conduct a can cutting analysis

a patient who has cancer reports using herbs in addition to prescribed medication. the dietitian should a. advise him to stop taking the herbs while on medications b. recommend adjusting herbal dosages according to symptoms c. consult with the health care team d. encourage high doses of herbs to promote healing

c. consult with the health care team

a renal patient has come for dialysis on Monday morning with a serum sodium of 130 mEq/L, a BUN of 100 mg/dl, creatinine of 8.0 and a 4kg weight gain since his last dialysis on Friday morning. his laboratory indicators were in acceptable ranges on Friday afternoon. what would you expect happened over the weekend. a. consumed too little sodium, too much water, too little protein. b. consumed too little sodium, not enough water, too much protein c. consumed too much sodium, too much water, too much protein d. consumed too much sodium, too little water, too little protein

c. consumed too much sodium, too much water, too much protein (ESRD associated with BUN>100, Cr of 10-12mg/dl so these levels over weekend could indicate an increase in protein intake, the 4kg weight gain would be indicative of excess water consumption)

which of the following is a medium fat meat a. peanut butter b. tunafish packed in water c. corned beef d. chicken without skin

c. corned beef

an older sister of a child who has diabetes requests the child's record so she may give it to their family physician. the dietitian should first a. ask the child to sign a release form b. ask for a written request from a physician for a copy of the record c. determine who is the legal guardian d. provide the older sister a copy of the record

c. determine who is the legal guardian

the food service director who wants to hire a dietitian should first a. publish a job listing in the journal of academy of nutrition b. screen potential applicants to determine who has an RD credential c. develop a job description and specification for the position d. review dietitians resumes on file

c. develop a job description and specification for the position

which management functions should require the greatest allotment of time by a production manager a. planning and organizing b. organizing and directing c. directing and controlling d. organizing and controlling

c. directing and controlling

good advice for a patient with a hiatal hernia is to a. increase intake of complex carbohydrates b. lower the fat content of the diet c. don't eat before bed d. avoid coffee

c. don't eat before bed

which of the following is not a sign of marasmus a. severe protein and calorie wasting b. weight loss c. edema d. low anthropometric measurements

c. edema (Marasmus - protein and calorie starvation, normal serum albumin, no edema, severe fat and muscle wasting, starved appearance, decreased triceps skinfold and arm muscle circumference)

when determining the labor turnover rate in one period look at the number of a. employees at the beginning of the period and the number of new employees b. employees at the beginning of the period and the number who left c. employees at the end of the period and the number of new employees d. the number of employees at the end of the period and the number who left

c. employees at the end of the period and the number of new employees

the span of control refers to the number of a. supervisors b. part-time employees c. employees under the direction of one person d. staff positions

c. employees under the direction of one person

if a patient is about to be discharged when you receive the discharge diet order, you should a. give him something to take home b. there is not enough time so forget it c. enroll him in the outpatient clinic and schedule an appointment d. teach him what you can in the time you have now

c. enroll him in the outpatient clinic and schedule an appointment

if a child is at the 95th percentile weight for height and after one month drops to the 10th percentile, what does it likely signify a. poor nutrition b. lack of adequate caloric consumption c. error in measurement d. clinical disorder indicating malabsorption

c. error in measurement

a supervising dietitian who wishes to delegate specific responsibilities should first a. organize a task force b. check the production schedule c. establish expected outcomes d. establish expected time frames to accomplish tasks

c. establish expected outcomes

a clinical dietitian has successfully met an indicator for 6 months. the next step should be to a. establish a new indicator b. report the successful completion of intervention c. evaluate the need to continue monitoring the indicator d. evaluate the need to revise the standard of care

c. evaluate the need to continue monitoring the indicator

what is the best therapy for weight loss a. reducing 500 calories per day, and exercise b. support groups, exercise, diet control c. exercise, diet, behavior modification d. exercise and diet control

c. exercise, diet, behavior modificcation

a patient suffering from bulimia may have all of the following symptoms except a. a sore throat b. dental problems c. extreme weight loss d. rectal bleeding

c. extreme weight loss

what is the first thing you should do if an employee frequently comes to work without the proper uniform a. fire him b. give him a written warning c. find out why d. give him a warning, and fire him the next time it happens

c. find out why

to maintain hydration during continuous endurance physical activity of 4 hours, consume a. fluids b. fluids and sodium c. fluids, sodium, carbohydrates d. non-CHO electrolyte solutions

c. fluids, sodium, carbohydrates

which food items require the highest humidity a. meat b. dairy c. fruits, vegetables d. poultry

c. fruits, vegetables

a woman who is 5'5" tall weighs 190 pounds before pregnancy and gains 30 pounds by the beginning of her second trimester. advise her to a. maintain her current weight b. begin a weight reduction diet c. gain weight at an decreased rate d. continue to gain weight at the same rate

c. gain weight at an decreased rate

how can you enhance usage of commodity foods from a food donation program a. teach food donation staff how to use the items b. make a poster on usage c. give each participant recipes using the food items d. calculate the amounts of nutrients provided in the donated foods

c. give each participant recipes using the food items

the theory of obesity that refers to receptors in the hypothalamus is the a. lipostatic theory b. set point theory c. glucostatic theory d. glucose-obese theory

c. glucostatic theory (this theory proposed that the rise in plasma glucose concentration after a meal was sensed by "glucoreceptor" neurons in the hypothalamus, which then signalled for meal termination)

policies are a. predetermined goals of management b. statements that provide direction and motivate c. guides that define the scope of permissible activity d. specific tasks to be performed

c. guides that define the scope of permissible activity

the mother of an infant who weighed 4.5kg at birth and was hypoglycemic, may a. have gained excessive amounts of weight during pregnancy b. not have gained enough weight during pregnancy c. have had gestational diabetes d. have reactive hypoglycemia

c. have had gestational diabetes

what may be asked during an interview a. do you own or rent your home b. of what country are you a citizen c. have you worked at the company under a different name d. will you list all organizations, clubs and lodges to which you belong

c. have you worked at the company under a different name

which of the following would be classified as NB in the NCP a. hypermetabolism b. inadequate enteral intake c. impaired ability to prepare foods d. excessive protein intake

c. impaired ability to prepare foods (NB=behavioral-environmental; problems related to knowledge, access to food and food safety; knowledge deficit, harmful beliefs, disordered eating, undesirable food choices, inactivity, excessive exercise, impaired ability to prepare foods, food safety and access)

a client who primarily consumes frozen dinners and breakfast cereal because of limited access to transportation most likely would have a nutrition diagnosis of a. not ready for diet change b. inability to manage self care c. impaired ability to prepare meals d. poor nutrition quality of foods

c. impaired ability to prepare meals

if you are concerned that the receiver is not accurately checking incoming stock, you should a. begin purchasing by bid b. use a prime vendor c. implement blind receiving d. conduct random checks of food cases

c. implement blind receiving

the primary goal of outcomes research in a health care setting is to a. decrease healthcare costs b. increase cost-effectiveness c. improve the quality of services offered d. give the facility a competitive advantage

c. improve the quality of services offered (The intent of this research is to identify shortfalls in practice and to develop strategies to improve care)

a talented catering employee is always requested by customers, but the employee is frequently absent. the manager should a. give the employee multiple warnings b. ignore the employee's absences c. initiate progressive disciplinary procedures d. require a physicians statement after each absence

c. initiate progressive disciplinary procedures

the dietitian at a school foodservice receives several phone calls concerning the risk of cancer to children who consume fresh fruits treated with pesticides. the first action should be a. refer callers to the American cancer society b. remove the suspected fruit from the menu c. investigate the issue and decide on a course of action d. do nothing unless the vendor initiates a recall of the fruit

c. investigate the issue and decide on a course of action

a normal pregnancy requires supplementation of a. iron b. vitamin C, calcium, iron c. iron and folic acid d. calcium and zinc

c. iron and folic acid

a flexible monthly budget would be recommended for a hospital which a. expects no change in expenses or revenue b. expects to be at 90% occupancy c. is planning to close some of its wings for renovation for three months d. has less than 100 employees

c. is planning to close some of its wings for renovation for three months

a patient with hepatitis needs a high protein intake. besides helping the liver to regenerate, what other purpose is served a. it helps increase iron absorption b. it restores glycogen reserves c. it helps prevent a fatty liver d. it helps prevent necrosis of the liver

c. it helps prevent a fatty liver

which of the following best describes a cash flow budget a. it outlines the assets b. it outlines expenditures and assets c. it is an estimate of income and expenditures over time d. it compares assets to liabilities

c. it is an estimate of income and expenditures over time (cash budget: projects revenues and expenses, showing inflow and output of cash; purpose is to determine if funds will be available when needed)

which is true with osteoporosis a. it is more common in men than in women b. it is treated with 500mg calcium per day c. it is common in those 65 years of age and older d. weight bearing exercise exacerbates the problem

c. it is common in those 65 years of age and older

a middle income female needs support for breast feeding. which group will help a. milk bank b. pritikin program c. la leche league d. Feingold program

c. la leche league

all of the following are operational costs except a. utilities b. cleaning c. labor cost d. laundry service

c. labor cost (labor is a semi-variable cost that has both a fixed and variable component; a portion remains fixed regardless of changes in sales volume)

a job breakdown a. can be used for employee appraisal b. studies and analyzes all aspects of the job c. lists what steps to and how to do them d. lists time limits on all aspects of the job

c. lists what steps to do and how to do them

you are assessing a 4'11" female patient who weights 160 pounds. she has hypertension, high cholesterol and diabetes. what do you recommend a. low sodium b. low carbohydrates c. low calories d. low fat

c. low calories

an infant with phenylketonuria requires which of the following dietary alterations a. low phenylalanine, high valine b. high phenylalanine, low tyrosine c. low phenylalanine, high tyrosine d. high isoleucine, low tyrosine

c. low phenylalanine, high tyrosine

a common result of treating an AIDS patient with zidovudine is a. diarrhea b. constipation c. macrocytic anemia d. hypokalemia

c. macrocytic anemia (zidovudine is a NRTI drug which can lead to anemia, loss of appetite, low B12, cooper, zinc and carnitine)

what is the best way to prevent contamination of food a. freeze it b. freeze it in sterile packages c. make employees wash hands frequently d. wash countertops regularly with sanitizing agents

c. make employees wash hands frequently

what should you do if you believe theft is occurring in the storeroom a. install a video camera b. lock the storeroom c. make everyone sign in and out of the storeroom d. limit access to certain hours of the day

c. make everyone sign in and out of the storeroom

a press conference announcing your new business is a. advertising b. networking c. media promotion d. personal selling

c. media promotion

which is a measure of somatic protein a. serum albumin b. serum transferrin c. mid-arm muscle circumference d. triceps skinfold

c. mid-arm muscle circumference (somatic protein is a measure of protein in skeletal muscle)

what is not found in a standardized recipe a. seasonings to be included in the recipe b. time and temperature of cooking c. mixing bowls, measuring spoons d. amounts of ingredients to include

c. mixing bowls, measuring spoons

a primary use of the dietary reference intakes is to a. calculate parenteral nutrition requirements b. assess nutrient intakes of at -risk populations c. plan menus for healthy people d. educate an individual with hyperlipidemia

c. plan menus for healthy people

management's basic function is a. controlling b. directing c. planning d. evaluating

c. planning

the major function of management is a. forecasting b. actuating c. planning d. organizing

c. planning

which of the following may result in hypoalbuminemia a. amino acid supplementation prior to surgery b. a high protein diet with inadequate water c. providing only 5% dextrose in water solution after stress d. giving a trauma patient enteral feeding only

c. providing only 5% dextrose in water solution after stress

appearance, taste and texture are considered a. quantity control b. safety control c. quality control d. production control

c. quality control

which of the following would reduce direct expenses in a foodservice a. increase selling prices b. reduce rent c. reduce food costs d. increase part time labor

c. reduce food costs (direct, variable, flexible costs: vary directly with changes in sales/revenue; directly involved in service to customer; china, silver, food, uniforms, laundry, repairs, benefits)

OSHA requires documentation on the use of the following a. latex gloves b. respiratory masks c. safety goggles d. hairnets

c. safety goggles

an employee consistently calls in sick the day before a holiday. his supervisor should a. discuss this with other managers b. telephone the employee to verify the illness c. schedule a counseling session with him d. file a letter of concern with the employee relations department

c. schedule a counseling session with him

which is most characteristic of marasmus a. serum albumin of 2.5g/dl b. edema c. severe fat and muscle wasting d. decreased lymphocytes

c. severe fat and muscle wasting

how do you convince administration that a former nutrition program should be reinstated a. show that 95% of patients enjoyed the program b. show that 85% of those who attended came to all sessions c. show that 75% of those who attended had fewer hospital stays d. show that only 10% felt the program was not worthwhile

c. show that 75% of those who attended had fewer hospital stays

psychographic marketing includes the following categories a. age, gender, race, education b. urban, suburban, cultural values c. social class, lifestyle, motive behind purchases d. income, education

c. social class, lifestyle, motive behind purchases (Psychographic divides the market into groups based on social class, lifestyle and personality characteristics. It is based on the assumption that the types of products and brands an individual purchases will reflect that persons characteristics and patterns of living)

a pan of au gratin potatoes has been cooked to an endpoint temperature of 140F. serving is scheduled to begin in 10 minutes. you should a. delay service until the potatoes are done b. begin service without the potatoes and deliver them when done c. substitute mashed potatoes until the au gratin are done d. complete the cooking process in the hot holding unit

c. substitute mashed potatoes until the au gratin are done

roman catholic dietary laws state that fish is a. to be served every Friday all year b. to be served on Ash Wednesday and Good Friday c. to be served on Ash Wednesday and Fridays during Lent d. no longer required on Fridays

c. to be served on Ash Wednesdays and Fridays during Lent

Which of the following is true regarding flour? a. all-purpose flour is made from soft wheat b. all-purpose flour has more protein than bread flour c. cake flour has the least and the weakest gluten d. cake flour has less starch than bread flour

cake flour has the least and the weakest gluten

Which of the following has a slow onset and is anaerobic? a. staphylococcus aureus b. clostridium perfringens c. clostridium botulinum d. salmonella

clostridium botulinum

299. Which of the following is an example of the Hawthorne effect? [NTR:593] a) "Chloe has always thought that she caught fewer colds when she remembered to take her zinc supplements, so she is pleased to be in a study now to determine the impact of zinc supplements on one's susceptibility to colds. She is sure that the supplement she is taking is helping her avoid catching colds." b) "Pat, a researcher for a number of clinical trials, has reviewed preliminary results from a study of the effects of caffeine on sleeplessness. The data indicate that the findings may be inconclusive so Pat has asked that the interview questions be revised to make it possible to identify what may be more subtle effects of caffeine." c) "Emma is participating in a study of the impact of artificially sweetened cold drinks on appetite. She believes that she is in the control group so she plans to report that the beverage she drinks does not affect her appetite." d) "Because Raul is in a study to determine the impact of eating breakfast on one's ability to achieve faster times in track and field events that include 'sprinting' periods, he is skipping breakfast less often."

d) "Because Raul is in a study to determine the impact of eating breakfast on one's ability to achieve faster times in track and field events that include 'sprinting' periods, he is skipping breakfast less often."

115. The local WIC nutritionist has collected considerable data about pregnant women and infants in her community. Which of the following would be correctly categorized as qualitative data? [NTR:509] a) The number of live births per 1000 deliveries b) The mean age of women with documented cases of gestational diabetes c) A graph of the gestational ages and birth weights of all infants born alive d) A tally that notes the reasons that pregnant women report for omitting dairy products from their diets

d) A tally that notes the reasons that pregnant women report for omitting dairy products from their diets

66. Which of the following disease states may require a fluid restriction? [NTR:461] a) Liver disease with ascites b) Chronic renal failure c) Congestive heart failure d) All of the above

d) All of the above

7. Your patient is a newly diagnosed type 2 diabetic who tells you she is relieved to only have a touch of sugar and will avoid dessert when she goes out for a big meal after leaving the hospital. Which lab results will you review with your patient? [NTR:402] a) Glucose and hemoglobin A1c b) Triglycerides c) Cholesterol panel including HDL, LDL, VLDL d) All of the above.

d) All of the above.

356. If the carbohydrate content of the diet is insufficient to meet the body's needs for energy, which of the following can be converted to glucose? [NTR:651] a) Acetyl-CoA b) Fatty acids c) Carbon dioxide d) Amino acids

d) Amino acids

354. Which of the following enzymes acts on complex carbohydrates? [NTR:649] a) Lactase b) Sucrase c) Maltase d) Amylase

d) Amylase

274. Which of the following statements is true regarding the potential benefits of dietary fiber? [NTR:568 a) Lower LDL cholesterol levels b) Reduce postprandial blood glucose levels c) Increased gastric emptying d) Both A and B

d) Both A and B

336. Which of the following statements best describes the similarities between the Special Supplemental Nutrition Program for Women, Infants, and Children and the Supplemental Nutrition Assistance Program? [NTR:630] a) Both programs are regulated by the USDA, provide referrals to health care and other social services providers, and are entitlement programs. b) Both programs are regulated by the USDA, contain income eligibility guidelines, and are entitlement programs. c) Both programs are regulated by the USDA, contain income eligibility guidelines, are entitlement programs, and recipients must have a documented medical or nutritional risk. d) Both programs are regulated by the USDA and contain income eligibility guidelines.

d) Both programs are regulated by the USDA and contain income eligibility guidelines.

357. The presence of fat in the intestine stimulates cells to release: [NTR:652] a) Gastrin b) Secretin c) Insulin d) Cholecystokinin (CCK)

d) Cholecystokinin (CCK)

337. Which of the following examples of a snack provided at a child care center would meet reimbursement guidelines and comply with USDA regulations for Child and Adult Care Food Program meals? [NTR:631] a) Carrot sticks and 100% fruit juice b) Oatmeal, corn muffins, and water c) Apple slices and orange wedges d) Cottage cheese, mandarin oranges, and water

d) Cottage cheese, mandarin oranges, and water

329. What organization is working to establish a standard definition of what makes an organization or institution culturally competent? [NTR:623] a) The American Dietetic Association b) US Dept. of Education c) US Dept. of Health and Human Services d) Joint Commission on Accreditation of Healthcare Organizations

d) Joint Commission on Accreditation of Healthcare Organizations

227. Which of the following adolescents is at greatest nutrition risk? [NTR:521] a) One who watches more than 2 hours of television per day. b) One who consumes more than three servings of fruit per day. c) One who engages in vigorous physical activity five or more times per week. d) One who consumes fast food three or more times per week.

d) One who consumes fast food three or more times per week.

269. Which of the following is an appropriate method to prevent aspiration during enteral feeding? [NTR:563] a) Use a fiber-containing formula b) Elevate the head of the bed >15 degrees c) Place a percutaneous gastrostomy d) Place a nasojejunal tube

d) Place a nasojejunal tube

260. Which of the following is true of Mexican American culture? [NTR:554] a) In general, it is better to ask questions about work than about family. b) It is generally preferable to address elders and leaders informally. c) Nodding affirmatively usually means agreement. d) Polite, agreeable, and respectful interactions are the cornerstone of the culture.

d) Polite, agreeable, and respectful interactions are the cornerstone of the culture.

340. Which of the following foods is the best source of omega-3 fatty acids? [NTR:634] a) Olive oil b) Shrimp c) Avocado d) Sardines

d) Sardines

16. What does the Nutrition Care Process provide to facilitate more effective comparison of nutrition assessment findings? [NTR:411] a) Research opportunities b) PES statements c) Information sharing technologies d) Standardized language

d) Standardized language

118. Which is the best tool for the community nutritionist to use to learn what locally grown foods residents consume most frequently? [NTR:512] a) Health Risk Appraisal b) Screening c) Focus Group d) Survey

d) Survey

8. What government funding supports the school lunch program? [NTR:403] a) Medicaid b) Health and Human Services c) CACFP d) USDA

d) USDA

117. Which of the following describes a limitation to a 24-hour recall? [NTR:511] a) Not useful if diet pattern varies considerably b) Requires literacy d) Recording process may influence food intake d) Underreporting or over reporting occurs

d) Underreporting or over reporting occurs

5. Which of the following is not a risk determinant of metabolic syndrome? [NTR:400] a) Waist circumference b) Blood pressure c) Triglycerides d) Weight

d) Weight

52. When performing nutritional screening of pediatric patients, which of the following on the Centers for Disease Control and Prevention growth chart would identify a patient at risk? [NTR:447] a) Body Mass Index of 80th percentile b) Length/height for age of 75th percentile c) Weight for age of 25th percentile d) Weight for length/height less than 3rd percentile

d) Weight for length/height less than 3rd percentile

273. The ethical principle of autonomy is supported when [NTR:567] a) a clinician treats all patients with similar attention. b) explaining the benefits and risks of enteral nutrition to the wife of a man in a persistent vegetative state. c) enteral nutrition is provided to a postoperative patient requiring mechanical ventilation. d) a patient at end of life is allowed to refuse parenteral nutrition.

d) a patient at end of life is allowed to refuse parenteral nutrition.

26. Symptoms associated with hypersensitivity to the ingestion of a food may stem from: [NTR:421] a) chemical contaminants. b) microbiological contaminants. c) parasitic contaminants. d) all of the above.

d) all of the above.

35. Data reviewed for nutrition assessment is also reviewed during the following step: [NTR:430] a) monitoring and evaluation. b) nutrition diagnosis. c) goal setting. d) all of the above.

d) all of the above.

96. Critical nutrition concerns in children ages 2-11 years of age include: [NTR:490] a) excessive intakes of dietary fat, especially saturated fats. b) decreased intakes of foods that are good sources of calcium, fiber, iron, vitamin E, folate, magnesium and potassium. c) sedentary lifestyles. d) all of the above.

d) all of the above.

309. Which of the components of total energy expenditure accounts for the largest percentage of total energy expenditure? [NTR:603] a) thermic effect of food b) physical activity c) non-exercise activity thermogenesis d) basal metabolism

d) basal metabolism

275. Two fats that are high in saturated fat and solid at room temperature include: [NTR:569] a) coconut oil and safflower oil. b) corn oil and palm kernel oil. c) soybean oil and palm oil. d) coconut oil and palm kernel oil.

d) coconut oil and palm kernel oil.

Quality in the foodservice system is [MGT:314] a) a program to eliminate defects. b) a radical redesign of business processes for dramatic improvement. c) a source of increasing costs. d) defined by the customer through his or her satisfaction.

d) defined by the customer through his or her satisfaction.

88. The Academy of Nutrition and Dietetics takes the position that pediatric overweight intervention programs require a combination of which of the following programs: [NTR:483] a) school-based multi-component programs and community-based interventions. b) family-based multi-component programs and environmental interventions. c) community-based interventions and environmental interventions. d) family-based and school-based multi-component programs.

d) family-based and school-based multi-component programs.

43. A child's weight is plotted at the 50th percentile on the growth chart. This means that the child's weight is: [NTR:438] a) greater than or equal to that of half the children of the same gender and ethnicity. b) greater than or equal to that of half the children of the same age. c) greater than or equal to that of half the children of the same ethnicity. d) greater than or equal to that of half the children of the same age and gender.

d) greater than or equal to that of half the children of the same age and gender.

250. The first step in conducting a community needs assessment is to [NTR:544] a) describe the problem and state why a needs assessment is necessary b) specify the data needed and design a plan for acquiring it. c) determine the purpose, goals, and objectives of the needs assessment. d) identify the target population and nutritional problem of concern.

d) identify the target population and nutritional problem of concern.

106. If the community nutritionist is concerned that the new educational activities provided for SNAP recipients have been ineffective in increasing the amount of fresh vegetables consumed, the best approach would be to conduct a(n) ______ evaluation [NTR:500] a) efficiency b) process c) structure d) impact

d) impact

331. The dominant force in how a patient views health care is likely to be ______. [NTR:625] a) social hierarchy b) race d) individual experience d) ethnicity

d) individual experience

79. In the Nutrition Care Process, it is expected that all Nutrition Diagnoses will fall within one of three domains - clinical, behavioral-environmental, and _____. [NTR:474] a) Social b) ethnic/cultural c) anthropometric d) intake

d) intake

322. The pathway of glucose to pyruvate during anaerobic conditions results in [NTR:616] a) acetyl CoA. b) carbon dioxide. c) fatty acids. d) lactic acids.

d) lactic acids.

300. Wheat does not contain enough lysine for humans to synthesize proteins from wheat alone. Thus, in wheat, lysine is considered to be a(n) _____. [NTR:594] a) incomplete protein b) inadequate protein c) nonessential protein d) limiting amino acid

d) limiting amino acid

237. Hyperosmolar hyperglycemic state, a rare condition that presents with extremely high blood glucose levels, elevated serum osmolality, extreme dehydration, and low amounts of ketones, usually occurs in [NTR:531] a) preterm infants. b) obese adolescents. c) adults ages 18 to 35 with hypertension. d) persons older than 65 with type 2 diabetes mellitus.

d) persons older than 65 with type 2 diabetes mellitus.

263. Cultural _____ is the ability to collect culturally relevant information from clients and perform culturally-based assessments and interventions. [NTR:557] a) Knowledge b) Awareness c) Desire d) skill

d) skill

67. The Nutrition Care Process includes: [NTR:462] a) recommended goals. b) required courses of treatment. c) consistent expected outcomes. d) standardized language.

d) standardized language.

27. A diagnosis of hypersensitivity to food requires [NTR:422] a) conducting a food challenge without restricting the suspected food. b) documented atopic dermatitis. c) using an elimination diet for 3 weeks. d) verifying the food that causes an adverse reaction and excluding other causes of adverse reactions.

d) verifying the food that causes an adverse reaction and excluding other causes of adverse reactions.

49. The Centers for Disease Control and Prevention growth chart that is used to determine if a child is underweight or overweight is [NTR:444] a) height for age. b) length for age. c) weight for age. d) weight for stature.

d) weight for stature.

248. The group of people that best exemplifies the term 'target population" are those [NTR: 542] a) with similar nutrition and fitness goals b) with common characteristics or a common bond c) who will conduct a community needs assessment d) who are the focus of a community needs assessment

d) who are the focus of a community needs assessment

264. In the Cajun and Creole culture, the traiteur is the one _____. [NTR:558] a) who provides valued health and wellness information b) who is considered the most revered member of the community c) who blesses the food before it is consumed d) who believes he or she possesses a divine gift for healing

d) who believes he or she possesses a divine gift for healing

the raw food cost of an item is $1.12 per serving. what is the hidden cost a. $.10 b. $.07 c. $.13 d $.11

d. $.11 (hidden cost is often calculated at 10% of raw food cost)

using the factor method, determine the selling price using the data below raw food cost: $3.14; prepared food cost: $5.20; labor cost: $3.39; overhead cost: $1.12; factor: 3.37 a. $6.40 b. $7.84 c. $8.10 d. $10.58

d. $10.58 (raw food cost 3.14 x 3.37 factor = 10.58)

DRGs allow $1600 for an appendectomy with a 4 day stay. the patient is released in 3 days. the hospital will receive a. $1200 b. $800 c. $2000 d. $1600

d. $1600

a young girl who is HIV positive with Karposi's sarcoma tells you she would like to take vitamin C supplements. what should you tell her a. ignore her request and talk about incorporating better balanced foods into her diet b. ignore her request and advise her to drink more juices c. explain the dangers of taking too much vitamin C d. ask her to keep a food diary to see where she is lacking

d. ask her to keep a food diary to see where she is lacking

a diet aide was injured in a car accident and is still nto able to perform a majority of her duties after a period of recuperation. to comply with the americans with disabilities act, the manager should a. create a new position that the employee can perform b. transfer the employee to another department c. hire another employee to assist the diet aide d. ask the employee to provide written recommendations from their physician

d. ask the employee to provide written recommendations from their physician

a food service worker confides in you that he has tested HIV positive. what should you do a. ask him to report to the health clinic b. instruct him to wear gloves on duty c. reassign him to another area of the kitchen d. assure him that confidentiality will be respected

d. assure him that confidentiality will be respected

a manager asks for input and advice from several employees on a situation. he then makes a decision which may or may not reflect the input provided by the employees. he is using what leadership decision making style a. democratic b. autocratic c. group d. consultative

d. consultative (asks for input but makes final decision alone)

a patient is attempting to lower blood pressure levels with dietary intervention. the dietitian should monitor his blood pressure and a. body weight and medication b. daily food intake and medication c. daily food intake and heart rate d. daily food intake and body weight

d. daily food intake and body weight

the primary focus of nutrition care for a man with hypertension and a BMI of 32 should be to a. decrease sodium intake b. increase potassium intake c. increase calcium intake d. decrease body weight

d. decrease body weight

a tube fed patient receiving bolus feedings develops a distended abdomen. what should you recommend a. change the tube feeding to isotonic, lactose free b. decrease the osmolality of the feeding c. decrease the amount of water flush d. decrease the rate

d. decrease the rate (rapid infusion can cause influx of water into gut - decreasing rate allows gut to fill slower and gives time to empty)

which of the following dish machine temperatures should be adjusted pre rinse 121F Wash 145F Final rinse 205F a. increase the temperature of the pre rinse b. increase the temperature of the wash cycle c. decrease the temperature of the wash cycle d. decrease the temperature of the rinse cycle

d. decrease the temperature of the rinse cycle

the Y theory of management is a. authoritarian b. autocratic c. based on managing tasks d. democratic

d. democratic (Theory Y: work is as natural as play or rest; management should arrange conditions so workers can achieve goals by directing own efforts; positive, participative)

an employee becomes loud and disruptive and refuses to comply with the dietitians request concerning nourishment preparation. what should the dietitian do a. attempt to calm the employee while meeting nourishment preparation deadlines b. take the employee aside and determine whether insubordination can be established c. ignore the behavior d. designate a replacement and remove the employee from the area for further discussion

d. designate a replacement and remove the employee from the area for further discussion

which of the following outcomes would best indicate parental support of a child's weight loss effort in a program for obese pre-teen children a. eliminate sweets b. discuss weight loss indicators c. decide to defer action until after puberty d. enroll child in physical activity programs

d. enroll child in physical activity programs

immediately following a severe burn, nutritional therapy should include a. high calories b. high protein c. high calories, vitamins, minerals d. fluid and electrolyte replacement

d. fluid and electrolyte replacement

if you open sealed bids and place the order with the lowest bid, you are using which buying method a. informal b. open market c. future contracts d. formal, competitive bids

d. formal, competitive bids

a patient needs a 2000 calorie, 2 gram sodium feeding. which formula is best calories/cc g Na/liter a. formula 1 1 2 b. formula 2 1.5 2 c. formula 3 2 1 d. formula 4 1 1

d. formula 4 1kcal/cc and 1g Na/liter

bacillus cereus intoxication is most likely to occur following a. macaroni and cheese b. baked beans c. corn pudding d. fried rice

d. fried rice (found in cereal crops such as rice products, starchy foods, casseroles, sauces, puddings, soups, pastries; can be emetic or diarrheal; lasts up to 24hours and onset of 30-6hr (emetic) 6-15hr (diarrheal)

a patient on parenteral nutrition is to be transitioned to enteral feeding. he should receive a. half-strength enteral feeding intitially b. half-strength, then 3/4 strength enteral feeding c. half-strength enteral feeding with a low osmolality d. full strength enteral feeding at a low rate

d. full strength enteral feeding at a low rate

which piece of food service equipment may be restricted by a local law a. toaster b. steam jacketed kettle c. convection oven d. garbage disposal

d. garbage disposal

you have developed a website to market a new business. which of the following components is a form of promotion a. online order form b. price and availability of products c. email address of the business d. graphics and text of products

d. graphics and text of products

what is the best way to increase attendance at a nutrition seminar for dentists a. hold it in a location convenient to large numbers of dentists b. have it held in a medical facility c. hold it on weekends when their offices are likely closed d. have it sponsored by a dental organization

d. have it sponsored by a dental organization

an appropriate measurement for iron status in a local community is a. serum iron b. transferrin c. TIBC d. hematocrit

d. hematocrit

Aseptically packaged milk should be stored: a. in dry storage b. in the refrigerator c. in the freezer d. below 40 degrees

in dry storage

the symbol U on a food label indicates a. it is treated with ultraviolet light b. it is treated with radiation c. vitamin retention has been retained during processing d. it is a Kosher designation

d. it is a Kosher designation

a child is below the 5th percentile weight for height. what can be said about the child a. the child is tall for their age b. the child has a disease that inhibits growth c. it is an indication of long term under-nutrition d. it is an indication of recent under-nutrition

d. it is an indication of recent under-nutrition

who resolves differences in bills from the House and the Senate a. House committee b. Senate committee c. committee on resolutions d. joint conference committee of the House and Senate

d. joint conference committee of the house and senate

in community surveys, a frequent symptom of malnutrition is a. scurvy b. rickets c. low serum albumin d. low hemoglobin

d. low hemoglobin

what is the best approach for controlling obesity in an otherwise healthy ten year old girl a. reduction of caloric intake to allow weight loss of 1-2 pounds weekly b. reduction of energy intake to no less than 1000cal/day c. avoidance of simple sugars and sugar containing products d. maintenance of weight during continued growth

d. maintenance of weight during continued growth

employees are consistently late returning from breaks. the dietitian should first a. have all employees clock in and out for breaks b. analyze activities to identify the best break times c. discipline tardy employees the next time they are late returning from breaks d. meet with all employees as a group and solicit recommendations for improvement

d. meet with all employees as a group and solicit recommendations for improvement

a patient with Parkinson's disease on levodopa should a. increase protein intake at breakfast b. consume no more than 40 grams protein per day c. evenly divide protein intake throughout the day d. minimize protein at breakfast and lunch and add it to the evening meal

d. minimize protein at breakfast and lunch and add it to the evening meal

theory Z is based on the following premise a. workers accept responsibility b. self-direction and control are emphasized c. task, structure and authority d. mutual worker-employer loyalty

d. mutual worker-employer loyalty (Ouchi - the value of the company is the people; everyone who will be affected by a decision is involved in making the decision; consensus decision-making)

a 2 year old child with iron deficiency anemia has anorexia, vomiting, weight loss and restlessness. look for a. excessive milk and fruit juice intake b. excessive vitamin C and D intake c. asthma and respiratory distress d. pica and lead poisoning

d. pica and lead poisoning

a nephrotic syndrome patient with edema has a serum albumin level that continues to fall. recognize that a. liver dysfunction is causing decreased albumin synthesis b. volume overload has diluted the albumin c. the patient is getting sicker and eating less d. protein is being lost in the urine

d. protein is being lost in the urine (collections of symptoms that indicate kidney damage; results in proteinuria, hyperlipidemia, edema, hypoalbuminia)

which would you recommend for a patient with mild dysphagia a. honey b. apple juice c. nectar d. pudding

d. pudding

to determine how profitable a department has been, determine the profit margin, which is a. ratio of income to revenue b. ratio of costs to income c. ratio of overhead to sales d. ratio of profit to sales

d. ratio of profit to sales

in making decisions, take the following steps a. gather data, choose solution, take action, follow-up b. determine workable solutions, choose one, take action c. choose solution, take action, follow-up d. recognize and analyze the problem, gather data, choose solution, take action, follow-up

d. recognize and analyze the problem, gather data, choose solution, take action, follow-up

which of the following is an achievable outcome for a weight loss program a. promote desirable body weight among young adults b. increase by 25% the number of physicians who refer patients for weight-management counseling c. screen and refer for treatment within two years those whose body weight is 10% above desirable d. reduce by 10% the number of obese adults, ages 25-49, within two years of starting the program

d. reduce by 10% the number of obese adults, ages 25-49, within two years of starting the program

the best way to justify continuing a wellness program would be to demonstrate that the participants a. lost weight b. recommended the program to others c. increased their intake of protein d. reduced their health risk scores

d. reduced their health risk scores

which of the following strategies is most likely to increase revenue contribution margin low high Sales low pot roast beef tacos high barbecued ribs beef stew a. replace the beef tacos b. replace the beef stew c. offer pot roast more often d. replace the pot roast

d. replace the pot roast

intervention by a dietitian conducting nutrition assessment is best determined by a. a pateints medical history b. a patients diagnosis c. the case mix of the patient population d. results of nutrition screening

d. results of nutrition screening

after an outbreak of food borne illness in the cafeteria, you discover one of your employees has an infected cut on his finger. you should a. call the state public health office to investigate b. have the lab culture the kitchen c. fire the employee d. send the employee home until the cut has healed

d. send the employee home until the cut has healed

which of the following lab values would most likely indicate that dialysis is necessary serum creatinine mg/dl Bun mg/dl a. 2.0 80 b. 3.0 90 c. 4.0 100 d. 6.0 70

d. serum creatinine of 6.0 and BUN of 70 (BUN:creatinine ratio of <10:1 suggests reduced BUN reabsorption due to renal damage and possible need for dialysis)

an employee voted in by the union who works to settle grievances while on paid time is the a. mediator b. arbitrator c. conciliator d. shop steward

d. shop steward

your patient is on Coumadin. which foods would you make sure to consistently include in their diet a. oranges, peaches b. sweet potatoes c. beef, lamb d. spinach, collard greens

d. spinach, collard greens

the FDA is responsible for a. pesticide residue tolerance levels b. standard of fill, assuring meat wholesomeness, nutrition labels c. grading of meat d. standard of identity, nutrition labels, imitation foods

d. standard of identity, nutrition labels, imitation foods

you called a meeting for 9am. three out of the five people arrived on time. what is the general guideline for starting meetings a. wait until all participants have arrived b. wait 10 minutes past the stated starting time c. wait until the most significant members have arrived d. start on time

d. start on time

a disgruntled employee is loud and disruptive and appears to be out control even after the dietitian has counseled him. what should the dietitian do a. send the employee to the break room to calm down b. return to the office to write a disciplinary report c. send the employee off the premises immediately and alert security d. take the employee into the office and discuss the consequences of his behavior

d. take the employee into the office and discuss the consequences of his behavior

gauge refers to a. ply b. finish c. polish d. thickness

d. thickness

two of the listed ingredients on a frozen food label are BHA and BHT. what is their function a. flavor enhancers b. emulsifiers c. to prevent enzymatic oxidation d. to prevent lipid oxidation

d. to prevent lipid oxidation

which of the following formulas is used to determine a meal equivalent in a commercial operation a. average of a prototype meal b. average dollars spent per patron over a one week period c. total sales divided by budgeted cost per meal d. total revenues for a specific period divided by the cost of a prototype meal

d. total revenues for a specific period divided by the cost of a prototype meal

which of the following is poor management a. one supervisor for one employee b. one supervisor for every two employees c. one supervisor for the entire department d. two supervisors for one employee

d. two supervisors for one employee

a 11 month old infant is at the 95th percentile weight for age and has been for the past 9 months. what should you do a. recommend increasing food intake by 25% b. recommend decreasing food intake by 50% c. nothing, he is fine d. watch his growth over the next six months

d. watch his growth over the next six months

athletes should receive a. salt pills only during the summer months b. extra protein c. carbohydrate loading two weeks prior to an event d. water during an event to replace lost fluids

d. water during an event to replace lost fluids

if you have a lactose intolerance, which food could you most likely tolerate a. whole milk b. skim milk c. ice cream d. yogurt

d. yogurt

Pyridoxine acts as a coenzyme in: a. iron transfer b. deamination and transamination c. fat transfer through the lymphatic system d. osmotic pressure of body fluids

deamination and transamination

Adding bran to a bread recipe will: a. increase the volume b. decrease the volume c. require an increase in oven temperature d. require a decrease in oven temperature

decrease the volume

An amylase deficiency would result in: a. decreased gluconeogenesis b. decreased lipolysis c. decreased glycolysis d. decreased proteolysis

decreased glycolysis

Which of the following may result in metabolic acidosis? a. decreased retention of bicarbonate b. decreased retention of hydrogen c. decreased retention of chloride d. increased retention of bicarbonate

decreased retention of bicarbonate

What usually occurs when iso-caloric amounts of omega 3 fatty acids are substituted for saturated fatty acids? a. increased HDL, reduced LDL b. decreased triglycerides, lower total cholesterol c. decreased triglycerides, little change in total cholesterol d. lower HDL, increased LDL

decreased triglycerides, little change in total cholesterol

Which is the best way to teach food service workers to prepare a full liquid diet: a. video and discussion b. prepare it themselves c. written instruction d. demonstration, then have them prepare it

demonstration, then have them prepare it

Vitamin A is involved in: a. growth and development b. blood clotting c. growth of bones and teeth d. development of tissue and skin

development of tissue and skin

An example of nominal (frequency) data is: a. temperature b. weight c. differentiate between men and women d. rank in class

differentiate between men and women

Which of the following is considered an output of a foodservice system? [MGT:377] a. Distribution and service. b. Policies and procedures c. Employee satisfaction d. Menu

employee satisfaction

Having the same or similar outputs result from different inputs is called: [MGT:372] a. dynamic equilibrium. b. equifinality. c. Synergy d. interdependency

equifinality (is the principle that in open systems a given end state can be reached by many potential means)

A cake with a fallen center may be the result of: a. too little fat b. too little sugar c. excess sugar and excess fat d. too little baking powder

excess sugar and excess fat

An example of upward communication in an organization is a(n): [MGT:334] a.exit interview b. newsletter c. pay insert d. policy manual

exit interview

Which of the following study designs can best determine the EFFECT of promotional techniques on cafeteria sales? a. experimental b. observational c. survey d. segmentation

experimental

You are asked to evaluate a new harbal supplement that you know is not healthy. You should: a. discredit the manufacturers of the product b. because of liability laws, you cannot say anything c. explain why it is not sound and give alternatives d. do nothing

explain why it is not sound and give alternatives

Searching for external trends and information that may impact the foodservice system is called a(n) [MGT:380] a. environmental scan. b. strategic direction. c. external feedback. d. strategic focus.

external feedback.

The absence of reinforcement following an undesirable behavior is: a. positive reinforcement b. punishment c. extinction d. avoidance

extinction

an infant formula for lactose intolerance might include a. whey, coconut oil b. casein hydrolysate, corn oil, glucose c. soy protein, corn syrup solids d. sodium caseinate, hydrolyzed cornstarch e. a, b, c f. b, c g. b h. a, c

f. b, c (casein hydrolysate, corn oil, glucose and soy protein, corn syrup solids)

Ketones are most associated with: a. complex carbohydrates b. simple sugars c. fats d. amino acids

fats

Biotin is considered a coenzyme in the synthesis of: a. protein b. carbohydrate c. glycogen d. fatty acids

fatty acids

Food iron is in the _____ form. a. ferric b. ferritin c. transferrin d. ferrous

ferric

The best laboratory value to use in assessing iron status is: a. hemoglobin b. hematocrit c. ferritin d. serum iron

ferritin

Food iron is reduced in the stomach to the more absorbable form of: a. ferric b. ferrous c. transferrin d. ferritin

ferrous

At your first counseling session with a new patient you should: a. tell him you have to change his diet b. find out what he eats c. give him diet materials d. teach him meal exchanges

find out what he eats

An edematous person may actually be dehydrated because: a. intravascular fluid is retained b. intravascular fluid is diluted c. fluid is retained intracellularly d. fluid is retained extracellularly

fluid is retained extracellularly

ABC Hospital Food and Nutrition Department prepares and delivers meals to other healthcare operations in a large metropolitan area. Other hospital foodservices in the metropolitan area have recently begun offering meal delivery service. ABC Hospital modified its service to offer home meal replacements to the healthcare staff in addition to its' other meal delivery service. ABC Hospital is using [MGT:381] a. strategic analysis b. competitive advantage. c. focus strategy. d. strategic implementation.

focus strategy.

Research has shown that neural tube defects can be reduced with proper supplementation of: a. iron b. folic acid c. glutamine d. arginine

folic acid

Carbohydrates listed in order of decreasing sweetness are: a. fructose, glucose, sucrose, sorbitol b. sucrose, glucose, fructose, sorbitol c. fructose, sucrose, glucose, sorbitol d. sorbitol, fructose, sucrose, glucose

fructose, sucrose, glucose, sorbitol

a patient with Crohn's disease needs the following diet a. high fat, high calorie, high protein b. low fat only with steatorrhea, vitamin C, B12 c. high calorie, high protein according to current BMI d. high residue, low fat, fat soluble vitamins e. low calorie, low residue, fat soluble vitamins, B12 f. a only g. b and c h. c and d I. e only

g. b and c (low fat only with steatorrhea, vitamin C, B12 and high calorie, high protein according to current BMI) Crohn's affects terminal ileum leads to B12 deficiency, energy needs according to current BMI, limit fat only with steatorrhea, water and fat soluble vitmains

A group of rates being fed gelatin as the sole protein source are not growing well. What is the likely reason? a. gelatin has no tryptophan and is low in methionine and lysine b. gelatin has no methionine and is low in tryptophan c. gelatin has no lysine or methionine, and is low in tryptophan d. gelatin has no arginine

gelatin has no tryptophan and is low in methionine and lysine

which of the following may be the cause of loose stools in a tube fed patient a. feeding was administered at too rapid a rate b. the osmolality of the feeding was too low c. the patient is constipated d. an inadequate amount of intestinal surface area e. a f. a and b g. b and c h. a and d

h. a and d (feeding was administered at too rapid a rate and an inadequate amount of intestinal surface area)

what is included in the study of demographics a. age of individuals b. population statistics c. sex of individuals d. socioeconomic factors e. a only f. all of the above g. b and c h. a, b and c

h. a, b and c (age of individuals, population statistics and sex of individuals) demographics is assessment information focused on population by age, ethnic groups, sex, birth rates and deaths

the dumping syndrome a. follows a gastrectomy b. is secondary to carbohydrate overload c. draws fluid into the intestine d. needs a high carbohydrate diet e. a, c, d f. b, c, d g. a only h. a, b, c

h. a, b, c (follows a gastrectomy, is secondary to carbohydrate overload and draws fluid into the intestine

Apples stored in a controlled environment would: a. ripen quickly b. last one to two years c. have extended life and improved marketing quality d. only be used as processed fruit

have extended life and improved marketing quality

What is the best way to assess what a group of students has learned after diet instructions? a. list their skills b. have them explain their skills c. have them demonstrate their skills c. post-test on their skills

have them demonstrate their skills

What may cause EKG abnormalities? a. high serum potassium level b. low serum potassium level c. low serum sodium level d. high serum sodium level

high serum potassium level

Commercial and onsite foodservice operations share many similarities in their subsystems. However, some characteristics are unique to a type of operation. A commercial buffet restaurant and a school nutrition program are distinguished by [MGT:375] a. how food is distributed and served. b. how food is procured. c. how sanitation is maintained. d. how food is produced.

how food is distributed and served.

When planning a program for rural elderly, which is the most important? a. the number of participants b. what you will serve c. where it will be held d. how they will get to you

how they will get to you

The first step in evaluating the effectiveness of a series of nutrition education programs would be to: a. identify the objectives of each of the program b. ask all participants to complete 24 hour recalls c. administer nutrition-knowledge tests to all d. verify attendance at each of the programs

identify the objectives of each of the program

Cabbage should be cooked: a. covered b. in a small amount of water c. in a large quantity of water d. covered in a small amount of water

in a large quantity of water

Decreased levels of serotonin are associated with: a. decrease in carbohydrate appetite b. increase in carbohydrate appetite c. decreased sensation of taste d. increased sensation of taste

increase in carbohydrate appetite

When adding bran to a flour mixture, what should be changed? a. baking temperature b. increase the fat and liquid c. increase the oxidizing agent d. increase the flour and liquid

increase the flour and liquid

Statistics that let you draw probable conclusions beyond your immediate universe of data, are called: a. descriptive statistics b. inferential statistics c. variable interest statistics d. continuous statistics

inferential statistics

In cereal production, enrichment adds: a. vitamin A b. vitamin D c. iron d. iodine

iron

What is the main purpose of water in the body? a. it acts as a medium for cell metabolism b. it helps transport calcium through cell membranes c. it maintains electrolyte balance d. to maintain body temperature

it acts as a medium for cell metabolism

A child is lethargic and irritable with diarrhea. The cause may be: a. zinc toxicity b. lead poisoning c. iron overload d. vitamin A toxicity

lead poisoning

If broccoli in a vegetable mixture turns olive green during holding, it may be due to: a. lemon juice b. baking soda c. salt d. vinegar

lemon juice

Sorbitol is: a. less sweet than sucrose and has a lower glycemic index than sucrose b. less sweet than sucrose and has a higher glycemic index than sucrose c. sweeter than sucrose d. sweeter than sucrose and has a lower glycemic response than sucrose

less sweet than sucrose and has a lower glycemic index than sucrose

The rate of oxidation of which branched-chain amino acid increases significantly during moderate exercise? a. leucine b. phenylalanine c. tyrosine d. threonine

leucine

Protein-bound iodine (PBI) measures: a. nutritional status b. toxic levels of iodine c. deficiency of epinephrine d. level of thyroxine being produced

level of thyroxine being produced

Which conversion requires glucose-6-phosphatase? a. glucose into liver glycogen b. glucose into pyruvic acid c. pyruvic acid into lactic acid d. liver glycogen into glucose

liver glycogen into glucose

A study reported a 10% morbidity rate in infants in the country and a 12% morbidity rate for infants in the city. This means that: a. 10% of infants died in the country b. more infants died in the city than i the country c. more disease in infants in the city than in the country d. more disease in infants in the country than in the ctiy

more disease in infants in the city than in the country

As operational costs increase, so has the value of management skills. The person who can effectively manage resources across subsystems is considered a [MGT:368] a. multidepartment manager. b. systems thinker. c. onsite manager. d. strategic manager.

multidepartment manager.

If there is no statistical difference between study groups, the hypothesis is: a. negative b. null c. even d. positive

null

Prevention of an outbreak of Listeria monocytogenes requires special care because: a. it is insensitive to light b. it is sensitive to low temperature c. of its ability to grow at temps of 34 - 113 'F d. of its ability to grow at temps of 140 - 160 'F

of its ability to grow at temps of 34 - 113 'F

Which would aid the absorption of an iron supplement? a. cow's milk, eggs b. orange juice, hamburger c. milk, eggs, cheese d, dates, eggs

orange juice, hamburger

Strategic management of a foodservice system means [MGT:383] a. effectively responding to environmental threats. b. integrated systems thinking of internal and external events. c. evaluation of stakeholders and the competition. d. organization and environmental analysis to achieve goals.

organization and environmental analysis to achieve goals.

The scientific process by which water makes lettuce crisp is: a. osmotic pressure of water-filled vacuoles b. diffusion c. transamination d. water active diffusion

osmotic pressure of water-filled vacuoles

Active listening in counseling sessions involves: a. probing into specific area of behavior b. empathizing c. selective perception d. paraphrasing and responding

paraphrasing and responding

To determine the effectiveness of an eighth grade nutrition program to be offered through a city: a. first determine how many schools are willing to offer the program b. pilot test the program in one school c. request a review of the program by local dietitians d. offer an incentive for schools to provide the program

pilot test the program in one school

When developing educational plans, when do you plan the evaluation criteria? a. when all else is complete b. do it before you plan anything else c. plan evaluation criteria at each step of the process d. plan it just before you offer the pogram

plan evaluation criteria at each step of the process

What is a function of cream of tarter in a recipe? a. prevents the Maillard reaction b. enhances the Maillard reaction c. increases the alkalinity allowing for better leavening d. enhances gelation

prevents the Maillard reaction

Ascorbic acid aids in healing. Which of the following applies? a. collagen -> hydroxyproline b. proline -> hydroxyproline -> collagen c. hydroxyproline -> proline -> collagen d. hydroxyproline -> collagen

proline -> hydroxyproline -> collagen

The lecture: a. increases teacher-student involvement b. is student-oriented c. reinforces material presented d. provides the most material covered in a short time

provides the most material covered in a short time

If protein intake is doubled, which nutrient needs to be increased? a. pyridoxine b. folic acid c. niacin d. vitamin C

pyridoxine

A three day recall is done and then repeated in 2 months. This is done to test: a. validity b. reliability c. precision d. assessment

reliability

A patient with metabolic alkalosis would have: a. excess excretion of base through the kidneys b. retention of base by the lungs c. excretion of base by the lungs d. retention of base by the kidneys

retention of base by the kidneys

Which statement best describes the "A" component of the FADE performance improvement model? [MGT:312] a. Choose a specific problem or process to improve b. Review data to better understand the current process c. Create a plan d. Implement and monitor the plan

review the data to better understand the current process

A high protein diet increases the need for: a. riboflavin and water b. thiamin c. water d. ascorbic acid

riboflavin and water

Illness due to Bacillus cereus is due to contaminated: a. rice products and starchy foods b. fruits c. dairy products d. eggs

rice products and starchy foods

When substituted for saturated fats in the diet, which of the following oils will likely lower total cholesterol but also lower HDL cholesterol? a. olive oil b. canola oil c. peanut oil d. safflower oil

safflower oil

Evaporated milk has: a. slightly more than half of the water removed b. added sugar c. a fat content of 3 - 6% d. lactose removed

slightly more than half of the water removed

Lactase is produced in the: a. stomach b. small intestine c. gallbladder d. pancreas

small intestine

When on Dicumarol, avoid increasing your intake of: a. spinach b. beets c. corn d. beef

spinach

Four hours after eating you develop nausea, diarrhea, but have no fever. The cause may be: a. staphylococcus aureus b. clostridium botulinum c. clostridium perfringens d. streptococcus

staphylococcus aureus

The outpatient Dietitian is planning a continuing education program. What should be done first? a. locate the guest speaker b. survey patients to see what interests them c. survey other Dietitian's groups to see what they are doing d. select a topic that has not been offered lately

survey patients to see what interests them

What is fluid seeping from a congealed product called? a. retrogradation b. syneresis c. coagulation d. gelatinization

syneresis

To generate revenue, the bakery department produces a photo sheet of their decorated cakes and distributes it to all units in the hospital. The cafeteria bundles the decorated cake service with a discounted coffee service, which is an example of: [MGT:369] a. transformation. b. output. c. synergy d. permeability of boundaries.

synergy

Which defines the statistical term "mean"? a. the number that occurs most frequently b. the average c. the central tendency d. a value in an ordered set of numbers below and above which fall in equal set of numbers

the average

The best indicator that children profited from a nutrition education program about iron is: a. their parents have increased knowledge about iron b. the children have increased knowledge a bout iron c. the children select more snacks with iron d. the parents have normal hemoglobin levels

the children select more snacks with iron

In a study on custards which would be the dependent variable? a. the amount of milk and sugar in the recipe b. the cooked custard c. the temperature of the oven d. the time of cooking

the cooked custard

When should performance objectives be given to workers? a. several weeks before their evaluation b. they should not be shown to the worker c. the day of the evaluation d. the day they are hired

the day they are hired

Which result is desirable? Before After Experimental group A B Control group C D a. the difference between A and B is more significant than the difference between C and D b. the difference between B and D is statistically significant c. the difference between A and C is more significant than the difference between B and D d. the difference between A and D is more significant than the difference between C and B

the difference between A and B is more significant than the difference between C and D

In a study of the effects of saccharine, who received the placebo? a. the experimental group b. the group receiving saccharine c. the group receiving sugar d. the group which has never been tested before

the group receiving sugar

The distinction between a franchise quick service restaurant (QSR) located in a shopping mall and the same franchise QSR located in a medical facility is that [MGT:367] a. the shopping mall QSR is the only one allowed to make a profit. b. the medical facility QSR can break even but not make a profit. c. the medical facility's QSR purpose is secondary to the purpose of the medical facility. d. the shopping mall QSR can offer lower prices.

the medical facility's QSR purpose is secondary to the purpose of the medical facility.

What is your first concern when deciding how to teach? a. the time you and the students have b. what learning resources are available c. what you are best at d. the needs and abilities of your students

the needs and abilities of your students

The best advantage of self-instruction models is: a. they minimize the time spend with each student b. they allow the student to proceed at their own pace c. they take less time than class instruction d. they are best used with large groups

they allow the student to proceed at their own pace

An increased plasma pyruvate level is an indication of: a. iron deficiency b. riboflavin deficiency c. excess carbohydrate ingestion d. thiamin deficiency

thiamin deficiency

If energy needs increase, which vitamins are needed in higher amounts? a. A, C, folate b. A, C, thiamin c. thiamin, niacin, riboflavin, pantothenic acid d. pyridoxine, cyanocobalamin

thiamin, niacin, riboflavin, pantothenic acid

The following are required for the conversion of pyruvic acid into active acetate? a. thiamin, NAD, oxaloacetic acid b. oxygen, pantothentic acid, vitamin E c. thiamin, niacin, riboflavin, pantothentic acid, NAD d. pantothentic acid, oxaloacetate, citric acid

thiamin, niacin, riboflavin, pantothentic acid, NAD

Coarse texture in a cake is due to: a. too little baking powder b. over-mixing c. too much baking powder or sugar d. oven temperature too high

too much baking powder or sugar

When taking an anticoagulant, avoid the following: a. thiamin supplements b. vitamin E supplements above 400 IU per day c. supplements of calcium and vitamin D d. niacin supplements

vitamin E supplements above 400 IU per day

The best way to prevent the spread of food-borne illness is to: a. wash hands regularly b. cook all foods thoroughly c. purchase food from trusted vendors d. prepare food shortly before serving

wash hands regularly

The end products of cellular oxidation are: a. water, carbon dioxide, energy b. carbon, hydrogen, nitrogen c. indigestible fiber, nitrogen d. monosaccharides, amino acids, energy

water, carbon dioxide, energy

Metabolites of the Kreb's cycle are: a. water, energy, carbon dioxide b. energy and pyruvic acid c. electrolytes and water d. lactic acid and glycogen

water, energy, carbon dioxide

The colon functions in the absorption of: a. amino acids, fatty acids b. unsaturated fatty acids, electrolytes c. water, salts, vitamin K, thiamin, robflavin d. glucose and fatty acids

water, salts, vitamin K, thiamin, robflavin

According to Chinese beliefs, foods and herbs that may be used to treat illnesses are grouped in: a. yin (bright, hot) / yang (dark, cold) b. yin (dark, cold) / yang (bright, hot) c. action / maintenance d. harmful / beneficial

yin (dark, cold) / yang (bright, hot)


Kaugnay na mga set ng pag-aaral

SB Introduction to Equilibrium & Equilibrium Expressions

View Set

TestOut IT Fundamentals - Chapter 1-8

View Set

Biology 12 - Reproductive System

View Set